Kỹ thuật sử dụng các định lý nội suy giải các bài toán đa thức - Nguyễn Văn Mậu

303 21 0
Kỹ thuật sử dụng các định lý nội suy giải các bài toán đa thức - Nguyễn Văn Mậu

Đang tải... (xem toàn văn)

Tài liệu hạn chế xem trước, để xem đầy đủ mời bạn chọn Tải xuống

Thông tin tài liệu

Khi đẳng thức cuối cùng trong giả thiết của định lý trên bị phá vỡ, ta cần phải điều chỉnh và có thêm giả thiết đối với hàm số đã cho để kết luận của Định lý dạng Karamata tương tự vẫn c[r]

(1)

Mục lục

Lời nói đầu

Chương 1 Một số dạng khai triển đồng thức 7 1.1 Một số tính chất hàm số

1.2 Một số đồng thức dạng đại số - lượng giác 14

1.3 Một số đẳng thức biến đổi dãy số 31

1.4 Tính tốn tập số ngun đa thức nguyên 56

Chương 2 Các toán nội suy cổ điển 79 2.1 Khai triển nội suy Taylor 80

2.2 Bài toán nội suy Lagrange 98

2.3 Nội suy Newton khai triển Taylor - Gontcharov 106

2.4 Bài toán nội suy Hermite 108

2.5 Bài toán nội suy Lagrange - Newton 118

2.6 Bài toán nội suy Newton - Hermite 120

Chương 3 Nội suy theo yếu tố hình học biểu diễn hàm 124 3.1 Nội suy theo nút điểm dừng đồ thị 124

3.2 Hàm số chuyển đổi tam giác 127

3.3 Biểu diễn đa thức theo hệ nghiệm nguyên hàm 138

3.4 Dạng nội suy tính chất hàm lồi, lõm bậc cao 145

3.5 Biểu diễn số lớp hàm số 161

Chương 4 Nội suy bất đẳng thức 172 4.1 Nội suy bất đẳng thức bậc hai đoạn 172

4.2 Tam thức bậc tuỳ ý hàm phân thức quy 181

4.3 Chuyển đổi điều chỉnh số theo thứ tự dần 186

4.4 Một số mở rộng định lý Jensen 194

4.5 Nội suy bất đẳng thức lớp hàm đơn điệu 204

(2)

Chương 5 Một số ứng dụng nội suy xấp xỉ hàm số 227

5.1 Tính chất đa thức lượng giác 227

5.2 Đa thức Chebyshev 232

5.3 Ước lượng đa thức 235

5.4 Xấp xỉ hàm số theo đa thức nội suy 246

5.5 Một số toán đa thức nhận giá trị nguyên 251

Chương 6 Bài toán nội suy cổ điển tổng quát 263 6.1 Bài toán nội suy cổ điển tổng quát 263

6.2 Bài toán nội suy Taylor mở rộng 271

6.3 Bài toán nội suy Lagrange mở rộng 274

6.4 Bài toán nội suy Newton mở rộng 276

6.5 Bài toán nội suy Hermite mở rộng 278

Chương 7 Các tốn nội suy dãy số 281 7.1 Khơng gian đại số dãy số 281

7.2 Đạo hàm dãy số 285

7.3 Phép tính sai phân tính chất 286

7.4 Nội suy dãy số 289

(3)

Lời nói đầu

Các tốn nội suy vấn đề liên quan đến phần quan trọng đại số giải tích toán học Các học sinh sinh viên thường phải đối mặt với nhiều dạng tốn loại khó liên quan đến chun đề Các tốn nội suy có vị trí đặc biệt tốn học khơng đối tượng để nghiên cứu mà cịn đóng vai trị cơng cụ đắc lực mơ hình liên tục mơ hình rời rạc giải tích lý thuyết phương trình, lý thuyết xấp xỉ, lý thuyết biểu diễn,

Trong hầu hết kỳ thi học sinh giỏi quốc gia, thi Olympic Toán khu vực quốc tế, thi Olympic sinh viên trường đại học cao đẳng, toán liên quan đến nội suy (thường dừng lại nội suy Lagrange khai triển Taylor) hay đề cập thường thuộc loại khó Các tốn khai triển, đồng thức, ước lượng tính giá trị cực trị tổng, tích tốn xác định giới hạn biểu thức cho trước thường có mối quan hệ nhiều đến tốn nội suy tương ứng

Các toán nội suy đặc biệt tập ứng dụng công thức nội suy chúng thường đề cập giáo trình sách tham khảo đại số giải tích tốn học Các tốn nội suy chuyên đề chọn lọc cần thiết cho giáo viên học sinh Hệ Chuyên Toán bậc trung học phổ thông năm đầu bậc đại học chuyên đề cần nâng cao cho bậc sau đại học

Để đáp ứng cho nhu cầu bồi dưỡng giáo viên bồi dưỡng học sinh giỏi chuyên đề các toán nội suy ứng dụng, viết sách nhỏ nhằm cung cấp tài liệu vấn đề liên quan đến nội suy số vấn đề ứng dụng liên quan Đồng thời cho phân loại số dạng toán nội suy bất đẳng thức theo nhận dạng thuật toán giải

Trong tính tốn, nhiều ta cần phải xác định giá trị hàm sốy=f(x)

tại điểmx∈Rtuỳ ý cho trước, cho biết số giá trị hàm số đạo hàm đến cấp số điểm xki ∈R cho trước, tức ta biết liệu

f(sk)(x

ki) =aki, (1)

(4)

Đối với số trường hợp khác biểu thức f(x) biết thường cho dạng biểu thức phức tạp, việc thực phép tính biểu thức củaf(x) gặp nhiều khó khăn Đối với trường hợp vậy, người ta tìm cách xây dựng hàm sốP(x)đơn giản hơn, thường đa thức, thỏa mãn điều kiện (1), tức là:

P(sk)(x

ki) =aki, xki, aki∈Rcho trước vàk, i, sk∈N

Ngồi ra, giá trịx∈Rmàxkhơng trùng vớixki P(x)≈f(x)(xấp xỉ theo độ xác đó)

Hàm sốP(x) xây dựng theo cách gọi làhàm nội suy f(x), điểmxki thường gọi cácmốc nội suyvà toán xây dựng hàm P(x) gọi bài toán nội suy

Sử dụng hàm nội suyP(x), ta dễ dàng tính giá trị tương đối xác hàm số f(x) x ∈Rtuỳ ý cho trước Từ tính gần giá trị đạo hàm tích phân R Vì đa thức đại số hàm số đơn giản nhất, nên trước tiên ta nghĩ đến việc xây dựng P(x) dạng đa thức đại số

Các toán nội suy cổ điển đời sớm, khởi đầu công trình tốn học Lagrange, Newton, Hermite, Tuy nhiên, việc xây dựng toán nội suy tổng quát thuật tốn tìm nghiệm việc xây dựng lý thuyết nội suy nói chung nhiều nhà toán học tiếp tục nghiên cứu phát triển theo hướng khác

Lý thuyết toán nội suy cổ điển số vấn đề liên quan đến đặc trưng hàm tính đơn điệu, tính lồi, lõm, tính tuần hoàn, mảng kiến thức quan trọng thường khó khó chương trình tốn giải tích

Mục tiêu sách nhằm cung cấp cho bạn đọc số kiến thức lý thuyết phương pháp nội suy theo thang liên tục để từ bất đẳng thức nhận cho ta khẳng định tính chất hàm quan trọng tính đơn điệu, tính lồi, lõm, tính tuần hồn, cung cấp cách nhìn tổng qt thứ tự bậc thang (liên tục) bất đẳng thức

Ngoài chuyên đề bản, sách cịn trình bày ngắn gọn hệ thống tốn nội suy chương trình giải tích tốn học bậc cuối phổ thơng năm thứ bậc đại học Hy vọng sách chuyên đề giúp ích nhiều việc sáng tác hệ thống tập phù hợp với trình độ đối tượng học sinh, giúp ích việc bồi dưỡng sinh viên học sinh khiếu tốn học

(5)

Có thể nói tốn nội suy cổ điển đóng vai trò quan trọng việc thiết lập đa thức thỏa mãn hệ điều kiện ràng buộc đặc biệt Việc nghiên cứu toán nội suy nhằm để giải toán ước lượng hàm số tập Tuy nhiên, trường trung học phổ thơng lý thuyết tốn nội suy cịn mẻ bỡ ngỡ giáo viên giảng dạy mơn tốn học Chính vậy, để đáp ứng cho nhu cầu giảng dạy học tập, viết tài liệu nhỏ Đây chuyên đề có ý nghĩa thực tiễn cơng việc giảng dạy, cho ta nhìn nhận quán toán nội suy toán giá trị ban đầu giá trị biên tương ứng giải tích biến

Cuốn sách gồm phần mở đầu chương

Chương 1 Một số dạng khai triển đồng thức đa thức Chương 2 Các toán nội suy cổ điển

Chương 3 Nội suy theo yếu tố hình học biểu diễn hàm số Chương 4 Nội suy bất đẳng thức

Chương 5 Một số ứng dụng nội suy xấp xỉ hàm số Chương 6 Bài toán nội suy cổ điển tổng quát

Đây giảng mà tác giả giảng dạy cho học sinh sinh viên đội tuyển thi Olympic toán quốc gia quốc tế tài liệu nghiệp vụ cho đồng nghiệp, nghiên cứu sinh học viên cao học quan tâm đến lý thuyết toán nội suy giải tích

Ngồi ra, chúng tơi đưa vào xét số vấn đề liên quan đến hệ thống ứng dụng toán nội suy cách tiếp cận phương pháp nhằm giúp độc giả hiểu sâu sở cấu trúc lý thuyết toán nội suy

Một số dạng ví dụ tập chọn lọc đề kỳ thi học sinh giỏi quốc gia Olympic quốc tế Một số tốn minh hoạ khác trích từ tạp chí Kvant, Mathematica, Crux, sách giáo khoa sách giáo trình giải tích, đề thi học sinh giỏi quốc gia quốc tế số đề thi Olympic sinh viên năm gần

(6)

Cuốn sách thuộcTủ sách chuyên tốndành cho học sinh giỏi mơn Tốn bậc trung học phổ thông, sinh viên học viên cao học, thầy giáo giáo tham gia chương trình bồi dưỡng học sinh giỏi

Tác giả xin bày tỏ lòng cảm ơn sâu sắc tới PGS TS Nguyễn Minh Tuấn, PGS TS Trần Huy Hổ, TS Trần Hữu Nam đọc thảo cho nhiều ý kiến đóng góp q báu cho sách hồn chỉnh Tác giả vô biết ơn bạn đọc có ý kiến đóng góp nội dung cách thức trình bày sách Mọi góp ý gửi địa : Nhà xuất Giáo dục, 81 Trần Hưng Đạo, Hà Nội

(7)

Một số dạng khai triển đồng nhất thức

Trong chương trình Tốn bậc phổ thơng hành, tốn tính giá trị biểu thức, chứng minh đồng thức bất đẳng thức chiếm thời lượng lớn Các toán tính giá trị biểu thức gắn liền với kỹ vận dụng hệ thức công thức biến đổi quen biết Đối với bạn đọc làm quen với việc khảo sát tính chất hàm số thường liên tưởng dạng toán với đồng thức quen biết dạng đẳng thức đáng nhớ, dạng đồng thức Lagrange, dạng khai triển hàm số quen biết khai triển Taylor, khai triển Abel,

1.1 Một số tính chất hàm số

Ta nhắc lại số khái niệm tính chất giải tích hàm số Giả sử D ⊂ R tập hợp số giả sử ứng với số xD, theo một quy luật hồn tồn xác định đó, đặt tương ứng số nhấty người ta nói trênD cho hàm (ánh xạ đơn trị) ký hiệu

y =f(x), xD hay xf(x), xD.

Định nghĩa 1.1. Đại lượng biến thiên x được gọi đối số hay biến độc lập và tập hợp D (xD) tương ứng gọi tập xác định hay miền xác định của hàm sốy =f(x) Đại lượng biến thiêny thường gọi hàm số hay đại lượng phụ thuộc tập hợpDtương ứng (yD) gọi tập giá trị hay miền giá trị hàm số dã cho.

Khi muốn mô tả hàm số quy luật người ta thường dùng ký hiệuf (g, h, ), ký hiệu f(x) dùng để đại lượngy mà theo quy luật

(8)

f tương ứng với giá trị xD Về sau, cho hàm số y = f(x) ta sử dụng ký hiệu Df miền xác định fEf miền giá trị f Đôi ta viết Ef = f(Df) nói f ánh xạ Df lên Ef; ảnh Ef =f(Df)⊂D∗,D∗ ⊂R thìf ánh xạDf vàoD

Hàm f g gọi đồng (bằng nhau) Df =Dg đẳng thức f(x) =g(x) thoả mãn với giá trị đối sốx∈ Df

Từ khả thực phép tính số học đại số khác

R cho phép ta mở rộng phép tính hàm số để thu hàm từ hàm cho

Ví dụ 1.1. Tổng hai hàm f g được hiểu hàm ký hiệu là f +g xác định điều kiện sau đây

(i)Df+g =Df ∩ Dg;

(ii) (f +g)(x) =f(x) +g(x) ∀x∈ Df+g.

Cũng theo cách tương tự, ta định nghĩa tíchαf vớiα =const , tích (số học) f g,và thươngf /gcủa hàm fg hiểu hàm xác định điều kiện: (iii)Df /g=Dfx∈ Dg :g(x)6= ,

(iv)

f

g

(x) = f(x)

g(x), ∀x∈ Df /g

Ngồi phép tính số học đại số thực hàm người ta xét phép hợp hàm số

Ví dụ 1.2. Hàm gf (đọc là: g hợp với f) xác định công thức (gf)(x) =

g(f(x))ứng với mọi x để biểu thức vế phải có nghĩa, gọi hàm hợp của f vàg.

Biểu thức g(f(x))có nghĩa nếu x∈ Df vàf(x)∈ Dg Từ đó Dg◦f =

x∈ Df :f(x)∈ Dg . Dễ dàng chứng minh rằngh◦(gf) = (hg)◦f.

Nhận xét 1.1. Nói chung, phép hợp hai hàm số khơng giao hốn, tức fg6=

gf Chẳng hạn, ta xét hàm f(x) = 2x g(x) =x+ 1 Khi đó

(fg)(x) =f[g(x)] = 2g(x) = 2(x+ 1) = 2x+ 2,

(gf)(x) =g[f(x)] =f(x) + = 2x+ 1. Ta nêu vài ví dụ cụ thể số hàm số đặc biệt Ví dụ 1.3 (Hàm Direchlet). Xét hàm số

y=D(x) =

(

0 nếux là số vô tỷ,

(9)

Hàm xác định trên Rvà tập hợp (miền) giá trị gồm hai số 1.

Ví dụ 1.4. Xét hàm số y = [x] hay y = E(x), ký hiệu [x] là phần nguyên số x hay xác số nguyên lớn không vượt quá x

([x]6x < [x] + 1).Hàm xác định trên Rvà tập hợp giá trị là Z

(tập hợp số nguyên).

Ví dụ 1.5. Xét hàm phần phân y = {x} (= x−[x]) Đó phần phân của x. Trong khoảng [n, n+ 1)thì đồ thị hàm y trong toạ độ vng góc 0xy là đoạn thẳng lập với trục 0x góc 45◦.

Ví dụ 1.6. Hàm xác định dấu (dương âm) số

y=sign x=

    

+ nếu x >0,

0 nếu x= 0,nếu x <0.

(Thuật ngữ "sign" có nguồn gốc từ tiếng Latinh, signum - có nghĩa dấu) Hàm này xác định trên Rvà tập giá trị là −1, 1.

Định nghĩa 1.2. f được gọi hàm tăng thực (đồng biến) nếu ∀x1, x2∈ Df, x1< x2⇒f(x1)< f(x2) f được gọi tăng (tăng theo nghĩa rộng, không giảm) nếu

∀x1, x2 ∈ Df, x1 < x2 ⇒f(x1)6f(x2).

Hàm giảm thực (nghịch biến) hàm giảm (giảm theo nghĩa rộng, không tăng) định nghĩa tương tự vàf gọi hàm đơn điệu thuộc bốn lọai hàm liệt kê

Ví dụ 1.7. Hàm y= sinx tăng đoạn

h

π

2,

π

2

i

Thật vậy

∀x1, x2∈

h

π

2,

π

2

i

, x1 < x2, ta có

sinx2−sinx1 = cos

x1+x2

2 sin

x2−x1

2 · (1.1)

π

2 6x1, x26

π

2, nên

π

2 <

x1+x2

2 <

π

2 0<

x2x1

2

π

2. Do đó, cả

hai thừa số vế phải của (1.1)đều dương đoạn

h

π

2,

π

2

i

(10)

Ví dụ 1.8. Hàm y= [x]là hàm tăng theo nghĩa rộng.

Thật vậy, [x] là số ngun lớn khơng vượt q x Do nếu x1 < x2 thì [x1] < x2 và vậy [x1] [x2], nếu m x1 < x2 < m+ thì

[x1] = [x2] =m ứng với m∈Z.

Định nghĩa 1.3. Hàm g được gọi hàm ngược hàm f nếu đồng thời xảy ra đồng thức gf =eDf fg=eDg, đó e(x)≡x.

Hàm ngược củaf ký hiệu làf−1

Điều kiện đầu có nghĩa x ∈ Dff(x) ∈ Dg (vì Ef ⊂ Dg) và g(f(x)) = x (vì DfEg) Điều kiện thứ hai giải thích theo ý nghĩa tương tự

Nhận xét hàm đơn điệu thực (đồng biến nghịch biến) có hàm ngược ánh xạ thực hàm đơn trị - Tuy nhiên, tính đơn điệu thực (đồng biến nghịch biến) điều kiện đủ để tồn hàm ngược điều kiện cần

Thật vậy, tồn hàm khơng đơn điệu lại có hàm ngược, chẳng hạn hàm

x xác định tậpR\ {0}có hàm ngược

Thơng thường, để chứng minh hàm khơng có hàm ngược ta cần có hai số phân biệtx1, x2 cho f(x1) =f(x2)

Ví dụ 1.9. Hàm y = x2, x ∈ R khơng có hàm ngược với x1 = −3 ∈ R x2 = ∈R thì f(−3) =f(3) = 9 Nhưng ta xét y =x2, x ∈ R+ thì có hàm ngược là y=√x.

Thật vậy, ứng vớix1, x2∈R+, x1 6=x2,ta cóy1 =x21, y2 =x22.Suy y1y2 =x21−x22= (x1x2)(x1+x2)6= 0.

Về sau, ta thường sử dụng kết sau

Định lý 1.1. Hàm ngược hàm tăng thực (đồng biến) hàm tăng thực Tương tự, hàm ngược hàm giảm thực (nghịch biến) hàm giảm thực sự.

Chứng minh. Giả sử f hàm tăng thực Ta cần chứng minh y1, y2∈ Df−1 vày1 < y2 suy raf−1(y1)< f−1(y2) Thật vậy, giả sử ngược lại,

f−1(y1) > f−1(y2). Do f đồng biến nên f(f−1(y1)) >f(f−1(y2)) hayy1 >y2,

(11)

Định nghĩa 1.4. Hàm f với tập xác định Df, gọi bị chặn trên tập Df nếuf(Df) là tập hợp bị chặn trên, tức là

∃M : f(x)6M, ∀x∈ Df.

Tương tự,f được gọi bị chặn tập Df nếu tập hợp f(Df)bị chặn dưới, tức là ∃m: f(x)>m, ∀x∈ Df.

Khi f(x) đồng thời vừa bị chặn bị chặn tập Df thì ta nói nó bị chặn (bị chặn hai phía).

Từ định nghĩa ta thấy hàmf bị chặn trênDf ∃M >0 : |f(x)|6M, ∀x∈ Df. Ví dụ 1.10. Hàm f(x) = x

x2+ 1, x∈R bị chặn (trên toàn trục thực R). Thật vậy, ta có

x2x+ 1

= |x|

x2+ 1

1

2, ∀x∈R.

Nhận xét rằng, hàm f không bị chặn với sốM (M > 0) tuỳ ý, tồn x∈ Df cho |f(x)|> M Nói cách ngắn gọn hàm f không bị chặn

∀M >0, ∃x∈ Df : |f(x)|> M. Ví dụ 1.11. Hàm số f(x) =

x2,x ∈R\ {0} không bị chặn. Thật vậy, giả sửM số dương tùy ý Ta có

x2 > M ⇔ |x|<

1

M, x

6

= Vậy, ta lấy x =

2

M thu bất đẳng thức

1

x2 = 4M > M Từ nhận xét vừa nêu suy hàm cho không bị chặn

Vậy hàm xác định nhờ phép tính số học thực R

(cộng, trừ, nhân, chia) Trong số đó, hàm dạng đơn giản đề cập chương trình tốn hàm sau

Hàm luỹ thừa với số mũ nguyênf(x) =xn,n∈Zvà hàm đa thức f(x) =a0xn+a1xn−1+· · ·+an.

Hàm hữu tỷ f(x) = P(x)

Q(x), đóP Qlà đa thức

(12)

ngược ta sử dụng định nghĩa ký tự quen thuộc sách giáo khoa phổ thông hành

Tiếp theo ta xét tính liên tục giới hạn hàm số f cho tập hợp D⊂R

Định nghĩa 1.5. Sốađược điểm tụ tập hợpA⊂Rnếu lân cận (mở) của chứa điểm của A.

Ví dụ 1.12. 0 điểm tụ tập

n1

n

o

.

Định nghĩa 1.6. Giả sử alà điểm tụ tập hợp D Số A được gọi giới hạn của hàm f tại điểm a nếu lân cận tuỳ ý U(A) của điểm A đều tồn tại lân cận thủng Ω(˙ a) của điểma sao cho f( ˙Ω(a)∩D)⊂Λ(A).

Ta sử dụng ký hiệu A= lim

x→af(x); f(x)→A (xa). Như

A= lim

x→af(x) ⇔ ∀Λ(A)∃

˙

Ω(a) : x ∈Ω(˙ a)∩Df(x)∈Λ(A). Vì lân cận điểm chứa ε-lân cận chính điểm ấy, nên ta phát biểu định nghĩa sau tương đương với định nghĩa 1.6

Định nghĩa 1.7. A= lim

x→af(x) nếu

∀ε >0, ∃δ >0 : xD∩Ω(˙ a;δ) ⇒ f(x)∈Λ(A, ε). (1.2) Nói cách khác

A= lim

x→af(x) nếu

∀ε >0 ∃δ >0 : xD, 0<|x−a|< δ ⇒ |f(x)−A|< ε. (1.3) Ta chứng minh định nghĩa 1.6 tương đương với định nghĩa 1.7 Giả sửAlà giới hạn hàmf theo định nghĩa 1.6 Ta cần chứng minh A giới hạn hàmf theo định nghĩa 1.7

Thật vậy, giả sử cho sốε >0 Ta cần số δ >0đòi hỏi định nghĩa 1.7 Ta đặtΛ(A) = (Aε, A+ε) sử dụng định nghĩa 1.6

(13)

tức

∀x∈D∩Ω(˙ a) ⇒ f(x)∈(Aε, A+ε). (1.4) Giả sửΩ(a) = (k, `) Vìalà điểm (k, `) nên ak >0,`a > Ta lấyδ = min(ak, `a) cần chứng minh số cần tìm

Rõ ràng δ > Ngồi ra, từ bất đẳng thức |x−a| < δ ta suy x∈Ω(a) Do

xD, 0<|x−a|< δxD∩Ω(˙ a). Từ (1.4) suy ra|f(x)−A|< ε.

Ngược lại, giả sửAlà giới hạn hàm f theo định nghĩa 1.7 Ta cần chứng minh A giới hạn f theo định nghĩa 1.6

Thật vậy, giả sử cho trước lân cận Λ(A) điểm A Khi đó ∃ε >0sao choε-lân cậnΛ(A, ε)⊂Λ(A)

Alà giới hạn củaf theo định nghĩa 1.7, nên với sốε >0đã nêu∃δ =δ(ε)

sao cho xD∩Ω(˙ a, δ)thì|f(x)−A|< ε Điều có nghĩa rằng ∀x∈D∩Ω(˙ a, δ) ⇒ f(x)∈Λ(A, ε)⊂Λ(A)

hayf(x)∈Λ(A).Nhưng xlà điểm Ω(˙ a, δ)∩Dnên f( ˙Ω(a, δ)∩D)⊂Λ(A).

Ví dụ 1.13. (i) Nếu f(x) =c=const xD∩Ω(˙ a) thì

lim

x→af(x) =c.

(ii) Giả sử D=R, f(x) = x−2, a= 5 Khi ta có

lim

x→5f(x) = 3.

Ta cần chứng minh rằng∀ε >0, ∃δ >0 (ở lấyδ =ε), ta ln có

0<|x−5|< δ hay|f(x)−3|< ε.

Thật vậy, nếu|x−5|< δ =εthì hiển nhiên

|f(x)−3|=|(x−2)−3|=|x−5|< ε. (iii) D=R\ {−1}, f(x) = x

x+ 1,a= Ta chứng minh

x x+ →

1

(14)

Giả sửε >0 Ta cần lân cậnΩ điểm (tức khoảng

(a, b)màa <1< b) cho

x∈Ω(1)˙ ∩D

x+ 1x

2

< ε. Từ đồ thị hàmf, vớiε

0,1

2

thì khoảng

1 −ε +ε

, +εε

= Ω

là lân cận cần tìm

Thật vậy, hàmf tăng khoảng (−1,+∞)nên

2 −ε +ε

< x < +εε

. Suy

1

2 −ε=f

1 2−ε 2+ε

< f(x).

1.2 Một số đồng thức dạng đại số - lượng giác Nhận xét đẳng thức để dẫn đến phong phú hệ thống đồng thức lượng giác công thức

sin2t+ cos2t= 0, ∀t∈R. (1.5)

Gắn với hệ thức (1.5) đồng thức Lagrange

(2x)2+ (1−x2)2= (1 +x2)2, ∀x∈R. (1.6) Hai công thức (đồng thức) (1.5) (1.6) hai cách viết hệ thức Nếu ta thayx= tan t

2 vào (1.6) dễ dàng thu (1.5) ngược lại Như

là công thức lượng giác tương ứng với đồng thức đại số tương ứng Điều thật dễ hiểu nhớ lại trình dẫn dắt đến định nghĩa hàm số lượng giác góc nhọn mơ tả dựa theo Định lý Pytago:

Trong tam giác vuông ABC với cạnh huyền BC ta ln có hệ thức

(15)

Tuy nhiên, với số lượng công thức biến đổi lượng giác nhiều, thân hệ thức lượng giác tạo thành chun đề có tính độc lập tương đối, dần tách hẳn sở đại số nó, làm cho quên lượng lớn hệ thức đại số có xuất sứ từ hệ thức lượng giác quen biết Đặc biệt, chương trình tốn bậc phổ thơng nay, hàm số lượng giác ngược, hàm lượng giác hyperbolic, không nằm phần kiến thức bắt buộc toán liên quan đến chúng thách thức lớn học sinh giáo viên

Ta nhắc lại công thức Euler quen biết

eiα= cosα+isinα, α∈R.

Khi 

   

cosα = e

+e−iα

2 ,

sinα = e

e−iα

2i .

Rõ ràng khảo sát hàm số cost nghĩ đầu có dạng

1

a+1

a

akhơng cịn số thực Nhưng ta ý đến biểu thức

+e−α

2 , α∈R,

thì cos() (= coshα) vậy, mặt hình thức, ta có nhiều biến đổi thu từ cơng thức liên quan đến biếnx6∈[−1,1]giống công thức hàmcost.

Ví dụ 1.14. Hệ thức đại số ứng với cơng thức

cos 2t= cos2t−1

chính công thức

1

a2+

a2

=

h1

2

a+1

a

i2 −1. Ví dụ 1.15. Hệ thức đại số ứng với cơng thức

cos 3t= cos3t−3 cost chính cơng thức

1

a3+

a3

= 4h1

a+1

a

i3 −3h1

2

a+1

a

i

(16)

hay

4x3−3x=

a3+

a3

với

x=

a+1

a

, a6= 0. Ví dụ 1.16. Hệ thức đại số ứng với công thức

cos 5t+ cost= cos 3tcos 2t chính cơng thức

1

a5+

a5

+1

a+1

a = h1

a3+

a3

ih1

2

a2+

a2

i

.

Từ ví dụ trên, sử dụng kết khai triển hàm lượng giáccos 3tvàcos 2t, ta thu đồng thức đại số dạng bậc

1

a5+

a5

=−m+ 2(4m3−3m)(2m2−1),

m=

a+1

a

.

Ví dụ 1.17. Cho số thực m với|m|>1 Tính giá trị biểu thức M = 8x3−6x,

trong đó

x=

q3

m+pm2−1 +

q

m−pm2−1. Giải Để ý rằng, do|m|>1nên tồn số thực q để có hệ thức

m=

q3+

q3

. Ta cần chọn

q=

q

m+pm2−1 đủ Khi

1

q+1

q

=

q3

m+pm2−1 +

q

m−pm2−1=x. Theo Ví dụ 1.15

(17)

Ví dụ 1.18. Khơng dùng máy tính, tìm giá trị góc nhọn x thoả mãn

cosx= q

1 + (

6 +

2− √

3−2)2 . Giải. Xét A= √ + √ 2− √

3−2. Ta có

A= (

3− √

2)(

2−1) =

3− √

2

2 + ,

hay A= √ − √ 2 √ 2 + = cosπ

6 −cos

π

4 sinπ

4 −sin

π

6

= tan π 24.

Vậy nên

1 +A2 = + tan2 π 24 =

1 cos2 π

24

. Suy

cosx=

r

cos2 π

24= cos

π

24,

hay

xπ

24+k2π, k∈Z.

Dox góc nhọn nên x= π 24.

Cách 2.Từ hệ thức cho

cosx= q

1 + (

6 +

2− √

3−2)2 ,

ta thu

1 + (

6 +

2− √

3−2)2 =

cos2x = + tan 2x. Do

tan2x= (

6 +

2− √

3−2)2, hay

tanx=

6 +

2− √

(18)

Tiếp theo ta sử dụng hệ thức góc nhân đơi hàm số tang hàm số cosin, ta thu cơng thức tính góc nhọnx.

Bây ta chuyển sang xét hệ thức đại số liên quan đến hàm sốsint Từ công thức Euler, ta thu hệ thức

isint= e

ite−it

2 .

Từ suy biểu thức isin(it) nhận giá trị thực Điều gợi ý cho ta cách chuyển đổi đồng thức hàm số sin sang đồng thức đại số

Ví dụ 1.19. Xét cơng thức khai triển

sin 3t= sint−4 sin3t. Từ ta thu cơng thức (hình thức)

isini(3t) = 3(isinit) + 4(isinit)3. Hệ thức đại số ứng với công thức đồng thức

1

a3− a3

= 3h1

a−1 a

i

+4h1

a−1 a

i3 , hay

4x3+ 3x=

a3− a3

với

x=

a−1 a

, a6= 0. Ví dụ 1.20. Xét cơng thức biến đổi

sin 5t+ sint= sin 3t(1−2 sin2t). (1.7) Ta viết lại công thức (1.7)dưới dạng

isini(5t) +isinit= 2isini(3t)(1 + 2(isinit)2. Hệ thức đại số ứng với cơng thức đồng thức

1

a5− a5

+1

aa

= 2h1

a3− a3

ih

1 +1

aa2

2i .

(19)

Ví dụ 1.21.

1

a5− a5

=−m+ 2(4m3+ 3m)(2m2+ 1), trong đó

m=

a−1 a

.

Ví dụ 1.22. Cho số thực m Tính giá trị biểu thức M =x3+3

4x,

trong đó

x=

q3

m+pm2+ +

q

m−pm2+ 1. Giải Để ý rằng, với mọim tồn số thực q để

m=

q3− q3

. Ta cần chọn

q=

q

m+pm2+ 1 đủ Khi

1

q−1 q

=

q3

m+pm2+ +

q

m−pm2+ 1=x. Theo Ví dụ 1.19

4x3+ 3x=m nên M =

4m.

Từ kết nhận được, ta giải biện luận nhiều dạng phương trình đại số bậc cao cơng thức tính giá trị số biểu thức chứa thức

Ví dụ 1.23. Dễ dàng giải biện luận phương trình

4x3−3x=m, m∈R

(20)

Với|m|61, ta đặtm= cosα (= cos(α±2π)) Sử dụng đẳng thức lượng giác

cosβ= cos3 β

3 −3 cos

β

3,

ta thu nghiệm phương trình x1 = cos

α

3; x2,3 = cos

α±2π

3 .

Tương tự, khi|m|>1, ta sử dụng đẳng thức đại số tương ứng Đặt m=

2

a3+

a3

với

a=

q

m±pm2−1. Ta viết phương trình cho dạng

4x3−3x=

a3+

a3

hay

4x3−3x= 4x30−3x0,

x0 =

a+1

a

.

Vậy phương trình cho có nghiệmx=x0 Dễ thấy nghiệm phương trình

Thật vậy, phương trình cho có nghiệmx0 vớix0 6∈[−1; 1] Do đó|x0|>1 Khi đó, từ hệ thức

4x3−3x= 4x30−3x0, ta thu

(xx0)[4x2+ 4xx0+ 4x20−3] = 0. Để ý phương trình

4x2+ 4x0x+ 4x20−3 = (1.8)

có ∆0 = 12−12x2

0 <0 phương trình (1.8)vơ nghiệm Do phương trình cho có nghiệm

x=

q3

m+pm2−1 +

q

(21)

Ví dụ 1.24. Bằng phương pháp tương tự, ta giải biện luận phương trình dạng

4x3+ 3x=m, m∈R.

Dễ thấy phương trình cho có nghiệmx=x0 nghiệm phương trình Thật vậy, vớix > x0 4x3+ 3x >4x30+ 3x0 =m vớix < x0 thì4x3+ 3x <4x30+ 3x0=m Đặt

m=

a3− a3

với

a3=m±pm2+ 1 nghiệm phương trình cho

x =

a−1 a

hay

x=

q3

m+pm2+ +

q

m−pm2+ 1.

Tiếp theo, ta xét số ứng dụng khác đẳng thức đại số - lượng giác Bài toán 1.1. Chứng minh phương trình

64x6−96x4+ 36x2−3 =

có nghiệm thực x0 thỏa mãn điều kiện

q

2 +p2 +

2

2 < x0 <

q

2 +p2 +

3

2 .

Giải. Từ công thứccos2α= + cos 2α

2 (với06α6π),ta suy ra

cosα =

v u u

t1 + cosα2

2 =

v u u u t1 +

r

1 + cosα

2

2 =

1

q

2 +

2 + cosα. Khi α= π

4,ta có

cos π 16=

1

r

2 +

q

2 +

(22)

Khi α= π 6,ta có

cos π 24=

1

r

2 +

q

2 +

3. Mặt khác, ta có

cos 6t= cos32t−3 cos 2t

= 4(2 cos2t−1)3−3(2 cos2t−1) = 32 cos6t−48cos4t+ 18cos2t−1

Suy

64 cos6t−96cos4t+ 36cos2t−3 = cos 6t−1. Từ phương trình 64x6−96x4+ 36x2−3 = 0,ta xét x∈[1; 1] đặt

x= cost, ta có cos 6t−1 = 0⇒cos 6t=

2 ⇒t=

π

18.

Do đóx0 = cos π

18 nghiệm phương trình Mặt khác

π

24 <

π

18 <

π

16

hay

cos π 24>cos

π

18 >cos

π

16.

Vậy phương trình 64x6−96x4+ 36x2−3 = ln có nghiệm x0 = cos π 18

thỏa mãn điều kiện

q

2 +p2 +

2

2 < x0 <

q

2 +p2 +

3

2 .

Bài tốn 1.2. Cho phương trình x3−px2+qxp= 0với p, q là số dương. Chứng minh phương trình cho có ba nghiệm phân biệt x1, x2, x3 lớn hơn 1, ta ln có

p>

4 +

2

!

(q+ 3). (1.9)

Giải. Giả sửx1 < x2 < x3 Theo định lý Vieete, ta có

    

(23)

Đặt x1 = tanA, x2 = tanB, x3 = tanC với A, B, C ba góc tam giác π

4 A, B, C <

π

2. Không giảm tổng quát, ta giả sử A = min{A, B, C}

π

4 6A6

π

3.

Khi (1.9) tương đương với

tanAtanBtanC> +

2

8 (tanAtanB+ tanBtanC+ tanCtanA+ 3)

⇔ 1> +

2

8 (cotA+ cotB+ cotC+ cotAcotBcotC)

⇔ 8−4

2>cotA+ cotB+ cotC+ cotAcotBcotC. (1.10) Để chứng minh (1.10), ta ý

cotA+ cotB+ cotC+ cotAcotBcotC

= cotA+ sinA

cos(BC) + cosA + cotA

cos(BC) + cos(B+C) cos(BC)−cos(B+C)

6 cotA+ sinA

1 + cosA + cotA

1−cosA

1 + cosA,

cotA= tanA =

1

2 tanA 1−tan2A

2 =

1−tan2A 2 tanA

2

,

2 sinA

1 + cosA =

2.2 sinA cos

A

2 cos2 A

2

= tanA

3 cotA1−cosA

1 + cosA =

1−tan2 A 2 tanA

2

2 sin2A 2 cosA

=

1−tan2A 2 tanA

2

tan2 A

=

1−tan2 A

tanA

2 =

tanA −tan

3 A

2

(24)

nên ta có

cotA+ cotB+ cotC+ cotAcotBcotC

6

1−tan2A 2 tanA

2

+ tanA +

tanA −tan

3 A

2

hay

cotA+ cotB+ cotC+ cotAcotBcotC

6

2 tanA

+ tanA

3 2tan

3 A

2

Mặt khác π

4 6A6

ππ A π

6 nên ta có

2−16tanA

1

3.

Xét hàm số

f(t) =−3

2t

3

+ 3t+ 2t, t

2−1;√1

3

. Ta có

f0(t) =−3

2t

2

+ 3−

2t2 =

−9t4 + 6t2−1

2t2 =−

(3t2−1)2 2t2 60, với ∀t6= 0nên f(t) nghịch biến

2−1;√1

3

.Suy f(t)6f(

2−1) = 8−4

2, ta có điều phải chứng minh

Dấu "=" xảy

  

cos(BC) = tanA

2 =

2−1 hay

  

B =C = 3π

A= π

Bài tốn 1.3. Tìm x∈(0; 1)thỏa mãn điều kiện

(25)

Giải.x∈(0; 1)nên ta đặt x= cosα vớiα∈0;π

. Ta có

32 cosα(cos2α−1)(2 cos2α−1)2= 1−

cosα ⇔ −32 cosαsin2αcos22α= 1−

cosα ⇔ sin22αcos22α= 1−cosα

⇔ sin24α= 1−cosα ⇔ cosα= cos 8α. Do đóα= k2π

7 α=

l2π

9 (vớik, l∈Z.)

α

0;π

nên k= 1, l= 1vàl=

Vậy phương trình cho có ba nghiệm thuộc (0; 1)đó x= cos2π

7 ;x= cos 2π

9 ;x= cos 4π

9 .

Bài tốn 1.4. Giải phương trình

x3+p(1−x2)3 =xp2(1−x2). Giải. Điều kiện để biểu thức có nghĩa:−16x61. Đặt x= sinα ( vớiα∈h−π

2;

π

2

i

),thì phương trình trở thành

sin3α+ cos3α=

2 sinαcosα ⇔(sinα+ cosα)3−3 sinαcosα(sinα+ cosα)−

2 sinαcosα= 0. Đặt sinα+ cosα=

2 sinπ +α

=tvới điều kiện |t|6

2. Suy rasinαcosα= t

2−1

2 phương trình trở thành

t3−3t

2−1

2 t

2t

2−1

2 =

t3+

2t2−3t− √

2 =

⇔ (t− √

2)(t+

2−1)(t+

2 + 1) =

Suy t=√2hoặct= 1−√2 |t|6√2 Vớit=

2 √

2 sin

π

4 +α

=

2⇔sin

π

4 +α

(26)

hayα= π

4 +k2π.

α

h −π 2; π i

nên α= π

4 đóx= cos

π

4 =

2 .

Vớit= 1− √

2,suy

sinα+ cosα= 1− √

2

hay

x+p1−x2 = 1−√2, tức

(

x61−√2 1−x2 = (1−

2−x)2 ⇔

(

x61−√2

x2−(1− √

2)x+ (1− √

2) =

x= 1−

2−p2

2−1

2 .

Vậy phương trình cho có nghiệm x=

2 ;x=

1− √

2−p2

2−1

2 .

Bài toán 1.5. Giải phương trình

q

1 +p1−x2[p(1 +x)3−p(1−x)3] = +p1−x2.

Giải. Điều kiện có nghĩa: −1 x 1.Đặt x = cost( với t ∈[0;π]), phương trình trở thành

1 + sinthp(1 + cost)3−p(1−cost)3i= +p1−cos2t hay

s

sin t 2+ cos

t

2

2

s2 cos2 t

2

3 −

s

2 sin2 t

3

= +psin2t (1.11)

t∈[0;π]nên t

2 ∈

h

0;π

i

.Do

sin t

2 60,cos

t

(27)

Vậy nên phương trình (1.11) tương đương với √ sint + cos

t

2 cos

3 t

2−sin

3 t

2

= + sint

2

sint + cos

t

2 cos

t

2 −sin

t × × cost + sin

t

2cos

t

2+ sin

2 t

2

= + sint

2 cost

1 + 2sint

= + sint⇔ √

2 cost(2 + sint) = + sint ⇔ (

2 cost−1)(2 + sint) = 0⇔ cost=

2

2 ⇔x=

2

Vậy phương trình có nghiệm làx=

2 .

Bài toán 1.6. Cho số dương x, y, z thỏa mãn điều kiện x+y+z = xyz. Chứng minh rằng

p

(1 +y2)(1 +z2)−p1 +y2−√1 +z2 yz

+

p

(1 +z2)(1 +x2)−√1 +z2−√1 +x2 zx

+

p

(1 +x2)(1 +y2)−√1 +x2−p1 +y2

xy = 0.

Giải. Đặt

x= tanα, y= tanβ, z = tanγ với

α, β, γ ∈0;π

. Dox+y+z=xyz,nên

tanα+ tanβ+ tanγ= tanαtanβtanγ ⇔ tanα+ tanβ = tanγ(tanαtanβ−1)

⇔ tanα+ tanβ

1−tanαtanβ =−tanγ

α+β =−γ+α+β+γ=kπ,(k∈Z)

Doα+β+γ

0;3π

,suy ra0< kπ < 3π

(28)

Vậy nênα+β+γ=π.Ta suy

p

(1 +y2)(1 +z2)−p1 +y2−√1 +z2 yz

=

p

(1 + tan2β)(1 + tan2γ)−p1 + tan2β−p1 + tan2γ

tanβtanγ

= cosβ

1 cosγ

1 cosβ

1 cosγ sinβ cosβ sinγ cosγ

= 1−(cosβ+ cosγ) sinβsinγ . Tương tự, ta có

p

(1 +z2)(1 +x2)−√1 +z2−√1 +x2

zx =

1−(cosγ+ cosα) sinγsinα

và p

(1 +x2)(1 +y2)−√1 +x2−p1 +y2

xy =

1−(cosα+ cosβ) sinαsinβ . Khi vế trái đẳng thức cần chứng minh

1−(cosβ+ cosγ) sinβsinγ +

1−(cosγ+ cosα) sinγsinα +

1−(cosα+ cosβ) sinαsinβ

= sinα+ sinβ+ sinγ−sin(α+γ)−sin(α+β)−sin(β+γ)

sinαsinβsinγ = 0,

điều phải chứng minh

Bài toán 1.7. Cho xy6= 1, yz6= 1, zx6= 1 Chứng minh rằng xy

1 +xy + yz

1 +yz + zx

1 +zx = xy

1 +xy. yz

1 +yz. zx

1 +zx. Giải. Đặtx= tanα, y= tanβ, z= tanγ,với α, β, γ

π 2; π

.Khi xy

1 +xy + yz

1 +yz + zx

1 +zx

= tanα−tanβ + tanαtanβ +

tanβ−tanγ

1 + tanβtanγ +

tanγ−tanα

1 + tanαtanγ

= tan(αβ) + tan(βγ) + tan(γα)

xy

1 +xy. yz

1 +yz. zx

(29)

Ta chứng minh đồng thức

tan(αβ) + tan(βγ) + tan(γα) = tan(αβ) tan(βγ) tan(γα). Thật vậy, ta có

tana+ tanb

1−tanatanb = tan(a+b)⇒tana+ tanb=

tan(a+b) 1−tanatanb. Vậy nên

tan(αβ) + tan(βγ) + tan(γα)

= tan(αβ+βγ)[1−tan(αβ) tan(βγ)] + tan(γα) = tan(αγ)−tan(γα) + tan(αβ) tan(βγ) tan(γα) = tan(αβ) tan(βγ) tan(γα)

Do

xy

1 +xy + yz

1 +yz + zx

1 +zx = xy

1 +xy. yz

1 +yz. zx

1 +zx, điều phải chứng minh

Bài toán 1.8. Cho x1, x2, x3 là nghiệm phương trình

x3+ax2+x+b= (1.12)

b6= 0.Chứng minh rằng

(x1−

1

x1)(x2−

1

x2) + (x2−

1

x2)(x3−

1

x3) + (x3−

1

x3)(x1−

1

x1) = 4. Giải.x1, x2, x3 nghiệm phương trình (1.19) nên

(

x1x2+x2x3+x3x1 =

x1x2x3 =b6=

hay (

x1x2+x2x3+x3x1 =

x1, x2, x3 6= Đặt x1 = tanα, x2= tanβ, x3= tanγ vớiα, β, γ∈(−π

2;

π

2)

Ta có

tanαtanβ+ tanβtanγ+ tanγtanα=

(30)

Nếu tanαtanβ = 1thìtanα+ tanβ= 0hoặctanγ= 0.

Nếu tanα+ tanβ = 0,thì tanα=−tanβ, đó −tan2β= 1, vơ lý Nếu tanγ= 0thìx3= 0, vơ lý

Vậytanαtanβ 6= 1.Khi từ (1.20),ta có

tanα+ tanβ

1−tanαtanβ.tanγ= ⇔ tan(α+β) = tan(π

2 −γ)

α+β+γ= π

2 +kπ,(k∈Z)

(x1−

1

x1)(x2−

1

x2) + (x2−

1

x2)(x3−

1

x3) + (x3−

1

x3)(x1−

1

x1)

= (tanα−cotα)(tanβ−cotβ) + (tanβ−cotβ)(tanγ−cotγ) + (tanγ−cotγ)(tanα−cotα)

= cot 2αcot 2β+ cot 2γ(cot 2α+ cot 2β) = 4cot 2αcot 2β−4 cot(2α+ 2β)(cot 2α+ cot 2β) (do 2γ=π−(2α+ 2β+k2π))

= cot 2αcot 2β−4cot 2αcot 2β−1

cot 2α+ cot 2β (cot 2α+ cot 2β) Vậy

(x1−

1

x1)(x2−

1

x2) + (x2−

1

x2)(x3−

1

x3) + (x3−

1

x3)(x1−

1

x1) = 4, điều phải chứng minh

Bài toán 1.9. Chứng minh với số tự nhiênn (n>2)và với mọia, ta

−(1 +a2)n6(2a)n+ (1−a2)n6(1 +a2)n. Giải. Bất đẳng thức cần chứng minh tương đương với

−16

2a

1 +a2

n

+

1−a2

1 +a2

n

61

Đặt tanα

2 =tvới −π < α < π, ta có 2a

1 +a2 = sinα;

1−a2

(31)

Bất đẳng thức cần chứng minh có dạng

−16sin+ cos 61. Thật ta có

−16sinα61⇒ −sin2α 6sin6sin2α,với ∀n>2. Tương tự, ta có

−16cosα 61⇒ −cos2α 6cos6cos2α,với ∀n>2. Do đó−16sin+ cos61, tốn chứng minh

1.3 Một số đẳng thức biến đổi dãy số

Ta thường thấy sách giáo khoa giải tích đề cập đến dạng biến đổi số tổng sinh cặp dãy số cho trước dùng để ước lượng tổng hữu hạn chứng minh tính hội tụ chuỗi số dạng đặc biệt Đặc biệt, dãy số hàm số xác định tập số tự nhiên nhận giá trị tập hợp số thực

Trước hết ta xét tính đơn điệu dãy số Tính đơn điệu hàm số đóng vai trị quan trọng dãy số xác định hàm số

Về sau ta thường dùng kết sau

Bổ đề 1.1. Cho dãy số {xn} (n= 1,2, ) được xác định theo công thức xn=f(xn−1), ∀n>2.

Giả sửxn∈[a, b],∀n∈N f là hàm đồng biến trên[a, b] Khi đó i) Nếux1 < x2 thì {xn} là dãy đơn điệu tăng.

ii) Nếux1 > x2 thì {xn} là dãy đơn điệu giảm. Chứng minh.

i) Vì x1 < x2 nên f(x1)< f(x2)⇔ x2 < x3 Từ đó, quy nạp toán học ta dễ dàng chứng minh xn < xn+1, ∀n∈ N Suy {xn} dãy đơn điệu tăng

ii) Khi x1> x2 thìf(x1)> f(x2)⇔x2 > x3 Từ đó, quy nạp tốn học ta chứng minh

xn> xn+1, ∀n∈N.

(32)

Bài toán 1.10. Xét dãy số{vn} (n= 0,1, ) được xác định công thức v0 = 1, vn= −1

3 +vn−1 với n= 1,2, Chứng minh rằng{vn} là dãy đơn điệu giảm.

Giải. Nhận xét dãy cho có giới hạn giới hạn dãy nghiệm phương trình v = −1

3 +v, tức v = −3±

5

2 Ta chứng minh

vn > −3 + √

5

2 với mọin ∈N Thật vậy, vớin= n = ta thấy Với

vk >

−3 +√5

2 +vk >

3 +√5

2

1 +vk <

2 + √ = 3− √ .

Do đóvk+1> −3 +

5

2

Ta chứng minh dãy {vn} dãy đơn điệu giảm Thật vậy, ta có vnvn+1=vn+

1 +vn

= v

2

n+ 3vn+

3 +vn

>0.

Nhận xét 1.2. Có thể dựng tường minh số hạng tổng quát dãy Bài toán 7.4 để từ có kết luận cần chứng minh.

Thật vậy, ta đặt

w0= 1, wn+1 = wn

vn, n= 0,1, thì

wn+1+ 3wn+wn−1 = 0. Từ đây, suy ra

wn=

5 +

−3 +√5

2

n

+1−

5

−3−√5

2

n . Vậy nên

vn= wn wn+1

=

5 +

−3 +√5

n

+1−

5

−3−√5

n

5 +

−3 +√5

2

n+1

+1−

5

−3−√5

2

(33)

Bài toán 1.11. Xét dãy số{vn} (n= 1,2, ) được xác định bởi v1=α, vn+1=

−a

b+cvn, n= 1,2, , trong đó

a, b, c∈R+,∆ :=b2−4ac >0, α > −b+ √

∆ 2c . Chứng minh rằng{vn} là dãy đơn điệu giảm.

Giải. Nhận xét rằng, dãy cho có giới hạn giới hạn dãy nghiệm phương trình v= −a

b+cv, tức làv= −b±

∆ 2c Ta chứng minh rằngvn>

−b± √

2c với mọin nguyên dương Thật vậy, với n= 1ta thấy mệnh đề vìv1 =α > v Giả sử mệnh đề tớin=k−1, tứcvk−1 > v Khi đóv(vk−1v)<0và vvk−1 < v2 hay

cvvk−1+bv+a < cv2+bv+a.

vlà nghiệm phương trìnhcx2+bx+a= 0nêncvvk−1+bv+a <0 Do cvvk−1+bv+a

b+cvk−1 <0 hay

a

b+cvk−1 +v <0. Suy vk > v, điều phải chứng minh.

Ta chứng minh {vn} dãy đơn điệu giảm Thật vậy, ta có vnvn+1=vn+ a

b+cvn =

cvn2+bvn+a b+cvn .

Dovn> v vớiv nghiệm lớn tam thức bậc haif(x) =cx2+bx+a, nên cvn2+bvn+a >0

vnvn+1= cv

2

n+bvn+a b+cvn

(34)

Bài toán 1.12. Cho k số dương (k>2) thoả mãn điều kiện a1+a2+· · ·+ak >k.

Đặt

un=an1 +an2 +· · ·+ank, n= 1,2, Chứng minh rằng{un} là dãy giảm.

Giải. Nhận xét với số dươnga, ta có

(an−1)(a−1)>0. Suy

an+1an>a−1. Từ suy

un+1un= (an+11 +an+12 +· · ·+an+1k )−(an1 +an2 +· · ·+ank)>u1k. Theo giả thiết u1k>0, nên un+1un>0.Vậy {un}là dãy giảm Bài toán 1.13. Xét dãy số{un} (n= 1,2, ) được xác định công thức

un=

n

X

k=1

(−1)k

k , k= 1,2, ,

Chứng minh rằng{u2n} là dãy đơn điệu tăng và {u2n+1} là dãy đơn điệu giảm. Giải. Nhận xét

f(x) :=

x+ +

n

X

i=1

(−1)ixi−1 = (−1)

n xn

1 +x .

Suy f(x)>0 với mọix >0 vànchẵn Do vậy, nguyên hàm F(x) = ln(1 +x) +

n

X

i=1

(−1)ixi i

là hàm đồng biến trong(0,+∞)ứng vớinchẵn F(x)>F(0) = 0,∀x >0.

Tương tự,f(x)<0 với mọix >0 vànlẻ Do vậy, nguyên hàm F(x) = ln(1 +x) +

n

X

i=1

(35)

là hàm nghịch biến trong(0,+∞)ứng với nlẻ F(x)6F(0) = 0,∀x >0.

Vậy vớix= 1thì F(1) = ln2−u2n>0 vàF(1) = ln2−u2n+1 60 Mặt khác, ta có

u2k+2=u2k+ 2k+ 1−

1

2k+ > u2k,

nên dãy{u2k}là dãy đơn điệu tăng bị chặn ln2 Tương tự, ta có

u2k+3 =u2k+1+

1 2k+ 2−

1

2k+ < u2k+1,

nên dãy {u2k+1} dãy đơn điệu giảm bị chặn ln2 Do đó, dãy cho có giới hạn

Bài toán 1.14. Cho dãy số{xn} được xác định sau xn=

2

xn−1+ a

xn−1

với n>2, a >0, x1 >0. Chứng minh rằng{xn} là dãy giảm.

Giải. Ta có

x2 =

1

x1+ a x1

>√a >0.

Do phép quy nạp theo bất đẳng thức trung bình cộng trung bình nhân, ta nhận xn>√a,tức{xn} bị chặn √a.

Mặt khác, ta có

xn xn−1 =

1 2+

a

2x2n−1 xn−1 >√a Suy

xn xn−1

1 +

a

2a = 1. Do đóxn6xn−1 hay{xn} dãy giảm

Bài toán 1.15. Cho dãy số{yn} được xác định sau yn=

3

2yn−1+ a

yn−12

(36)

Giải. Tương tự Bài toán 7.3, dùng bất đẳng thức trung bình cộng trung bình nhân phép quy nạp toán học, ta thu

yn=

1

yn−1+yn−1+ a y2n−1

>√3a.

Mặt khác

yn yn−1

6

3 + =

nên

yn6yn−1. Vậy, dãy số{yn}là dãy giảm

Bài toán 1.16. Cho dãy số {xn} với < xn <1 xn+1(1−xn)>

1

4 Chứng

minh dãy cho dãy đơn điệu tăng bị chặn. Giải. Vì0< xn<1, ∀n∈N, nên dãy cho bị chặn Mặt khác xn+1(1−xn)>

1

4 >xn(1−xn) do0< xn<1nên xn+1 > xn

Do đó, dãy cho đơn điệu tăng bị chặn

Bài toán 1.17. Cho dãy số{yn} xác định theo công thức yn+1 = (1−x)yn+ Ax

y

1−x x

n

, với A >0, 0< x <1, y0 >0, n∈N. Hãy chứng minh dãy số dãy giảm.

Giải. Xét hàm số

f(y) = (1−x)y+ Ax

y1−xx

với y >0, A >0, 0< x <1. Lấy đạo hàm theo y, ta được

f0(y) = (1−x)(1−Ay−x1),

nên

f0(y) = 0⇔y=Ax.

Vậyf(y)>Ax, ∀y >0.Theo công thức tính yn, từ y0 >0, ta cóyn>0với n. Mặt khác, doyn=f(yn−1),∀n>1 nên yn>Ax, ∀n>1.

Xét hiệu

ynyn−1=x.y

x−1

x

n−1(Ay

1

x

(37)

Vậy, dãy số{yn}giảm dần y0 bị chặn bởiAx

Tiếp theo, xét hệ thức liên hệ đại lượng trung bình cộng trung bình nhân bộnsố khơng âm Với dãy số không âm {xk}, ta ký hiệu

An=

a1+a2+· · ·+an

n , Gn=

n

a1a2 an hay

An=

n n

X

i=1

xi, Gn= n

v u u t

n

Y

i=1 xi. Từ đẳng thức quen biết

tnnt+n−1 = (t−1)[tn−1+tn−2+· · ·+t−(n−1)], n∈N∗, đồng thức Jacobsthal

An= Gn−1

n

h

(n−1)An−1

Gn−1 +

Gn

Gn−1

ni

. (1.14)

ta suy

G

n Gn−1

n

>n Gn Gn−1

+ 1−n. Từ ta suy

An> Gn−1

n

h

(n−1)An−1

Gn−1

−(n−1) +n Gn Gn−1

i

hay

AnGn> n−1

n (An−1Gn−1), n >1.

Ta thu bất đẳng thức quen biết giá trị trung bình cộng (trung bình số học) trung bình nhân (trung bình hình học)An>Gn.

Bài tốn 1.18. Chứng minh điều kiện cần đủ để dãy số {an} lập thành một cấp số cộng dãy cho phải thoả mãn hệ thức

2am+n =a2m+a2n, ∀m, n∈N. (1.15) Giải. Điều kiện cần Giả sử dãy {an} cấp số cộng với công sai d. Khi

an=a0+ (n−1)d, ∀n∈N∗. Vậy nên

(38)

2am+n= 2[a0+ (m+n−1)d]. (1.17) Từ (1.16)và(1.17), ta có ngay(7.36)

Điều kiện đủ Giả sử dãy{an}thoả mãn điều kiện(7.36) Ta chứng minh dãy {an}là cấp số cộng với công sai d=a1a0 Thaym = 0vào (7.36), ta

2an=a0+a2n. Suy

a2n= 2ana0. (1.18)

Thay (1.18)vào (7.36),ta thu

2am+n= 2am+ 2an−2a0 hay

am+n=am+ana0. (1.19)

Thay m= 1vào(1.19),ta có

an+1 =an+d, d=a1−a0. Từ đó, ta thu {an} cấp số cộng

Bài toán 1.19. Chứng minh điều kiện cần đủ để dãy số dương {an} lập thành cấp số nhân dãy cho phải thoả mãn hệ thức

a2m+n=a2ma2n, ∀m, n∈N. (1.20) Giải. Đặt lnan=bn với mọin∈N Khi đóan=ebn và(1.20)có dạng

e2bm+n =eb2m+b2n, ∀m, n∈N

hay

2bm+n=b2m+b2n, ∀m, n∈N.

Đây điều kiện cần đủ để dãy số {bn} lập thành cấp số cộng với cơng said=b1−b0 Từ suy điều phải chứng minh

Bài toán 1.20. Cho dãy số{an} thoả mãn hệ thức a2n =an−1an+1, an6= 0, ∀n∈N∗. Chứng minh rằng

an1 +an2 +· · ·+ann an2 +an3 +· · ·+ann+1 =

(39)

Giải. Từ giả thiết, ta có hệ thức a1

a2

= a2

a3

=· · ·= an

an+1

=P

an1 an2 =

an2

an3 =· · ·= ann ann+1 =

an1 +an2 +· · ·+ann an2 +an3 +· · ·+ann+1 =P

n . Mặt khác, ta có

Pn = a1

a2 ·a2

a3

· · · an an+1

= a1

an+1 . Từ đó, ta thu điều phải chứng minh

Bổ đề 1.2. Giả sử hàm số f(x) thoả mãn điều kiện f(√xy) = f(x) +f(y)

2 , ∀x, y >0.

Khi hàm f(x) chuyển đổi cấp số nhân dương thành cấp số cộng, tức là với cấp số nhân {an} (an>0, ∀n∈N thì dãy {f(an)} là cấp số cộng. Chứng minh. Từ giả thiết, ta có hệ thức

a0

a1 =· · ·= an−1

an = an

an+1 =· · ·

a2n=an−1an+1, an>0, ∀n∈N∗

hay

f(an) =f( √

an−1an+1) =

f(an−1) +f(an+1)

2 .

Từ đó, ta thu điều phải chứng minh

Bổ đề 1.3. Giả sử hàm số f(x) thoả mãn điều kiện f

x+y

2

=pf(x)f(y), ∀x, y∈R.

(40)

Chứng minh. Từ giả thiết, ta có hệ thức

a1a0 =· · ·=anan−1 =an+1an=· · ·

2an=an−1+an+1, ∀n∈N∗ hay

f(an) =f

an−1+an+1

2

=pf(an−1)f(an+1).

Từ đó, ta thu {f(an)}là cấp số nhân Bổ đề 1.4. Giả sử hàm số f(x) thoả mãn điều kiện

fxy= 2f(x)f(y)

f(x) +f(y) , ∀x, y∈R

+.

Khi hàm f(x) chuyển đổi cấp số nhân dương thành cấp số điều hoà, tức là với cấp số nhân {an} (an > 0, ∀n ∈ N, thì dãy {f(an)} là cấp số điều hoà.

Chứng minh. Từ giả thiết, ta có hệ thức a1

a0 =· · ·= an an−1 =

an+1 an =· · ·

a2n=an−1an+1, ∀n∈N∗ hay

f(an) =f

an−1an+1

= 2f(an−1)f(an+1)

f(an−1) +f(an+1) .

Từ đó, ta thu điều phải chứng minh

Nhận xét 1.3. Đối với dãy số {un} xác định theo công thức truy hồi un+1 =aun+b, a, b∈R,

(41)

Bài toán 1.21. Cho dãy số{un} là cấp số suy rộng thoả mãn điều kiện un+1 =aun+b, a, b∈R.

Chứng minh rằng

un=an−1u0+ (an−2+an−3+· · ·+a+ 1)b, n>2. (1.21) Giải. Thật vậy, (1.25) với n = Giả sử (1.25) với n=k Khi đó, theo giả thiết quy nạp

uk+1 =auk+b=a[ak−1u0+ (ak−2+ak−3+· · ·+a+ 1)b] +b

=aku0+ (ak−1+ak−2+· · ·+a+ 1)b. Vì vậy,(1.16)đúng với n>2

Nhận xét 1.4. Khi a= thì un=u0+ (n−1)b và khi a6= thì an−2 +an−3+· · ·+a+ = a

n−1−1

a−1 , n>2,

nên

un=an−1u0+

an−1−1

a−1 b.

Bài toán 1.22. Cho dãy số{un} là cấp số suy rộng thoả mãn điều kiện un+1 =aun+b, a, b∈R.

Tính tổng

Sn=u1+u2+· · ·+un. Giải. Ta có

u2+u3+· · ·+un+1 =a(u1+u2+· · ·+un) +nb

Sn+1=u1+u2+u3+· · ·+un+1 =u1+a(u1+u2+· · ·+un) +nb. Mặt khác

un+1nbu1= (u1+u2+· · ·+un)(a−1). Vậy nên, nếua6= 1thì

Sn=

(42)

Sử dụng phương pháp quy nạp, ta chứng minh

Sn=

anu1+ an−1

a−1 bnbu1

a−1 .

Nếu a= 1thì{un}là cấp số cộng với công sai b Do vậy Sn=

(u1+un)n

2 =

[2u1+ (n−1)b]n

2 .

Tương tự hàm số thông thường, ta coi dãy số {xn} hàm f(n) = xn xác định tập N nhận giá trị R Ta quan tâm đến hai loại dãy tuần hồn tuần hồn cộng tính tuần hồn nhân tính

Ví dụ 1.25. Dãy{un} tuần hồn (cộng tính) chu kỳ dãy có dạng un=

2[α+β+ (αβ)(−1)

n+1

], α, β∈R. (1.22) Thật vậy, giả sửu0 =α, u1 =β un+2 =un,∀n∈N Khi ta thấy (bằng quy nạp tốn học) dãy {un} có dạng (1.22) Ngược lại, dãy xác định theo (1.22) dãy tuần hồn chu kỳ

Bài tốn 1.23. Cho k ∈ Q\Z Chứng minh dãy số {un} xác định theo công thức

u0 = 1, u1=−1, un+1 =kunun−1, n∈N∗ khơng dãy tuần hồn.

Giải. Khi|k|>2

|un+1|>|k||un| − |un−1|>2|un| − |un−1|

Nếu ln ln xảy ra|un|<|un−1|với mọin∈N∗ ta có điều phải chứng minh Nếu xảy ra|um|>|um−1|>0,thì suy

|um|<|um+1|<· · · dãy{un} khơng dãy số tuần hoàn

Xét|k|62 vớik= p

q,(p, q) = 1,26q ∈Z

, p∈Z Bằng quy nạp theonta thu

uj = pj

(43)

Từ suy

un+1 = p

qunun−1 = pn+1

qn ,

pn+1=ppnq2pn−1 ∈Z

và (pn+1, q) = Doq>2nên un=6 um n6=m dãy{un} không dãy số tuần hồn

Bài tốn 1.24. Xác định giá trị của k ∈ Q để dãy số {un} xác định theo công thức

u0 = 1, u1=−1, un+1 =kunun−1, n∈N∗

là dãy số tuần hoàn.

Giải. Theo kết Bài toán 5.17, khi|k|>2và|k|62,k= p

q với(p, q) = 1,

26q ∈Z∗ dãy {un} khơng dãy số tuần hồn Xét|k|62 k∈Z

Với k= thì{un} cấp số cộng với công sai −2 nên hiển nhiên dãy {un} khơng dãy tuần hồn

Vớik= 1thì{un} dãy tuần hoàn chu kỳ :

u2=−2, u3 =−1, u4 = 1, u5 = 2, u6= 1, u7=−1, Vớik= 0thì{un} dãy tuần hồn chu kỳ :

u0= 1, u1 =−1, u2 =−1, u3 = 1, u4= 1, u5=−1, Vớik=−1thì {un} dãy tuần hồn chu kỳ :

u0 = 1, u1 =−1, u2 = 0, u3 = 1, u4 =−1, Vớik=−2thì {un} dãy tuần hoàn chu kỳ :

u0 = 1, u1 =−1, u2 = 1, u3 =−1, u4 = 1,

Bài toán 1.25. Cho f(x) là đa thức với degf =k >1, f(x) ∈Z ứng với mọi x ∈ Z Ký hiệu r(k) = min{2s|s∈ N∗, 2s > k} Chứng minh dãy số {(−1)f(k)} (k= 1,2, ) là dãy tuần hồn với chu kỳ r(k).

Giải. Ta cók!f(x)∈Z[x] Biểu diễn f(x) dạng f(x) =a0+a1

x

1

+· · ·+ak

x k

(44)

trong x k

= x(x−1)· · ·(xk+ 1)

k! .

Ta cần chứng minh f(x+r(k))−f(x) chia hết cho2 với mọix∈Z Nhận xét

Mi=

x+ 2s

i

x i

chia hết cho với mọii∈N∗, 2s>i, x∈Z Thật vậy, ta có Mi =

1

i!

h

(2s+x)(2s+x−1) .(2s+xi+ 1)−x(x−1) .(xi+ 1)

i

. Tử số hiển nhiên chia hết cho2s Mặt khác, số mũ khai triển củai!

X

j=1

h i

2j

i

<

X

j=1 i

2j =i62 s

,

nên Mi chia hết cho2 với mọii∈N∗, i62s, x∈Z Từ suy Ti =

x+r(k)

i

x i

chia hết cho với mọii∈Z, i6k, ∀x∈Z Doaj ∈Z nên f(x+r(k))−f(x) =

k

X

j=0 ajTj

chia hết cho 2, điều phải chứng minh

Bài toán 1.26. Xác định dãy số{un} thoả mãn điều kiện

u2n+1= 3un, ∀n∈N. (1.23) Giải. Đặtn+ =m,m= 1,2, Khi viết (1.25) dạng

u2m−1 = 3um−1, ∀m∈N∗ hay

v2m= 3vm, ∀m∈N∗ (1.24) với

(45)

Từ (1.24) ta có v0 = Đặtvm =mlog23ym, m∈N∗ Khi (1.24) có dạng

y2m=ym, m∈N∗.

Vậy {ym}là dãy tuần hồn nhân tính chu kỳ Khi theo Bài tốn ta có

yn=

(

tuỳ ý với nlẻ,

y2k+1 với ncó dạng 2m(2k+ 1), m∈N∗, k∈N. Từ suy

um =vm+1 =mlog23ym+1,

với

yn=

(

tuỳ ý với nlẻ,

y2k+1 với ncó dạng 2m(2k+ 1), m∈N∗, k∈N.

Nhận xét tính đơn điệu hàm số đóng vai trị quan trọng dãy số xác định hàm số

Ví dụ 1.26. Cho dãy số {xn} (n= 1,2, ) được xác định theo công thức xn=f(xn−1), ∀n>2.

Giả sửxn∈[a, b],∀n∈N f là hàm đồng biến trên[a, b] Khi đó a) Nếu x1 < x2 thì {xn} là dãy đơn điệu tăng.

b) Nếu x1 > x2 thì{xn} là dãy đơn điệu giảm. Chứng minh.

a) Vìx1 < x2 nên f(x1) < f(x2)⇔x2 < x3 Từ đó, quy nạp tốn học ta dễ dàng chứng minh xn < xn+1, ∀n∈ N Suy {xn} dãy đơn điệu tăng

b) Khi x1> x2 thìf(x1)> f(x2)⇔x2 > x3 Từ đó, quy nạp tốn học ta chứng minh

xn> xn+1, ∀n∈N.

Suy {xn}là dãy đơn điệu giảm

Bài toán 1.27. Xét dãy số{vn} (n= 0,1, ) được xác định bởi v0 = 1, vn=

−1

(46)

Giải. Nhận xét dãy cho có giới hạn giới hạn dãy nghiệm phương trình v = −1

3 +v, tức v =

−3±√5

2 Ta chứng minh

vn >

−3 +√5

2 với mọin ∈N Thật vậy, vớin= n = ta thấy Với

vk > −3 + √

5

2 +vk > +

5

2

1 +vk <

2 +√5 =

3−√5

2 .

Do đóvk+1> −3 +

5

2

Ta chứng minh dãy {vn} dãy đơn điệu giảm Thật vậy, ta có vnvn+1=vn+

1 +vn =

v2

n+ 3vn+

3 +vn >0. Bài toán 1.28. Xét dãy số{vn} (n= 1,2, ) được xác định bởi

v1=α, vn+1= −a

b+cvn, n= 1,2, , trong đó

a, b, c∈R+,∆ :=b2−4ac >0, α > −b+ √

∆ 2c . Chứng minh rằng{vn} là dãy đơn điệu giảm.

Giải. Nhận xét rằng, dãy cho có giới hạn giới hạn dãy nghiệm phương trình v= −a

b+cv, tức làv= −b±

∆ 2c Ta chứng minh rằngvn>

−b± √

2c với mọin nguyên dương Thật vậy, với n= 1ta thấy mệnh đề vìv1 =α > v Giả sử mệnh đề tớin=k−1, tứcvk−1 > v Khi đóv(vk−1v)<0và vvk−1 < v2 hay

cvvk−1+bv+a < cv2+bv+a.

vlà nghiệm phương trìnhcx2+bx+a= 0nêncvvk−1+bv+a <0 Do cvvk−1+bv+a

b+cvk−1

<0

hay

a

(47)

Suy vk > v, điều phải chứng minh.

Ta chứng minh {vn} dãy đơn điệu giảm Thật vậy, ta có vnvn+1=vn+

a b+cvn

= cv

2

n+bvn+a b+cvn

.

Dovn> vvớivlà nghiệm lớn tam thứcf(x) =cx2+bx+a, nêncvn2+bvn+a >

0

vnvn+1=

cv2n+bvn+a b+cvn

>0. Từ suy ra{vn}là dãy đơn điệu giảm

Bài toán 1.29. Cho k số dương (k>2) thoả mãn điều kiện a1+a2+· · ·+ak >k.

Đặt

un=an1 +a n

2 +· · ·+a n

k, n= 1,2, Chứng minh rằng{un} là dãy giảm.

Giải. Nhận xét với số dươnga, ta có

(an−1)(a−1)>0. Suy

an+1an>a−1. Từ suy

un+1un= (an+11 +a n+1

2 +· · ·+a n+1 k )−(a

n

1 +an2 +· · ·+ank)>u1−k. Theo giả thiết u1−k>0, nên un+1un>0.Vậy {un}là dãy giảm Bài toán 1.30. Xét dãy số{un} (n= 1,2, ) được xác định công thức

un= n

X

k=1

(−1)k

k , k= 1,2, ,

Chứng minh rằng{u2n} là dãy đơn điệu tăng và {u2n+1} là dãy đơn điệu giảm. Giải. Nhận xét

f(x) :=

x+ +

n

X

i=1

(−1)ixi−1 = (−1)

nxn

(48)

Suy f(x)>0 với mọix >0 vànchẵn Do vậy, nguyên hàm F(x) = ln(1 +x) +

n

X

i=1

(−1)ixi i

là hàm đồng biến trong(0,+∞)ứng vớinchẵn F(x)>F(0) = 0,∀x >0.

Tương tự,f(x)<0 với mọix >0 vànlẻ Do vậy, nguyên hàm F(x) = ln(1 +x) +

n

X

i=1

(−1)ixi i , hàm nghịch biến trong(0,+∞)ứng với nlẻ

F(x)6F(0) = 0,∀x >0.

Vậy vớix= 1thì F(1) = ln2−u2n>0 vàF(1) = ln2−u2n+1 60 Mặt khác, ta có

u2k+2=u2k+

1 2k+ 1−

1

2k+ > u2k,

nên dãy{u2k}là dãy đơn điệu tăng bị chặn ln2 Tương tự, ta có

u2k+3 =u2k+1+ 2k+ 2−

1

2k+ < u2k+1,

nên dãy {u2k+1} dãy đơn điệu giảm bị chặn ln2 Do đó, dãy cho có giới hạn

Bài toán 1.31. Cho dãy số{xn} được xác định sau xn=

1

xn−1+ a xn−1

với n>2, a >0, x1 >0. Chứng minh rằng{xn} là dãy giảm.

Giải. Ta có

x2 =

x1+ a

x1

>√a >0.

Do phép quy nạp theo bất đẳng thức trung bình cộng trung bình nhân, ta nhận xn>

a,tức{xn} bị chặn √

(49)

Mặt khác, ta có

xn xn−1

= 2+

a

2x2n−1 xn−1 >

a Suy

xn xn−1

6

2 +

a

2a = 1. Do đóxn6xn−1 hay{xn} dãy giảm

Bài toán 1.32. Cho dãy số{yn} được xác định sau yn=

1

2yn−1+ a yn−12

với n>2, a >0, x1>0. Chứng minh rằng{yn} là dãy giảm.

Giải. Tương tự Bài toán 6, dùng bất đẳng thức trung bình cộng trung bình nhân phép quy nạp toán học, ta thu

yn=

yn−1+yn−1+ a

y2 n−1

>√3

a. Mặt khác

yn yn−1

6

3 + =

nên

yn6yn−1. Vậy, dãy số{yn}là dãy giảm

Tiếp theo ta xét phép biến đổi Abel sử dụng để tính số tổng quen biết Phép biến đổi Abel tỏ hiệu lực tính số tổng đặc biệt liên quan đến tổng tích dãy lập thành cấp số

Bổ đề 1.5 (Về biến đổi Abel). Cho hai dãy số {an} {bn} Xét hai dãy số {Bn} {Sn} xác định sau

Bn=

n

X

k=1

bk, Sn=

n

X

k=1 akbk. Khi đó

Sn=

n−1X k=1

(50)

Chứng minh. Ta coiB0 = Vì vậy, ta cóBk−Bk−1 =bkvớik= 1, , n

n−1

X

k=1

(akak+1)Bk+anBn=

n

X

k=1

akBkn−1

X

k=1

ak+1Bk

= a1b1+

n

X

k=2

akBkn

X

k=2

akBk−1 =a1b1+

n

X

k=2

ak(BkBk−1)

= a1b1+

n

X

k=2

akbk =

n

X

k=1

akbk =Sn.

Hệ 1.1. Cho cấp số cộng{an} với công sai dvà cấp số nhân{bn} với công bội q (q 6= 1) Khi đó

Sn=

n

X

k=1

akbk =

h

(1−qn)a1+ qd

1−q(1−nq n−1

+ (n−1)qn)

i b1

1−q· Chứng minh. Nhận xét rằngakak+1 =−d (k= 1,2, , n−1)và

Bn =

n

X

k=1

bn= b1(1−q

n)

1−q .

Từ ta có điều phải chứng minh

Dưới ta xét số ví dụ minh hoạ áp dụng trực tiếp đồng thức Abel

Ví dụ 1.27. Tính tổng

Sn=

n

X

k=1

k.qk−1 (q6= 1).

Theo Định lý biến đổi Abel áp dụng cấp số cộng

1,2,3, , n (d= 1), cấp số nhân

(51)

thì

Sn= [(1−qn).1 + q

1−q(1−nq

n−1+ (n−1)qn)]

1−q

=

1−qn+qnq

n+nqn+1qn+1

1−q

1 1−q

= nq

n+1−(n+ 1)qn+ 1

(1−q)2 . Ví dụ 1.28. Tính tổng

Sn=

n

X

k=1

ksinkx. Đặtak =k, bk = sinkx Khi đó

Bn= n X k=1 bk = n X k=1 sinkx. Suy

Sn= n

X

k=1

ksinkx=− n−1

X

k=1

Bk+nBn

= n−1 X k=1 cosx −cos

(2k+ 1)x)

1 sinx

2

+nsin

n+ 1

2 x sinnx sinx

= sinnx .

n.sinn+

2 x−sin

n−1

2 x

sinx

.

Ví dụ 1.29. Cho cấp số cộng{a1, a2, , an} có cơng sai d Tính tổng Sn=

n

X

k=1

ksinak. Giải. Xét tổng Bn= n X k=1

(52)

Ta có Sn=

n−1

X

k=1

[k−(k+ 1)]Bk+n.Bn=− n−1X k=1

Bk +n.Bn

=

n−1X k=1

cos

a1d

2 + cos a1+

k−1

2

d

2 sind

n

sin

a1+n−1

2 d sin n 2d sind

= −1

2 sind

h

(n−1) cos(a1d

2)−

n−1 X k=1 cos h a1+

k−1

2

d

i

+ 2nsina1+ n−1

2 d

sinn 2d i , n−1 X k=1 cos

a1d

2 +kd

= cos

a1+d +

n−2

2 d

sin(n−1 d) sind

2

.

Nhận xét 1.5. Tương tự, ta tính tổng dạng Tn=

n

X

k=1

kcosak, Un= n

X

k=1

aksinbk, Vn= n

X

k=1

akcosbk với {ak} là cấp số cộng,{bk} là cấp số nhân.

Ví dụ 1.30. Tính tổng Sn=

n

X

k=1

qksin(α+),

Tn=

n

X

k=1

qkcos(α+). Ta có

Tn+iSn= (cosα+i.sinα)

(53)

với ε= cosβ+isinβ Từ suy ra Tn+iSn= (cosα+isinα)

()n+1−1

−1 = (cosα+isinα)(()

n+1−1)(−1)

(−1)(−1) = q

n+2[cos(+α) +i.sin(+α)]−q[cos(α) +i.sin(α)]

1−2qcosβ+q2

+−q

n+1{

cos[(n+ 1)β+α] +i.sin[(n+ 1)β+α]}+ cosα+i.sinα

1−2qcosβ+q2 ,

trong ε¯= cosβisinβ. Vậy nên

Sn=

sinαq.sin(β)−qn+1sin[(n+ 1)β+α] +qn+2sin(+α) 1−2qcosβ+q2

Tn=

cosαq.cos(β)−qn+1cos[(n+ 1)β+α] +qn+2cos(+α)

1−2qcosβ+q2 .

Nhận xét 1.6. Bằng phương pháp tương tự, ta tính tổng Vn=

n

X

k=1

aksin(α+), Un=

n

X

k=1

akcos(α+), Wn=

n

X

k=1

aksinbk Rn= n

X

k=1

akcosbk,

trong đó {ak} là cấp số nhân với công bộiq 6= {bk} là cấp số cộng với công said.

Ví dụ 1.31. Tính giá trị biểu thức

3

r

cos2π

7 +

3

r

cos4π +

3

r

cos8π .

Giải. Nhận xét

xk = cos2

7 +isin 2

7 (k= 0,1, ,6)

là nghiệm phương trìnhx7 = Từ suy xk = cos2

7 +isin 2

(54)

là nghiệm phương trình

x6+x5+· · ·+x+ =

và đồng thời nghiệm phương trình

x+

x

3

+x+

x

2

−2x+1

x

−1 = 0. Từ suy

yk =xk+

1

xk

=xk+xk = cos

2

7 (k= 1,2,3)

là nghiệm phương trìnhy3+y2−2y−1 = Nhưng

cos8π = cos

6π

7

nên ta thay cos6π

7 cos 8π

7

Lập phương trình bậc ba với nghiệm

3

r

2 cos2π ,

3

r

2 cos4π ,

3

r

2 cos8π .

Dựa theo hệ thức nghiệm hệ số phương trình ta dễ dàng tính tổng cần thiết

Một cách tổng quát, α, β, γ nghiệm phương trình x3+ax2+bx+c=

cịn √3α,√3β,√3γ là nghiệm phương trình

x3+Ax2+Bx+C = 0,

(−A)3 = (√3α+p3

β+√3γ)3

=α+β+γ+ 3(√3α+p3

β+√3γ)(p3 αβ+p3 βγ+√3γα)−p3 αβγ

=−a−3AB−3√3

−c, hay

(55)

Tương tự, ta tìm B3 =b+ 3ABC−3C2. Trong trường hợp tốn cho

a= 1, b=−2, c=−1, C =−1.

Do đó, ta có (

A3 = 3AB+ 4,

B3 =−3AB−5. (1.26)

ĐặtAB =zvà nhân vế với vế hai đẳng thức (1.26), ta thu z3+ 9z2+ 27z+ 20 = 0.

Suy raz=

7−3 Do A=p3 33

7−5 Vậy

3

r

2 cos2π

7 +

3

r

4 cos2π +

3

r

8 cos2π

7 =

3

r

1 2(5−3

3

7). Nhận xét 1.7. Bằng phương pháp tương tự, ta có

3

r

cos2π +

3

r

cos4π +

3

r

cos8π

9 = r 2(3 √

9−6),

cos

cosϕ = 1−C ntan

2

ϕ+Cn4tan4ϕ− · · ·+A, trong đó

A=

(

(−1)n2 tan với nchẵn

(−1)n−21Cn−1

n tann−1ϕ với nlẻ

sin

cosϕ =C

ntanϕC ntan

3

ϕ+Cn5tan5ϕ+· · ·+B, trong đó B =       

(−1)

n

2 + 1Cnn−1tann−1ϕ, vớin chẵn

= (−1)

n+

2 tannϕ, vớin lẻ

Bài toán 1.33. Xét dãy số{xn} xác định theo công thức xn= 1khi

h

(n+ 1)√2004

i

h

n√2004

i

là số lẻ

xn= 0khi

h

(n+ 1)

2004i−hn

2004i là số chẵn. Tính

(56)

Giải.

Nhận xét

h√

2004i= 44nên √2004 = 44 +α với0< α <1 vàα số vô tỷ Vậy nênh(n+ 1)√2004i−hn√2004ilà số lẻ [(n+ 1)α]−

[] =

Tương tự,h(n+ 1)√2004i−hn√2004ilà số chẵn khi[(n+ 1)α]−

[] = Suy

xn= 44 + [(n+ 1)α]−[]

S =x1964+x1965+· · ·+x2004= 44.41 + [2005α].

1.4 Tính tốn tập số ngun đa thức ngun Các tốn tính tốn liên quan đến đa thức nhận giá trị nguyên đa thức với hệ số nguyên hệ số hữu tỷ dạng tốn quen thuộc bậc phổ thơng Đặc biệt, tính thường có quan hệ mật thiết với phép tính số học Chẳng hạn, ta có sốp= 247là số ngun tố ta có dạng

p= 2(10)2+ 4(10) +

và ta xét đa thức sinh

P(x) = 2x2+ 4x+

như phép nội suy (thác triển) tự nhiên số p (= P(10) Đa thức có tính chất tương tự sốp khơng phân tích thành tích hai đa thức (khác hằng) với hệ số ngun Chính vậy, sau ta thường quan tâm đến phép tính đa thức phân thức tập số nguyên số tự nhiên phép tínhnội suytự nhiên phép tính số học thơng thường tập số ngun

Bài tốn 1.34. Tìm tất đa thức P(x) bậc nvới hệ số nguyên không âm không lớn và P(9) = 32078.

Giải Giả sử

P(x) =anxn+an−1xn−1 +· · ·+a0. Khi theo giả thiết

(57)

Do0≤ak ≤8 nên a0 số dư phép chia 32078 cho 9, tứca0 = Như a1+ 9a2+· · ·+ 9n−1an = 3564.

Lập luận tương tự ta nhận đượca1 số dư phép chia 3564 cho 9, tứca1 =

a2+ 9a3+· · ·+ 9n−2an= 396.

Tương tự, ta nhận a2 số dư phép chia 396 cho 9, tức a2= 0và a3+· · ·+ 9n−3an= 44.

Tiếp theo, lập luận tương tự ta nhận a3 = a4 = Vậy đa thức cần tìm có dạng

P(x) = 4x4+ 8x3+ 2.

Bổ đề 1.6. Phân số tối giản p

q ((p, q) = 1), là nghiệm đa thức với hệ số nguyên

f(x) =anxn+an−1xn−1+· · ·+a0 thì p là ước củaa0 q là ước của an.

Thật vậy, giả sử phân số tối giản p

q nghiệm đa thứcf(x) Khi ta có f

p

q

=anp n

qn +an−1 pn−1

qn−1 +· · ·+a1 p

q +a0 = 0. Từ đó, ta có

anpn=−q(an−1pn−1+· · ·+a1qn−2p+a0qn−1) (1.27)

a0qn=−p(anpn−1+an−1pn−2q+· · ·+a1qn−1). (1.28) Từ (1.27)suy anpn chia hếtq mà (pn, q) = 1,nên an chia hếtq.

Từ(1.28)suy raa0qn chia hếtpmà(pn, q) = 1,nên a0 chia hết q. Bổ đề 1.7. Phân số tối giản p

q là nghiệm đa thức với hệ số nguyên. f(x) =anxn+an−1xn−1+· · ·+a0

(58)

Phân tíchf(x) theo lũy thừa của(xm),ta thu

f(x) =an(xm)n+bn−1(xm)n−1+· · ·+b1(xm) +b0=ϕ(xm). Nhận xét hệ sốb0, bn−1 số ngun mlà số ngun Ta có f(m) =b0 Thay xbởi p

q,ta thu đẳng thức f p

q

=ϕp qm

=ϕpmq q

= 0. Do pmq

q nghiệm ϕ(x) Theo Bài tốn 1.6 pmq ước b0 =f(m)

Ví dụ 1.32. Phân số tối giản p

q là nghiệm đa thức với hệ số nguyên f(x) =anxn+an−1xn−1+· · ·+a1x+a0,

thì pq là ước của f(1) p+q là ước củaf(−1).

Sử dụng kết bổ đề ứng với m= vàm =−1,ta thu điều phải chứng minh

Bài toán 1.35. Chứng minh đa thứcf(x)với hệ số ngun khơng có nghiệm ngun nếu f(0) f(1)là số lẻ.

Giải Giả sử đa thức f(x) có hệ số nguyên có nghiệm hữu tỷ p

q, p, q∈Zvà(p, q) =

Theo Bài tốn vớim = 0, p=p−0.q ước f(0) vớim = 1thì pq ước củaf(1)

Nhưngf(0) vàf(1) số lẻ nênp pq số lẻ Do p lẻ, nên buộc q phải chẵn đóq 6=±1 Vậy, phân số tối giản p

q nghiệm nguyên f(x)

Từ ta thu điều phải chứng minh

Bài toán 1.36. Cho đa thức bậcn với hệ số nguyên f(x) =anxn+an−1xn−1+· · ·+a0. Chứng minh nếu α là nghiệm nguyên đa thức

ϕ(y) =yn+anyn−1+· · ·+an−2n a1y+an−1n an thì α

an

(59)

Giải Ta có

an−1n f(x) = (anx)n+an−1(anx)n−1+· · ·+an−2n .a1(anx) +an−1n an

=yn+an−1yn−1+· · ·+an−2n a1y+an−1n an

=ϕ(y) với y=anx. Do α nghiệm đa thức ϕ(y) nên dễ dàng suy α

an

là nghiệm f(x) y=anx.

Bài tốn 1.37. Chứng minh khơng tồn đa thức f(x)∈Z[x]màf(m) =

m+ vàf(m+ 2) =m2 ứng với m nguyên đó. Giải Giả sử

f(x) =anxn+an−1xn−1+· · ·+a0, ai ∈Z ∀i∈ {0,1, , n}. Khi đó, ta có

f(m+ 2)−f(m) =an((m+ 2)nmn) +an−1((m+ 2)n−1mn−1) +· · ·+a1((m+ 2)−m)

chia hết cho2 Mặt khác f(m+ 2)−f(m) =m2−m−1là số lẻ nên không chia hết cho Vậy không tồn đa thức f(x)thỏa mãn điều kiện Bài toán 1.38. Giả sử f(x) là đa thức với hệ số nguyên, k l là hai số nguyên nguyên tố Chứng minh nếu f(k) chia hết cho l f(l)

chia hết cho kthì f(k+l) chia hết cho tích k.l

Giải Tương tự Bài tốn??, ta có f(k+l)−f(k) chia hết cho (k+l)−k hayf(k+l)−f(k) chia hết chol Mà f(k) chia hết chol nên f(k+l) chia hết cho l.

Tương tự f(k+l)−f(l) chia hết cho kf(l) chia hết cho k nên suy f(k+l)chia hết cho k.

Do(k, l) = 1nên suy f(k+l) chia hết cho kl. Bài toán 1.39. Cho p là số nguyên tố (p>5) và

f(x) =ax4+bx3+cx2+dx+e∈Z[x]. Chứng minh nếu f(x) chia hết cho p với mọi x∈Z thì

(60)

Giải Ta có

f(x) =ax4+bx3+cx2+dx+e f(0) =enên echia hết cho p.

Mặt khác, ta có

f(1) =a+b+c+d+e p

f(−1) =ab+cd+e p

nên 2(a+c+e) p và2(b+d) p Do(2, p) = 1nên suy ra(a+c) pvà(b+d) p. Tương tự, ta có

f(2) = 16a+ 8b+ 4c+ 2d+e p

f(−2) = 16a−8b+ 4c−2d+e p nên 4a+c p và4b+d p.

Từ suy

a p, b p, c p, d p vàe p.

Bài toán 1.40. Cho f(x) là đa thức với hệ số nguyên Chứng minh rằng nếu f(0), f(1), , f(m−1), không chia hết cho m (m là số nguyên dương cho trước, m>2) phương trìnhf(x) = khơng có nghiệm ngun.

Giải Giả sử phương trìnhf(x) = có nghiệm nguyên c Khi đó f(x) = (xc)g(x), đóg(x)là đa thức với hệ số nguyên Ta có

f(0) = (0−c)g(0), f(1) = (1−c)g(1),

f(m−1) = (m−1−c)g(m−1).

(61)

Bài toán 1.41. Cho đa thức f(x) với hệ số nguyên Chứng minh nếu phương trình f(x) = có nhiều hơn3 nghiệm nguyên phân biệt phương trình f(x) =−1 khơng có nghiệm ngun.

Giải Giả sửa nghiệm ngun phương trình f(x) = −1 Khi f(a) = −1 Gọi x1, x2, x3, x4 nghiệm nguyên phân biệt phương trình f(x) = 1thì

f(x)−1 = (xx1)(xx2)(xx3)(xx4)g(x). Suy

f(a)−1 =−2 = (ax1)(ax2)(ax−3)(ax4)f(a), ax1, ax2, ax3, ax4 là4 số ngun phân biệt

Nhưng−2 khơng thể phân tích tích của4 số ngun khác Vậy phương trìnhf(x) =−1khơng thể có nghiệm ngun

Bài tốn 1.42. Cho đa thức

P(x) =ax3+bx2+cx+d∈Z[x],

trong đó a, b, c, d∈Z c không chia hết cho 3, akhông chia hết cho 3;b không chia hết cho 3. Chứng minh với số tự nhiên n luôn tồn số nguyên an sao cho P(an) không chia hết cho 3n.

Giải Vì c khơng chia hết tồn số r1 ∈ {0,1,2} cho cr1+dkhơng chia hết cho3 TrongP(x)thay xbởi 3x+r1,ta có

P(3x+r1) = 3[9ax3+ (9ar1+ 3b)x2+ (3ar12+ 2br1+c)x+ar13+br12+d1]

với 3d1=cr1+d Do đó P(3x+r1) = 3P1(x),

P1(x) = 9ax3+ (9ar1+ 3b)x2+ (3ar12+ 2br1+c)x+ar31+br12+d1. Dễ thấyP1(x) có dạng giống P(x) Sử dụng thuật toán tương tự với P1(x) ta tìm r2 ∈ {0,1,2}sao cho P1(3x+r2) = 3P2(x) với P2(x) có dạng P(x)

Cứ tiếp tục trình vậy, ta thu dãy r1, r2, , rn∈ {0,1,2} cho

(62)

với Pn(x) có dạng P(x)

Suy với số tự nhiênn luôn tồn

an= 3nx+ 3n−1rn+· · ·+ 3r2+r1 với xnguyên cho P(an)chia hết cho 3n

Bài toán 1.43. Chứng minh đa thức Pn(x) :=

xn n! +

xn−1

(n−1)! +· · ·+

x

1!+ 1, n∈N

khơng thể có nhiều nghiệm thực.

Giải Ta chứng minh vớinchẵn thìPn(x)>0với mọix∈Rvà vớinlẻ Pn(x) có nghiệm thực Ta chứng minh qui nạp

Vớin= 0thìP0(x) = 1>0 với mọix∈RnênP0(x)khơng có nghiệm thực Với n = 1thì P1(x) = +x có nghiệm thực x =−1 Giả sử khẳng định với giá trị nhỏ thuan Ta chứng minh với n.

Vớinlẻ Pn(x) có nghiệm

Pn0(x) =Pn−1(x)>0, ∀x∈R,

nên Pn(x)đồng biến trênRvà phương trìnhPn(x) = 0có khơng q nghiệm thực

Vớinchẵn thìPn0(x) =Pn−1(x)có nghiệm thựcx06= VìPn00(x) = Pn−2(x) > với x ∈ R (do n−2 chẵn, n−2 < n) Do với x < x0 Pn0(x) =Pn−1(x)<0 vàPn0(x) =Pn−1(x)>0 khix > x0

Suy với mọix∈R ta có

Pn(x)>Pn(x0) =Pn−1(x0) + xn0

n! =

xn0 n! >0,

Pn−1(x0) = 0ứng với nchẵn

Vậy điều khẳng định chứng minh Từ ta suy phương trình cho có khơng q nghiệm thực

(63)

Giải Giả sử q(x), q1(x), , qk(x) nhân tử bất khả quy khác P(x) xếp theo thứ tự tăng dần theo bội chúng, nghĩa

P(x) = [q(x)]m[q1(x)]m1· · ·[qk(x)]mk

với m m1 6· · ·6 mk Nhận xét rằngq(x) có vô số ước nguyên tố p cho từ q(n) ≡0 ( mod p) phải có q(n) 6= 0,q1(n)6= ( mod p), ., qk(n) 6= (

mod p) Khi hai số P(n)vàP(n+p) chia hết chopm không chia hết cho luỹ thừa bậc cao củap (điều phải chứng minh)

Tiếp theo ta xét số toán mối liên hệ số nguyên tố tính bất khả quy đa thức liên quan đến

Bài tốn 1.45. Giả sử P(x) là đa thức hệ số nguyên giả sử tồn số n nguyên thoả mãn ba điều kiện sau.

i) Các không điểm đa thứcP(x)nằm nửa mặt phẳng Rez < n−1

2,

ii) P(n−1)6= 0,

iii) P(n) là số nguyên tố. Khi đa thức P(x) là bất khả quy.

Giải Giả sửP(x) đa thức khả quy Khi P(x) =f(x)g(x)vớif(x),g(x) hai đa thức với hệ số nguyên có bậc >1 Do khơng điểm P(x) nằm nửa mặt phẳng Rez < n

2 nên tất không điểm f(x)

nằm nửa mặt phẳng Từ suy

fn

2−t

<

fn−1

2 +t

với t >0. (1.29)

f(n−1) 6= (do P(n−1)6= 0) số nguyên nên |f(n−1)|> f(n)nguyên nên kết hợp với(1.29)(ứng vớit= 1/2), ta có f(n)> f(n−1)>1 Tương tự đối vớig(x) ta cóg(n)>1 Nhưng thìP(n) có ước làf(n)vàg(n), điều mâu thuẫn với giả thiết rằngP(n)là số nguyên tố VậyP(x) bất khả quy

Bài toán 1.46. Giả sử số tự nhiên n chữ số p = a0a1 an là số nguyên tố Chứng minh đa thức tương ứng

(64)

Giải Vớik= 10, thìP(x)có khơng điểm nằm nửa mặt phẳng Rez <

91

2 P(9) 6= P(10) =p số nguyên tố Vậy P(x) đa thức bất

khả quy

Bài toán 1.47. Chứng minh đa thức dạng

P(x) = (xa1)(xa2)· · ·(xan)−1, trong đó a1, a2, , an là số nguyên phân biệt bất khả quy.

Giải Giả sử P(x) đa thức khả quy Khi P(x) = f(x)g(x), f(x), g(x)là đa thức có hệ số nguyên có bậc lớn Ta có

(xa1)(xa2)· · ·(xan)−1 =f(x)g(x).

Do đóf(ak)g(ak) = −1hayf(ak) =−g(ak) =±1với mọik= 1,2, , nKhi đa thức Q(x) =f(x) +g(x)là đa thức có bậc 6n−1 vàf(ak) +g(ak) = nên f(x) +g(x)≡0

P(x) = (xa1)(xa2)· · ·(xan)−1 =−[f(x)]2.

Điều vơ lý hệ số củaxn của đa thức vế trái hệ số của xn ở vế phải>0 Vậy đa thứcP(x) bất khả quy

Bài toán 1.48. Cho a1, a2, , an nsố nguyên phân biệt Chứng minh rằng P(x) = (xa1)2(xa2)2· · ·(xan)2+

là bất khả quy.

Giải Giả sử P(x) khả quy Khi P(x) = f(x)g(x), f(x), g(x) đa thức có hệ số nguyên có bậc lớn Có thể coi hệ số cao f(x) dương Khi f(x), g(x) đa thức dương R f(ak)g(ak) = với mọik= 1,2, , n Giả sử f(x) =xr+· · · vàg(x) =xs+· · · vớis+r= 2n.

Nếu xảy rar < s thìs < n nên g(x)≡1, vơ lý

Nếu xảy rar=s=n thìf(x)−g(x) có bậc 6n−1và triệt tiêu nđiểm ak nên f(x)−g(x)≡0 hayf(x)≡g(x) Do ta có

P(x) = (xa1)2(xa2)2· · ·(xan)2+ = [q(x)]2,

(65)

với mọix∈R Điều xảy Vậy P(x) bất khả quy

Tiếp theo ta xét số dạng toán liên quan đến số nguyên số hữu tỷ Dãy số ngun khơng âm (số tự nhiên) xem phép biến đổi tập N vào Vì vậy, nhiều tính chất số học liên quan đến tính chia hết, đồng dư, nguyên tố nhau, số phương, thường xuất toán dãy số nguyên

Bài toán 1.49. Với số tự nhiên mcho trước, xét dãy{xk(n)}được xác định như sau :

xk(n) = k n+m+ 1C

m+n

2n , n>m.

Tìm k nhỏ cho dãy cho ứng với giá trị k tìm dãy số nguyên với mọi n>m.

Giải. Khin=m

k n+m+ 1C

m+n 2n =

k

2m+ 1.

Do kphải chia hết cho(2m+ 1) Xétk= 2m+ Khi vớin > mthì xk(n) =

k n+m+ 1C

m+n 2n =

2m+

n+m+ 1C

m+n 2n

=1− nm n+m+

C2nm+n=C2nn+mC2nn+m+1 ∈Z.

Vậy k= 2m+ số tự nhiên nhỏ để dãy cho ứng với giá trịk dãy số nguyên với mọin>m.

Bài toán 1.50. Dãy số{xn} được xác định sau : xn=

3 +√5

2

n

+3−

5

n

−2, n∈N. Chứng minh rằngx2k+1 (k∈N) số phương. Giải. Nhận xét

xn=

3 +√5

2

n

+

3−√5

2

n −2 =

h√5 + 1

2

n

√5−1

2

ni2 . Đặt

an=

√5 + 1

2

n

5−1

(66)

Để ý

an+2=

√5 + 1

2

n+2

√5−1

2

n+2

=

5an+1an. Vớia1= 1,a2=

5, quy nạp toán học ta nhận a2k+1 nguyên, a2k có dạngm

5 vớim∈Z Suy x2k+1 vớik∈Nlà số phương Bài toán 1.51. Cho dãy số{xn} được xác định sau :

x1= 7, x2= 50, xn+1= 4xn+ 5xn−1−1975 (n>2). Chứng minh rằngx1996 1997.

Giải. Xét dãy{yn} với y1= 7, y2 = 50và

yn+1 = 4yn+ 5yn−1+ 22, n>2. Dễ thấy ynxn(mod 1997) Do cần chứng minh

y1996≡0(mod 1997).

Đặt zn= 4yn+ 11 Suy raz1 = 39, z2= 211 Nhận xét

zn+1= 4yn+1+ 11 = 16yn+ 20yn−1+ 99 = 4zn+ 20yn−1+ 55. Ta lại có

zn−1= 4yn−1+ 11 suy ra20yn−1 = 5zn−1−55. (1.30) Thế (1.30)vào (??), ta

zn+1 = 4zn+ 5zn−1. Suy

zn+1−4zn−5zn−1 = 0. (1.31) Phương trình đặc trưng của(1.31)là

λ2−4λ−5 = có hai nghiệm λ1 =−1, λ2 = 5. Nghiệm tổng quát (1)là

zn= (−1)+ 5nβ.

Ta có (

(67)

nên α= 3, β =

25

3 .Do đó, ta nhận

zn = 3·(−1)

n

+25 ·5

n

. (1.32)

Từ (1.32)suy

z1996=

8 + 25·51996

3 .

Ta cần chứng minh

z1996≡11(mod1997).

Do 

 

51996−1 1997 51996−1

nên 51996−1 3·1997 Từ đó, ta có 51996= 3n·1997 + 1,và đó, z1996=

8 +

25(3n·1997 + 1)

= +

25

3 + 25·n·1997 = 25·n·1997 + 11. Vậy,z1996≡11( mod 1997).

Bài toán 1.52. Cho dãy số nguyên dương {an} được xác định sau : a0 = 20, a1 = 100, an+2 = 4an+1+ 5an+ 20 với n∈N. Tìm số ngun dương h bé có tính chất

an+han 1998 vớin∈N. Giải. Ta có

an+1 = 4an+ 5an−1+ 20 vớin∈N∗. Đặt bn= 2an+ ta thu b0 = 45, b1= 205

bn+1 = 4bn+ 10an−1+ 25 vớin∈N∗. (1.33) Mặt khác, bn−1= 2an−1+ nên

(68)

Thế (1.34)vào (1.34)ta

bn+1−4bn−5bn−1= 0. (1.35) Phương trình đặc trưng của(1.35): λ2−4λ−5 = 0có hai nghiệm là 5và −1. Nghiệm tổng quát của(1.35)là bn= 5+ (−1)nβ. Chon= vàn= ta tìm

α= 125 , β=

10 .

Do

bn= 125 .5

n

+10 .(−1)

n , an=

125 .5

n+10

6 .(−1)

n−5

2.

Giả sửh số nguyên dương thỏa mãn hệ thức

an+han(mod 1998). (1.36)

Từ a0 = 20, a1 = 100,suy h >2. Ta chứng minh h thỏa mãn (1.36) khih chẵn (h>2)

ah−1 ≡0(mod1998). (1.37)

Điều kiện cần Ta có

ah =a0+h≡a0 = 20(mod 1998)

ah+1a1= 100(mod1998),

5ah−1 =ah+1−4ah−20≡0(mod 1998). Do

ah−1 ≡0(mod 1998)vì (5,1998) = 1. Nếu h lẻ suy h−1 chẵn

ah= 5ah−1 ≡0(mod 1998),

trái với điều kiện ah ≡20(mod 1998).Vậy h phải chẵn,h>2 ah−1 ≡0(mod1998).

Điều kiện đủ Giả sửh>2,h chẵn ah−1 ≡0(mod 1998).Khi

(69)

Do

ah = 4ah−1+ 5ah−2+ 20≡20 = a0(mod1998), ah+1 = 4ah+ 5ah−1+ 20≡100 =a1(mod 1998). Bằng quy nạp, dễ dàng chứng minh

an+han(mod 1998).

Từ điều kiện (1.37)với h>2,h chẵn vàah−1 ≡0(mod 1998),suy ah−1 =

52 6(5

h

1)≡0(mod1998). Do

5h−1

6 ≡0(mod 1998).

Ta thấy điều kiện (9)tương đương với

5h≡1(mod 6·1998), hchẵn

và do1998 = 2·33·37suy ra6·1998 = 22·34·37,nên theo Định lý Euler,

5h ≡ 1(mod 22) thỏa mãn với h,

5h ≡ 1(mod 34) h 2·33= 54,

5h ≡ 1(mod 37) h 36, hchẵn Từ đó, điều kiện (1.37) tương đương với

h [54,36] = 108. Vậy,h= 108là số nhỏ phải tìm

Bài toán 1.53. Cho dãy số{an} được xác định theo công thức a0 = 0, a1 = 1, an+1 = 2anan−1+ (n>2). Chứng minh số A= 4anan+2+ là số phương. Giải. Ta có

an+1 = 2anan−1+ 1. (1.38) Trong (1.38)thay nbởin−1, ta

(70)

Trừ vế tương ứng (1.38)cho (1.39)ta thu an+1an= 2anan−1−2an−1+an−2 hay

an+1−3an+ 3an−1an−2 = 0. (1.40) Phương trình đặc trưng của(1.40)là

λ3−3λ2+ 3λ−1 = 0, có nghiệmλ= 1và nghiệm bội bậc ba

Vậy nên, nghiệm tổng quát của(1.40)là

an= (α+βn+γn2)1n. Cho n= 0, n= 1vàn= 2, ta

    

a0= = α,

a1= = α+β+γ, a2= = α+ 2β+ 4γ. Suy

α= 0, β=γ= 2.

Ta nhận an= n(n+ 1)

2 từ ta có

A= 4anan+2+ =n(n+ 1)(n+ 2)(n+ 3) + = (n2+ 3n+ 1)2. Điều chứng tỏ Amột số phương

Bài tốn 1.54. Tìm tất hàm số đồng biến f : N∗ → N∗ thoả mãn các điều kiện

(

f(2n) =f(n) +n, ∀n∈N∗,

f(n) chính phương kéo theo nchính phương. Giải. Dof hàm số đồng biến nên

f(n)< f(n+ 1)<· · ·< f(2n) =f(n) +n.

(71)

Bài toán 1.55. Cho a là số nguyên dương Dãy {an} được xác định theo

công thức (

a1 = 1, an+1 = 5an+

p

aa2

n−8, ∀n∈N∗. Với giá trị của athì dãy {an} là dãy số nguyên. Giải. Đặt√a−8 =t (t∈N) thìa2= +t,

a3 = 5(5 +t) +

p

(t2+ 8)(5 +t)2−8. Do vậy, để a3 ∈Zthì ta phải có

f(t) = (t2+ 8)(t+ 5)2−8 =q2 (q∈N). Ta có

(t2+ 5t+ 4)2 < f(t)<(t2+ 5t+ 14)2

f(t)là số chẵn nên suy

q =t2+ 5t+v vớiv∈ {6,8,10,12} Thử trực tiếp, ta thu v= 8vàt= 4nên a= 24 Ngược lại, với a= 24thì

an+1 = 5an+

p

24a2 n−8 kéo theo

an+2 = 10an+1an, a1 = 1, a2 = 9. Vậy, vớia= 24thì dãy{an} dãy số ngun

Bài tốn 1.56. Dãy số{an} được xác định theo công thức

(

a1 = 2,

an = 3an−1+ 2n3−9n2+ 9n−3, n= 2,3, Chứng minh với số nguyên tố p thì dãy tổng tương ứng

(72)

Giải. Theo giả thiết

an+n3 = 3[an−1+ (n−1)3] = 32[an−2+ (n−2)3] =· · ·= 3n−1(a1+ 1) = 3n. Vậy nên an= 3nn3 với mọin∈N∗

Vớip= 2thìa1 = 2 Vớip số nguyên tố lẻ

a1+a2+· · ·+ap−1 = (3 + 32+· · ·+ 3p−1)−[13+ 23+· · ·+ (p−1)3]. Dok3+ (pk)3 p

3 + 32+· · ·+ 3p−1 = 2(3

p

3) p nên suy

a1+a2+· · ·+ap−1 p.

Bài tốn 1.57. Dãy số{an} được xác định theo cơng thức an=

(2 +

3)n, n∈N∗. Chứng minh dãy {an} là dãy số nguyên lẻ. Giải. Ta có

(2 +

3)n=A+B

3, (2− √

3)n=AB

3, ∀n∈N∗, A, B∈N∗.Vậy nên

(2 +

3)n+ (2− √

3)n= 2A, ∀n∈N∗. Do

0<(2− √

3)n<1, ∀n∈N∗

nên ta có điều phải chứng minh

Bài tốn 1.58. Dãy số{an} được xác định theo công thức an= (

2 + 1)n, n∈N. Chứng minh mọi an đều viết dạng

an= √

(73)

Giải. Ta có

(

2 + 1)n=pn+qn

2 vớipn, qn∈N. Thật vậy, ta có p1 =q1 = 1và giả sử

(

2 + 1)n=pn+qn

2. Khi

pn+1+qn+1

2 = (

2 + 1)n+1 = (pn+qn

2)(

2 + 1) =pn+ 2qn+ (pn+qn)

2.

Đặt (

pn+1 =pn+ 2qn, qn+1 =pn+qn. Ta có

(

2 + 1)n+1 =pn+1+qn+1

2.

Bằng quy nạp, ta chứng minh |p2n−2qn2|= Thật vậy, ta có |p21−2q12|=|1−2.1|= 1.

Giả sửp2n−2qn2|=

|p2n+1−2qn+12 |=|(pn+ 2qn)2−2(pn+qn)2|

=| −p2n+ 2qn2|= 1. Từ ta có

an=pn+qn

2 =pp2

n+

p

2q2

n=

p

p2

n+

p

p2 n+ 1. Đặt p2n+ =m, ta có an=√m+√m−1, điều phải chứng minh Bài toán 1.59. Xét hàm số f : N∗ →N∗ thoả mãn điều kiện

f(n+ 1)> f(f(n)), ∀n∈N∗. Chứng minh rằngf(n) =nvới mọi n∈N∗.

Giải. Nhận xét rằng, tập giá trịRf tập khác rỗng củaN∗ nên có phần tử nhỏ Vì

(74)

nên f(1) phần tử nhỏ tập giá trịRf Do f(1) >1 nên f(n)> n với mọin∈N∗\ {1} Vì vậy, ta xét

f : N∗\ {1} →N∗\ {1}

Lập luận hoàn toàn tương tự trên, ta thu đượcf(1.16)là phần tử nhỏ hàm vừa nhận

Vậy,f(1)< f(1.16)và f(n)>2 với mọin >2 cho phép ta xét hàm f : N∗\ {1,2} →N∗\ {1,2}

Theo cách đó, ta thu f(n)>n với mọin∈N∗ Nếu xảy raf(m) > m thì ta cóf(m)>m+ f(f(n))>f(n+ 1) (dof tăng thực sự), điều

Vậy ln ln cóf(n) =nvới mọin∈N∗

Bài tập

Bài 1.1. Cho dãy số đơn điệu giảm

x1 >x2 >· · ·>xn

     

x1+x2+· · ·+xn = x21+x22+· · ·+x2n = 10

x1xn

r

10

n. Chứng minh rằngn là số chẵn Tính x1, x2, , xn. Bài 1.2. Cho dãy số đơn điệu giảm

x1 >x2>· · ·>x10 thoả mãn điều kiện

  

x5 = 15 x1+x2+· · ·+x10 = 100 x21+x22+· · ·+x210 = 1250.

Chứng minh rằngxj, j = 1,2, ,10,là số dương Tính x4 and x6 .

(75)

Bài 1.4. Xác định tất hàm số liên tục f : R→ R+ chuyển đổi cấp số cộng {xn} thành cấp số điều hoà{yn} vớiyn =f(xn),∀n∈N.

Bài 1.5. Xác định tất hàm số liên tục f : R+→R+ chuyển đổi cấp số điều hoà {xn} thành cấp số nhân {yn} với yn=f(xn),∀n∈N.

Bài 1.6. Xác định tất hàm số liên tục f : R+→R+ chuyển đổi cấp số điều hoà {xn} thành cấp số điều hoà{yn} với yn=f(xn),∀n∈N.

Bài 1.7. Xác định tất hàm số liên tục f : R+ →R chuyển đổi cấp số điều hoà {xn} thành cấp số cộng{yn} vớiyn =f(xn),∀n∈N.

Bài 1.8. Xác định tất hàm số liên tục f : R+ →R+ chuyển đổi dãy dương {xn} thoả mãn điều kiện

xn+1 =

s

x2n+2+x2n

2

thành cấp số nhân {yn} vớiyn=f(xn),∀n∈N. Bài 1.9. Xét dãy số{vn} được xác định bởi

v0 =a, vn+1 =

vn(v

n ln2−1) +

2vn ln2−1 ,∀n∈N.

a) Chứng minh với a <log2log2e thì dãy {vn} tăng bị chặn bởi 0.

b) Chứng minh vớia >log2log2e thì dãy{vn} giảm bị chặn bởi 1.

Bài 1.10. Cho a1 =a∈R vàan+1 =an.an−1, ∀n∈N∗ Hỏi với giá trị của a thì dãy {an} là dãy đơn điệu.

Bài 1.11. Cho

Sn= n+

2n+1

2 +

2

2 + 23

3 +· · ·+ 2n

n

.

Chứng minh dãy {Sn} là dãy đơn điệu giảm bị chặn 0. Bài 1.12. Các dãy số {xn} {yn} được xác định theo hệ thức sau

x1 =a >0, y1 =b >0, xn=

xn−1+yn−1

2 , yn=

(76)

Bài 1.13. Cho a, b, c∈Z, a chẵn, b lẻ Chứng minh với mọi n ∈N∗ luôn tồn tại x∈N∗ sao choax2+bx+c chia hết cho 2n.

Bài 1.14. Cho P(x) ∈ Z[x] A = {a1, a2, , an} ⊆ N∗ Biết với mọi k∈Zluôn tồn tại aiA sao cho aj|P(k) Chứng minh tồn tại aA sao cho P(k) chia hết cho a với mọi k∈Z.

Bài 1.15. Dãy {xn} được xác định sau : xn=

(2 +

3)n−(2− √

3)n

2√3 , ∀n∈N

.

Chứng minh dãy {xn} nguyên Xác định n đểxn chia hết cho 3. Bài 1.16. Chứng minh với dãy Fibonassi{xn},

x1 =x2= 1, xn+2 =xn+1+xn vớin∈N∗

luôn tồn số tự nhiên (a, b, c) (a > b, a > c) sao choxnnbcn luôn luôn chia hết cho a.

Bài 1.17. Xét hàm số f : Z+→Z+ thoả mãn điều kiện : (i)f(n+ 1)hoặc bằng f(n)−1 hoặc 4f(n)−1,

(ii) Với mỗi m∈Z+ cho trước, tồn tại n∈Z+ để f(n) =m. Tínhf(2007).

Bài 1.18. Xác định số nghiệm phương trình sau trong (0; 1) 8x(1−2x2)(8x4−8x2+ 1) =

Bài 1.19. Hỏi phương trình 4x3−3x= √1−x2 có nghiệm thực (kể cả bội)?

Bài 1.20. Chứng minh phương trình x4−6x2+ =

có nghiệm là tanπ 8,tan

3π

8 ,tan 5π

8 ,tan 7π

8

Bài 1.21. Cho 0< m <1.Giải phương trình

(77)

Bài 1.22. Giải biện luận bất phương trình

p

x2+a2 6x+ 2a

x2+a2

Bài 1.23. Cho x6=±√3,y6=±√3,z6=±√3 và thỏa mãn điều kiệnx+y+z=

xyz. Chứng minh rằng

3xx3

1−3x2 +

3yy3

1−3y2 +

3zz3

1−3z2 =

3xx3

1−3x2.

3yy3

1−3y2.

3zz3

1−3z2. Bài 1.24. Cho 0< a, b, c <1 thoả mãn điều kiện

a2+b2+c2+ 2abc= 1. Chứng minh rằng

abc+ =cp(1−a2)(1−b2) +ap(1−b2)(1−c2) +bp(1−c2)(1−a2). Bài 1.25. Cho |a|>1,|b|>1 Chứng minh rằng

p

a2−1 +pb2−16|ab| Bài 1.26. Chứng minh với mọi x, y ta có

−1

4

(x2−y2)(1−x2y2) (1 +x2)2(1 +y2)2

1 4.

Bài 1.27. Cho dãy số {xn} với < xn <1 xn+1(1−xn) >

1

4 Chứng minh

rằng dãy cho dãy đơn điệu tăng bị chặn. Bài 1.28. Cho dãy số {yn} xác định theo công thức

yn+1 = (1−x)yn+ Ax y

1−x x

n

, với A >0, 0< x <1, y0 >0, n∈N.

Hãy chứng minh dãy số dãy giảm.

Bài 1.29. Cho f(x) là đa thức với hệ số nguyên thỏa mãn điều kiện f(0) =

(78)

Bài 1.30. Giả sửa0, a1, , an là chữ số số có n (n>1) chữ số A=a0+ 10a1+ 102a2+· · ·+ (10)nan.

Chứng minh không điểm đa thức

P(z) =a0+a1z+a2z2+· · ·+anzn

hoặc nằm nửa mặt phẳng bên trái nằm hình trịn

O(0, r) ={z∈C: |z|< + √

37

(79)

Các toán nội suy cổ điển

Trong chương này, ta xét số toán liên quan đến khai triển Taylor khai triển Lagrange Các dạng khai triển cịn có tên cơng thức Taylor (nội suy Taylor) cơng thức nội suy Lagrange Ngồi ra, cịn số dạng khai triển nội suy cổ điển tiếng khác khai triển Taylor - Gontcharov nội suy Hermite đề cập sách giáo khoa sách tham khảo hành Vì thế, chương này, phần xét số toán liên quan đến khai triển Taylor khai triển Lagrange, trình bày sơ lược ý tưởng phát biểu toán dạng tổng quát, nêu cách chứng minh tồn nghiệm công thức tường minh nghiệm Bạn đọc chưa quen với khai triển Taylor - Gontcharov toán nội suy Hermite chắn gặp vài khó khăn nội dung kỹ thuật chứng minh dễ dàng khắc phục đọc kỹ phần ví dụ minh hoạ trước, sau quay lại đọc phần trình bày lý thuyết chắn lĩnh hội tư tưởng hệ thống phương pháp giải Cũng cần lưu ý cịn số tốn nội suy cổ điển tổng quát khác liên quan mật thiết đến toán ban đầu toán biên hỗn tạp phương trình vi phân phương trình đạo hàm riêng khơng đề cập Vì thế, tốn nội suy cổ điển xét chương hiểu bốn toán trình bày mức độ sơ lược thơng qua mơ tả ví dụ ý tưởng chủ yếu Đó tốn khai triển Taylor (Nội suy Taylor), đồng thức Lagrange (Nội suy Lagrange), Khai triển Taylor - Gontcharov (Nội suy Newton) đồng thức Hermite (Nội suy Hermite)

Phần cuối chương có trình bày sơ lược số toán nội suy hỗn hợp tổ hợp dạng nội suy biết cho tồn tính nghiệm tốn nội suy không bị phá vỡ

(80)

2.1 Khai triển nội suy Taylor

Ta thường thấy sách giáo khoa hành, dạng tắc đa thức đại số P(x) bậc n(n >0, thường ký hiệu degP(x) =n) có dạng

P(x) =p0xn+p1xn−1+· · ·+pn, p06= 0.

Đa thức dạng tắc đa thức viết theo thứ tự giảm dần luỹ thừa Tuy nhiên, khảo sát đa thức, người ta thường quan tâm đến lớp đa thức bậc không số nguyên dương ncho trước Vì thế, sau, ta thường sử dụng cách viết đa thứcP(x) dạng tăng dần bậc luỹ thừa P(x) =b0+b1x+b2x2+· · ·+bnxn. (2.1) Nhận xét rằng, đa thức (2.1) có tính chất

P(k)(0) =k!bk, k= 0,1, , n

P(k)(0) = 0, k=n+ 1, n+ 2,

Vì đa thức (2.1) thường viết dạng công thức (đồng thức) Taylor

P(x) =a0+ a1

1! +

a2

2!x

2

+· · ·+an

n!x

n

. (2.2)

Với cách viết (2.2) ta thu công thức tính hệ sốak (k= 0,1, , n) đa thứcP(x), giá trị đạo hàm cấpk đa thức x= 0:

ak =P(k)(0), k= 0,1, , n. Từ ta thu đồng thức Taylor tạix= 0:

P(x) =P(0) + P

0(0)

1! x+

P(2)(0)

2! x

2

+· · ·+P

(n)(0) n! x

n

. (2.3)

Ví dụ 2.1. Viết biểu thức

Q(x) = (x2−2x−2)5+ (2x3+ 3x2−x−1)2

dưới dạng (chính tắc) cơng thức Taylor tại x= 0: Q(x) =a0+a1

1!x+

a2

2!x

2

+· · ·+ a10 10!x

10

. (2.4)

(81)

Theo cơng thức(2.3)thì ta có hệ thứca8 =Q(8)(0).

Dạng (2.2) cho ta mối liên hệ trực tiếp hệ số đa thức tắc với giá trị đạo hàm đa thức x = Trong trường hợp tổng quát, công thức Taylor tạix=x0, có dạng

P(x) =P(x0)+P

0(x 0)

1! (x−x0)+

P(2)(x0)

2! (x−x0)

2

+· · ·+P

(n)(x 0)

n! (x−x0)

n . (2.5) Ví dụ 2.2. Viết biểu thức

Q(x) = (x−1)(x−2)· · ·(x−9)

dưới dạng (chính tắc) công thức Taylor điểm x= 10 :

Q(x) =a0+a1

1!(x−10) +

a2

2!(x−10)

2

+· · ·+ a9

9!(x−10)

9

. (2.6) Tính giá trị của a7?

Cơng thức(2.5)cho ta hệ thứca7 =Q(7)(10).

Trong trường hợp đa thức bậc tuỳ ý, ta có kết hồn tồn tương tự Ví dụ 2.3. Chứng minh đa thứcP(x) thoả mãn điều kiệndegP(x)6n P(k)(α) = qk với mọi k ∈ {0, , n}, đó α, qk là số cho trước; P(0)(x)≡P(x),thì P(x) có dạng

P(x) =

n

X

k=0 qk

k!(xα)

k.

Đẳng thức chứng minh trực tiếp cách lấy đạo hàm liên tiếp hai vế sử dụng giả thiết giá trị ban đầuP(k)(α) =qk với mọik∈ {0, , n}. Việc chứng minh tính suy từ tính chất đa thức (khác 0) bậc khơng vượt qnlà có khơng q nnghiệm (kể bội)

Bây giờ, ta chuyển sang xét toán nội suy Taylor

Bài toán 2.1. Cho x0, ak ∈ R, với k = 0,1, , N −1. Hãy xác định đa thức T(x) có bậc khơng q N−1 (degT(x)6N−1)và thỏa mãn điều kiện

T(k)(x01) =ak1, ∀k= 0,1, , N−1. (2.7) Giải. Trước hết, dễ thấy đa thức

T(x) =

NX−1 k=0

(82)

có bậc degT(x)6N −1 Tiếp theo, ta cần xác định hệ sốαk1 ∈R cho T(x) thỏa mãn điều kiện

T(k)(x01) =ak1, ∀k= 0,1, , N−1.

Lần lượt lấy đạo hàmT(x) đến cấp thứ k, k = 0,1, , N −1,tại x = x01 sử dụng giả thiết

T(k)(x01) =ak1, ∀k= 0,1, , N−1, ta suy

αk1= ak1

k! , ∀k= 0,1, , N−1.

Thay giá trị củaαk1 vào biểu thức củaT(x), ta thu T(x) =

N−1X k=0

ak1

k! (xx01)

k (2.8)

Với mỗik= 0,1, , N−1,ta có T(k)(x) =ak1+

N−1X j=k+1

aj1

(jk)!(xx01)

j−k .

Do đa thứcT(x) thỏa mãn điều kiện

T(k)(x01) =ak1, ∀k= 0,1, , N−1.

Cuối cùng, ta chứng minh đa thức T(x) nhận từ (2.8) đa thức thỏa mãn điều kiện toán nội suy Taylor (2.8), ta gọi đa thức đa thứcnội suy Taylor

Thật vậy, có đa thức T∗(x), có bậc degT∗(x) N −1 thỏa mãn điều kiện tốn (2.8) đó, đa thức

P(x) =T(x)−T∗(x)

cũng có bậcdegP(x)6N −1 đồng thời thỏa mãn điều kiện P(k)(x01) = 0, ∀k= 0,1, , N−1.

Tức là, đa thứcP(x) đa thức có bậc khơng qN−1(degP(x)6N−1)

(83)

Nhận xét 2.1. Chú ý đa thức nội suy Taylor T(x)được xác định từ (2.8)

chính khai triển Taylor đến cấp thứ N−1 của hàm sốT(x) tại điểmx=x01. Ta xét công thức khai triển Taylor đa thức Tiếp theo, mục ta xác lập công thức Taylor với phần dư khác Ta nhắc lại, hàmf khả vi điểmx=a, theo định nghĩa, ta có

f(a+h)−f(a) =f0(a)h+o(h). Nếu đặta+h=x thìh =ax

f(x)−f(a) =f0(a)(xa) +o(xa). Nói cách khác, tồn hàm tuyến tính

P1(x) =f(a) +f0(a)(xa) cho

f(x) =P1(x) +o(xa),

P1(a) =f(a), P01(a) =f0(a). Ta phát biểu số toán sau

Bài toán 2.2. Giả sử hàm f xác định tập hợp Ω⊂ R, đólà hợp của khoảng mở trục thực Giả sử f khả vi cấp n tại điểm a∈ Ω Hãy xác định đa thức Pn(x) có bậc 6n, sao cho

Pn(γ)(a) =f(γ)(a), γ= 0, , n. Giả sửP(x) đa thức (bậcn) tuỳ ý Ta viết

P(x) =

n

X

γ=0

γ!(xa)

γ . Khi

P(µ)(a) =

µ!µ! =aµ.

Nếu ta đặt

=f(µ)(a), µ= 0, , n,

f(µ)(a) =P(µ)(a). Như tốn giải xong

(84)

Định nghĩa 2.1. Đa thức

Tn(f;x) =

n

X

γ=0

(x0)

γ! (xa)

γ

được gọi đa thức Taylor bậc nvới tâm a của hàm f, khả vi cấp n tại điểm a. Ta đặt

f(x) =Tn(f;x) +Rn(f;x) =

=f(a) +f0(a)(xa) +· · ·+

n!f

(n)

(a)(xa)n+Rn(f;x). (2.9) Công thức (2.9) gọi công thức Taylor (dạng đầy đủ) hàm f(x) Nếu a= 0thì (2.9) gọi công thức Maclaurin

Biểu thứcRn(f;x)được gọi phần dư công thức Taylor Với điều kiện khác đặt hàm f, phần dư biểu diễn công thức khác Lời giải tốn ước lượng hiệuf(x)−Pn(x) ước lượng biểu thức phần dư

Bổ đề 2.1. Nếu hàm ϕcó đạo hàm đến cấp n tại điểma ϕ(a) =ϕ0(a) =· · ·=ϕ(n)(a) = 0, thì ϕ(x) =o((xa)n) khi xa, tức là ϕ(x)

(xa)n →0(xa). Chứng minh.

Ta chứng minh phương pháp quy nạp Vớin= 1,ta có ϕ(a) =ϕ0(a) ϕ(x)

xa =

ϕ(x)−ϕ(a)

xaϕ

(a) = 0(xa),

tức ϕ(x) =o((xa)).Giả sử bổ đề vớinnào đó, tức với điều kiện ϕ(a) =ϕ0(a) =· · ·=ϕ(n)(a) = 0,

thì

ϕ(x) =o((xa)n). Ta cần chứng minh vớiϕ(n+1)(a) = 0,thì

ϕ(x) =o((xa)n+1), xa. Ta xét hàmψ(x) =ϕ0(x) Ta có

(85)

và đóψ(x) =o((xa)n), tức ψ(x) (xa)n =

ϕ0(x)

(xa)n →0 (xa) (2.10) Theo định lý Lagrange, ta có

ϕ(x) (xa)n+1 =

ϕ(x)−ϕ(a) (xa)n+1 =

ϕ0(ξ)(xa)

(xa)n+1 =

ϕ0(ξ)

(ξa)n

ξa xa

n , ξ nằm xen giữaax Từ đó, ta thu được

ϕ0(ξ)

(ξa)n → (xa), 0<

ξa xa<1.

Như vậy, ϕ(x) = o((xa)n+1) xa bổ đề chứng minh

Định lý 2.1 (Taylor). Giả sử f :U(a, δ) → R là hàm khả vi liên tục đến cấp n−1 trongδ-lân cận U(a, δ) của điểm avà có đạo hàm hữu hạn cấp ntại điểm a Khi đó, hàm f có thể biểu diễn dạng

f(x) =

n

X

k=0

f(k)(a)

k! (xa)

k

+o((xa)n) (2.11) khi xa, đó 0! = 1, f(0)(a) =f(a).

Công thức (2.11) gọi công thức Taylor dạng địa phương với phần dư Peano

Chứng minh Đặt

ϕ(x) =f(x)− n

X

k=0

f(k)(a)

k! (xa)

k

, ψ(x) = (xa)n. (2.12) Từ (2.12) ta dễ dàng thấy ϕ(a) = ϕ0(a) = · · · = ϕ(n)(a) = Do theo bổ đề 2.1, ta thu đượcϕ(x) =o(ψ(x)),xavà hệ thức (2.11) chứng minh

Công thức (2.11) cho ta dáng điệu củaf(x)−Tn(f;x) với giá trị xđủ gầna Để sử dụng đa thứcTn(f;x)làm công cụ xáp xỉ hàmf(x),ta cần phải đưa dạng khác phần dưRn(f;x)

(86)

Định lý 2.2 (Taylor). Giả sử f : (a, b) → R khả vi liên tục cấp n trên khoảng

(a, b)và có đạo hàm cấp n+ tại điểm khoảng(a, b)có thể trừ điểm x0 ∈ (a, b) Khi đó, điểm x0 và điểm x ∈ (a, b) bất kỳ, tồn điểm ξ, sao cho

f(x) =

n

X

k=0 f(k)(x

0)

k! (xx0)

k

+Rn+1(f;x), (2.13) trong đó

Rn+1(f;x) =

n!p

xx0 xξ

p

(xξ)n+1f(n+1)(ξ), p∈R, p >0. (2.14) Công thức (2.13) gọi công thức Taylor hàm f với phần dư Rn+1 dạng Schlomilch - Roche

Chứng minh.

Để cho xác định ta coix > x0 Xét hàm số h(t) =f(x)−

n

X

k=0 f(k)(t)

k! (xt)

k −(xt)p

n!p λ, x0 6t6x, (2.15) p∈R,p >0,λlà tham số

Hàm h(t) liên tục đoạn [x0, x],h(x) = đạo hàm h0(t) tồn ∀t∈

(x0, x) Ta chọn số λsao cho h(x0) =f(x)−

n

X

k=0

f(k)(x0)

k! (xx0)

k−(xx0)p

n!p λ= (2.16) Với cách chọn đó, hàmh(t)thoả mãn điều kiện Định lý Rolle đoạn

[x0, x] Do đó∃ξ ∈(x0, x),sao cho h0(ξ) =−f

(n+1)(ξ)

n! (xξ)

n

+(xξ)

p−1

n! λ= 0. (2.17)

Thật vậy, từ hệ thức (2.15), ta có h0(t) =−f0(t) + f

0

(t) 1! −

f00(t)

1! (xt) +

f00(t)

2! 2(xt)− · · · (2.18) +f

(n)(t)

n! n(xt)

n−1f(n+1)(t) n! (xt)

n

+ (xt)

p−1

n! λ. (2.19)

Dễ dàng thấy số hạng vế phải (2.18) trừ hai số hạng cuối khử hết Từ cách thay t=ξ ta thu (2.17) Từ (2.17), ta có

(87)

Thayλtừ (2.20) vào (2.16), ta thu (2.13) (2.14), điều phải chứng minh

Bằng cách chọn giá trị p >0 hoàn toàn xác định, ta thu trường hợp riêng phần dưRn+1(f;x) Ta xét trường hợp quan trọng p=n+ vàp=

Khi p=n+ 1thì từ (2.14), ta thu phần dư công thức Taylor dạng Lagrange

Rn+1(f;x) = f

(n+1)(ξ)

(n+ 1)! (xx0)

n+1

, ξ=x0+θ(xx0), 0< θ <1. (2.21) Khi p = 1, từ (2.14), ta thu phần dư công thức Taylor dạng Cauchy:

Rn+1(f;x) = f

(n+1)(x0+θ(xx0))

n! (xx0)

n+1

(1−θ)n, 0< θ <1, (2.22) ξ=x0+θ(xx0)

Nhận xét 2.2. Công thức Maclaurin với phần dư (2.21) (2.22) có dạng tương ứng

Rn+1(f;x) = f

(n+1)(θx) n! x

n+1

, 0< θ < (dạng Lagrange),

Rn+1(f;x) = f

(n+1)(θx)

(n= 1)! (1−θ)

n

xn+1, 0< θ <1 (dạng Cauchy). Sau ta xét khai triển Maclaurin số hàm sơ cấp đơn giản Ví dụ 2.4. Hàm f(x) =ex, x∈R f(n)(x) =ex ∀x∈R Do đó

ex = +x+x

2

2! +· · ·+

xn

n! +Rn(x),

Rn(x) = (n+ 1)!e

θx

xn+1, 0< θ <1. Ta có

|Rn(x)|6

|x|n+1

(n+ 1)!e

x

(88)

Ví dụ 2.5. Hàm f(x) = sinx f(n)(x) = sin

x+

2

, n= 0,1,· · ·,

f(0) = 0, f0(0) = 1, f00(0) = 0, f(3)(0) = 1,· · ·, f(2n)(0) = sin2

2

= 0,

f(2n+1(0) = sin

h

(2n+ 1)π

i

= (−1)n. Do đó

sinx=xx

3! +

x5

5! − · · ·+

(−1)n−1x2n−1

(2n−1)! +R2n(x),

R2n+1(x) =

sin

h

(θx+ (2n+ 1)π2

i

x2n+1

(2n+ 1)! , 0< θ <1. (2.23)

Từ (2.23) suy rằng

|R2n+1(x)|6 |x|

2n+1

(2n+ 1)! →0(n→ ∞), ∀x∈R.

Ví dụ 2.6. Hàm f(x) = cosx f(n)(x) = cos

x+

, f(n)(0) = cos ,

f(n)(θx) = cos

θx+

2 ), n= 1,2,· · ·.

Công thức Maclaurin theo lũy thừa củax với phần dư dạng Lagrange có dạng

cosx= 1−x

2! +

x4

4! − · · ·+ (−1)

n−1 x2(n−1)

[2(n−1)]!+R2n(x),

R2n(x) = x

2n

(2n)!cos

θx+ 2

2

→0 (n→ ∞), ∀x∈R.

Ví dụ 2.7. Xét hàm sốf(x) = ln(1 +x) xác định khả vi vô hạn lần ứng với mọi x >−1

f(x) =xx

2 +· · ·+ (−1)

nx n−1

(89)

Trong truờng hợp ta viết hai công thức phần dư Rn(x) = (−1)

n+1xn

n(1 +θx)n, 0< θ <1, −dạng Lagrange; Rn(x) = (−1)n+1

xn

(1 +θx)

1−θ

1 +θx

n−1

, 0< θ <1, −dạng Cauchy Giả sử06x61 Khi áp dụng cơng thức phần dư dạng Lagrange ta có

|Rn(x)|6

n|x|

n

0 (n→ ∞).

Trong trường hợp−1< x <0,công thức phần dư dạng Lagrange không cho ta kết luận dáng điệu Rn (vì ta biết 0< θ <1 vàθ=θ(x, n)) Ta áp dụng công thức phần dư dạng Cauchy Ta có

|Rn|< |x|n

1− |x|→0 (n→ ∞), 0<|x|<1, 1−θ1 +θx <

1−θ 1−θ

=

Trong trường hợp nàyx >1, Rn(x)6→0 n→ ∞ Để thấy rõ điều ta đặt

Sn(x) =xx

2 +· · · −(−1)

nxn−1 n−1.

Khi đó,

Sn(x) +Rn(x) =Sn+1(x) +Rn+1(x)

Rn(x)−Rn+1(x) = (−1)n+1x

n n .

Vớix >1và n→ ∞ vế phải đẳng thức khơng dần đến Do Rn(x) khơng dần đến n → ∞ khơng thoả mãn tiêu chuẩn tồn giới hạn Cauchy

Như phần dưRn(x) công thức Taylor hàm ln(1 +x)chỉ dẫn đến với−1< x <1

Xét hàmf(x) = (1 +x)m Đối với hàm ta cần xétm∈R\N Ta có f(n)(x) =m(m−1)· · ·(mn+ 1)(1 +x)m−n,

(90)

Công thức Taylor theo lũy thừa củax có dạng

(1 +x)m= +mx+m(m−1)

2! x

2

+· · ·+m(m−1)· · ·(mn+ 1)

(n−1)! x

n−1

+Rn(x). Khi đó, phần dư dạng Lagrange có dạng

Rn=

m(m−1)· · ·(mn+ 1)

n! x

n(1 +θx)m−n (2.24) dạng Cauchy

Rn= m(m−1)· · ·(mn+ 1)

(n−1)! x

n

(1 +θx)m−1

1−θ

1 +θx

n−1

. (2.25) Với06x <1, theo (2.24) vớin > m,ta có

|Rn|6

|m(m−1)· · ·(mn+ 1)|

n! |x|

n

0 (n→ ∞). Thật vậy, ta đặtUn= m(m−1)· · ·(mn+ 1)

n! x

n

Khi đó, |Un+1|

|Un| =

|m−n| n+ |x|

Suy

|Un+1|= |m−n|

n+ |x||Un|

Với n đủ lớn vế phải < |m−n|

n+ → n→ ∞ x < Do

dãy(Un) có giới hạnu Chuyển qua giới hạn đẳng thức khin→ ∞, ta thu được|u|=|x||u|hayu= Do Rn→0 (n→ ∞) với06x <1

Với−1< x <0, từ (2.25), ta có |Rn|6C

|m(m−1)· · ·(mn+ 1)|

(n−1)! |x|

n

, (2.26)

trong C số phụ thuộcx khơng phụ thuộcn. Thật vậy, để ước lượng|Rn|, ta xét

1−θ

1 +θx

n−1

6

1−θ

1−θ

n−1

=

và vớim−1>0

(91)

Khi m−1<0thì

(1 +θx)m−1 <

(1− |x|)1−m.

Từ suy (2.26) Bằng cách chứng minh tương tự phần trước ta dễ dàng thấy Rn→0 khin→ ∞

Nhận xét 2.3. Vấn đề mấu chốt việc tìm cơng thức Taylor hàm f cho trước tính hệ số an của Các hệ số tính theo cơng thức an =

f(n)(a)

n! . Tuy nhiên công thức tổng qt thường tiện lợi việc tính tốn đạo hàm cấp cao cồng kềnh Thông thường hệ số đa thức Taylor Tn(f;x) được tính cách sử dụng khai triển nêu trên.

Bây ta xét vài ví dụ Để cho tiện lợi, ta tìm cơng thức Taylor với phần dư dạng Peano

Ví dụ 2.8. Khai triển hàm f(x) =

2x+ theo cơng thức Taylor đến số hạng

o(xn). Ta có

1 2x+ =

1 3(1 + 23x).

Sử dụng khai triển

1 +x =

n

X

k=0

(−1)kxk +o(xn), ta có

1 2x+ =

n

X

k=0

(−1)k

k

3k+1x k

+o(xn). Ví dụ 2.9. Hàm f(x) = ln(5−4x)

ln(5−4x) = ln5 + ln

1−4

5x

. Theo công thức khai triển trên, ta thu được

ln(5−4x) = ln5− n

X

k=0

1

k

4

k

xk+o(xn). Ví dụ 2.10. Xét hàm số

f(x) = ln3 +x

(92)

Trước hết ta nhận xét nếu

f(x) =

n

X

k=0

ak(xx0)k+o((xx0)n),

g(x) =

n

X

k=0

bk(xx0)k +o((xx0)n), thì

f(x) +g(x) =

n

X

k=0

(ak+bk)(xx0)k +o((xx0)n). Từ đẳng thức

f(x) = ln3

2 + ln(1 +

x

3)− ln

1−x

2

, suy ra

f(x) = ln3

2+ n X k=0 k 2k +

(−1)k−1 3k

xk+o(xn). Ví dụ 2.11. Xét hàm số

f(x) = x

2+ 5 x2+x−12.

Để khai triển theo công thức Taylor hàm hữu tỷ thông thường, ta biểu diễn hàm hữu tỷ dạng tổng đa thức phân thức tối giản Ta có

f(x) = 1− x+ +

2

x−3

= 1−

4

1 + x

3

1−x

3

= 1−

4

n

X

k=0

(−1)kx

k

4k

2 n X k=0 xk

3k +o(x n)

=−

12+

n

X

k=0

3(−1)k+1 4k+1

2 3k+1

xk+o(xn).

(93)

Ví dụ 2.12. Tính giới hạn

lim

x→0

sin(sinx)−x

1−x2

x5 .

Giải. Vì mẫu số đa thứcx5 nên ta cần khai triển tử số thành đa thức Taylor với độ xác đến 0(x5) khix→0

Vìsinxx khix→0 nên o(x5) =o(sin5x) x→0 Theo công thức Taylor, ta có

sinx=xx

6 +

x5

120+o(x

5

)

sin sinx= sinx−sin

x

6 +

sin5x

120 +o(sin

5

x), x→0. Khi

sin3x= [(xx

6 +

x5

120 +o(x

5

)]3= = [x+α(x)]3 =x3+ 3x3α(x) + 32(x) +α3(x),

α(x) =−x

6 +

x5

120+o(x

5

)≈ −x

6 .

Suy

2(x)≈ x

36 =o(x

5), α3(x)≈ − x

216 =o(x

5) khi x→0. Do

sin3x=x3−1

2x

5

+ 0(x5) x→0. Tiếp theo, ta chứng minh

sin5x=x5+ 0(x5), x→0. Thật vậy,

α(x)≈ −x

6 x→0

nên

sin5x=x5+o(x5), x→0. Như vậy, x→0 ta có

sin(sinx) =xx

3 +

x5

10+o(x

5

(94)

Tương tự

xp3 1−x2 =x(1−1

3x

2−1

9x

4

+o(x4)) = =x−1

3x

3−1

9x

5

+o(x5), x→0. Do

sin(sinx)−xp3 1−x2 = 19

90x

5

+o(x5). Vậy nên

lim

x→0f(x) = limx→0(

19 90+

0(x5) x5 ) =

19 90.

Ví dụ 2.13. Tính giới hạn

lim

x→0

1 + tanxex+x2

arcsinx−sinx .

Giải. Tử số mẫu số phân thức vơ bé khix→0 Vì

sinx=xx

6 + 0(x

3)

arcsinx=x+x

3

6 + 0(x

3

) x→0

nên mẫu số có dạng

arcsinx−sinx= x

3

3 +o(x

3

), x→0. Từ đó, ta cần khai triển tử số với độ xác đến0(x3) Ta có

ex = +x+x

2

2! +

x3

3! +o(x

3), x→0,

1 +t= + 2t

1 8t

2

+ 16t

3

+o(t3), t→0.

tanx=x+x

3

3 +o(x

3), x→0. Vậy nên

1 + tanx= +

2(2 tanx)−

8(2 tanx)

2

+

16(2 tanx)

3

(95)

= +x+x 3 − x2 + x3

2 +o(x

3

)

= +xx

2 + 6x

3+o(x3), x→0. Khi tử số có khai triển

1 + tanxex+x2 = 3x

3

+o(x3), x→0. Từ suy

lim

x→0

1 + tanxex+x2

arcsinx−sinx = limx→0 3x

3+o(x3)

3x3+o(x3)

= 2. Ví dụ 2.14. Tính giới hạn

lim

x→0

tan(tanx)−sin(sinx) tanx−sinx . Giải. Theo công thức Taylor vớin= 3,ta có

tanx=x+ x

3

3 + 0(x

3

), x→0

sinx=xx

6 + 0(x

3

), x→0.

Khi theo tính chất khai triển Taylor hàm hợp ta có

tan(tanx) = tan(x+x

3

3 + 0(x

3

))

=x+x

3

3 + 0(x

3

) +1

x+x

3

3 + 0(x

3

)

3

=x+2 3x

3

+ 0(x3), x→0,

sin(sinx) = sin

xx

6 + 0(x

3

)

=xx

6 + 0(x

3

)−1

6

xx

6 + 0(x

3

)

3

=x−1

3x

3

+ 0(x3), x→0. Vậy nên

lim

x→0

tan(tanx)−sin(sinx) tanx−sinx = limx→0

x3+ 0(x3)

x3

2 + 0(x3)

(96)

Ví dụ 2.15. Tính giới hạn

lim

x→0

1

x2 −

1 sin2x

.

Giải. Giới hạn cần tìm có dạng "∞ − ∞" Ta biến đổi dạng "00" sau

lim

x→0

1

x2 −

1 sin2x

= lim

x→0

sin2xx2 x2.sin2x

= lim

x→0

[xx63 + 0(x3)]2−x2 x2[x+ 0(x)]2

= lim

x→0x4

3 + 0(x 4) x2[x2+ 0(x2)] =−

1 3. Vậy nên lim x→0 1

x2 −

1 sin2x

=−1

3.

Ví dụ 2.16. Tính giới hạn

lim

x→0+

1

xx

aarctan r x a− √ barctan r x b

(a >0, b >0).

Giải. Giới hạn cho có dạng "0.∞" Ta đưa dạng "00" phép đổi biến Đặt √x=t thìt2 =x khix→0+ thìt→0+ Ta có

lim

x→0+

1

xx

aarctan r x a − √ barctan r x b = lim t→0aarctan q t a− √ barctan q t b

t3 .

Vì mẫu số đa thức bậc nên ta cần khai triển Taylor tử số xác đến

0(t3) Khi t→0+thì √ aarctan r t a = √ a h ta+t

3

3√a3+ 0( t

3

3√a3)

i

=t+ t

3

3a+ 0(t

), t→0+,barctan r t b = √ bh√t

b+t

33√b3+ 0( t

3

b3

)i=t+ t

3

3b+ 0(t

(97)

Khi

lim

x→0+

1

xx

aarctan r x a − √ barctan r x b = lim t→0

t+3at3 + 0(t3)

t+3bt3 + 0(t3)

t3 = lim x→0 a−b 3a t

3 + 0(t3) t3 =

ab

3ab . Vậy

lim

x→0+

1

xx

aarctan r x a − √ barctan r x b

= ab 3ab . Ví dụ 2.17. Tính giới hạn

lim

x→0(cos(x.e

x)− ln(1−x)−x)cotx3. Giải. Giới hạn cần tìm có dạng1∞ Ta có

lim

x→0(cos(x.e x

)− ln(1−x)−x)cotx3 =exlim→0cotx

3 lnf(x)

với

f(x) = cos(x.ex)− ln(1−x)−x. Ta cần tính

lim

x→0cotx

ln[cos(x.ex)− ln(1−x)−x]. Để ý

cotx3 = tanx3 =

1

x3+ 0(x3), x→0.

Do ta cần phải khai triển hàm [f(x)−1]theo công thức Taylor tương đương với 0(x3) bằng cách sử dụng khai triển sau

xex=x+x2+ 0(x2), x→0,

cost= 1−t

2! + 0(t

3

), t→0. Suy

cosxex = 1−x22−x3+ 0(x3), x→0, −ln(1−x) =x+ x

2

2 +

x3

3 + 0(x

3

(98)

Ta thu dược

f(x)−1 =−2

3x

3+ 0(x3), x→0

lim

x→0

−23x3+ 0(x3)

x3+ 0(x3) =−

2 3.

Vậy

lim

x→0(cos(x.e x

)− ln(1−x)−x)cotx3 =e−23.

2.2 Bài toán nội suy Lagrange

Chú ý rằng, số trường hợp, để tính tổng hữu hạn phân thức, người ta thường sử dụng số tính chất đa thức, đặc biệt công thức nội suy Lagrange

Dưới số đồng thức áp dụng chúng

Định lý 2.3 (Đồng thức Lagrange). Nếu x1, x2, , xm làm giá trị tuỳ ý, đôi khác và f(x) là đa thức bậc nhỏ thua m thì ta có đồng thức sau

f(x) =f(x1) (xx2)(xx3) .(xxm) (x1x2)(x1x3) .(x1xm)+ +f(x2)

(xx1)(xx3) .(xxm)

(x2x1)(x2x3) .(x2xm) +· · ·+f(xm)

(xx1)(xx2) .(xxm−1) (xmx1)(xmx2) .(xmxm−1)

. (2.27)

Chứng minh Ta cần chứng minh công thức

f(x)−f(x1) (xx2)(xx3) .(xxm) (x1−x2)(x1−x3) .(x1−xm)

−f(x2)

(xx1)(xx3) .(xxm)

(x2−x1)(x2−x3) .(x2−xm) − · · · −f(xm) (xx1)(xx2) .(xxm−1)

(xmx1)(xmx2) .(xmxm−1) ≡0.

Nhận xét vế trái công thức đa thức bậc không m−1

(99)

Hệ 2.1. Từ Định lý 2.3, ta có đồng thức sau đây.

(x−√3)(x−√5)(x−√7) ( √ 2− √ 3)( √ 2− √ 5)( √ 2− √ 7)+

(x−√2)(x−√5)(x−√7) ( √ 3− √ 2)( √ 3− √ 5)( √ 3− √ 7)

+ (x

2)(x−√3)(x−√7) ( √ 5− √ 2)( √ 5− √ 3)( √ 5− √ 7) +

(x−√2)(x−√3)(x−√5) ( √ 7− √ 2)( √ 7− √ 3)( √ 7− √

5) ≡1,

a2(xb)(xc)

(ab)(ac) +b

2(xc)(xa)

(bc)(ba) +c

2(xa)(xb)

(ca)(cb) ≡x

2

(a < b < c). Định lý 2.4. Giả sử f(x) là đa thức bậc nhỏ thua bằngm−2 x1, x2, , xm m giá trị đơi khác cho trước tuỳ ý Khi ta có đồng nhất thức

f(x1)

(x1−x2)(x1−x3) .(x1−xm)

+ f(x2)

(x2−x1)(x2−x3) .(x2−xm)

+ · · ·+ f(xm)

(xmx1)(xmx2) .(xmxm−1) ≡0. Chứng minh.

Nhận xét vế trái đẳng thức cho hệ số hạng tử ứng với bậc m−1 cách viết tắc đa thức f(x) Đồng hệ số đồng bậc ta có điều phải chứng minh

Tiếp theo, ta xét số ứng dụng trực tiếp đồng thức Lagrange Ví dụ 2.18. Tính tổng

S= cos

o

(cos 1o−cos 2o)(cos 1o−cos 3o)+

cos 2o

(cos 2o−cos 1o)(cos 2o−cos 3o)

+ cos

o

(cos 3o−cos 1o)(cos 3o−cos 2o) Giải.Sử dụng Định lý 2.3, với

f(x) =x, x1 = cos 1o, x2= cos 2o, x3 = cos 3o, ta thu đượcS =

Ví dụ 2.19. Ta có đồng thức b+c+d

(ba)(ca)(da)(xa) +

c+d+a

(100)

d+a+b

(dc)(ac)(bc)(xc) +

a+b+c

(ad)(bd)(cd)(xd)

xabcd

(xa)(xb)(xc)(xd).

Thật vật, ta cần chứng minh

(a+b+c+d)−a

(ab)(ac)(ad)(ax) +

(a+b+c+d)−b

(ba)(bc)(bd)(bx)+ + (a+b+c+d)−c

(ca)(cb)(cd)(cx) +

(a+b+c+d)−d

(da)(db)(dc)(dx)+ + (a+b+c+d)−x

(xa)(xb)(xc)(xd) = 0.

Ta có, với đa thức bậc

f(y) =a+b+c+dy, y1 =a, y2=b, y3 =c, y4 =d, y5 =x, theo Định lý 2.3 ta thu điều phải chứng minh

Định lý 2.5. Cho x1, x2, , xm m giá trị tuỳ ý đôi khác Đặt

Sn= x

n

(x1−x2)(x1−x3) .(x1−xm)

+ x

n

(x2−x1)(x2−x3) .(x2−xm)

+· · ·+ x

n m

(xmx1)(xmx2) .(xmxm−1) . Khi đó

a) Sn= 0nếu 0≤n < m−1, b) Sm−1 = 1,

c) Sm+k bằng tổng tích, tích cók+ thừa số (giống khác nhau) lấy số x1, x2, , xm.

Chứng minh.

a) Theo Định lý 2.4, với

f(x) = 1, x, x2, , xm−2, ta ngayS0 =S1 = .=Sm−2 =

b) Để chứng minhSm−1 = 1,ta cần thay f(x)trong Định lý??bởixm−1, so sánh hệ số hạng tử bậc m−1 hai vế đồng thức vừa thu

(101)

Giả sửx1, x2, , xm thoả mãn phương trình bậcm

αm+p1.αm−1 +p2.αm−2+· · ·+pm−1.α+pm = 0,

trong 

         

−p1 =x1+x2+· · ·+xm

p2 =x1x2+x1x3+· · ·+xm−1xm . .

(−1)k.pk =x1x2x3 xk+· · · Nhân hai vế phương trình vớiαk, ta

αm+k+p1.αm+k−1 +p2αm+k−2 +· · ·+pm−1.αk+1+pm.αk = 0.

Thay α đẳng thức bởix1, x2, , xm;và chia đẳng thức thứ cho

(x1−x2)(x1−x3) .(x1−xm), đẳng thức thứ hai cho

(x2−x1)(x2−x3) .(x2−xm)

., cộng vế với vế đẳng thức vừa nhận được, ta thu được

Sm+k+p1.Sm+k−1+· · ·+pm−1.Sk+1+pm.Sk = 0. (2.28) Đặt k= 0, ta thu Sm+p1Sm−1 =

Do đóSm =−p1=x1+x2+· · ·+xm

Nhờ đẳng thức (2.28) ta tính tiếp biểu thức Sm+1, Sm+2, Ta đặt

1

(x1−x2)(x1−x3) .(x1−xm)

=α1;

1

(x2−x1)(x2−x3) .(x2−xm)

=α2;

1

(xmx1)(xmx2) .(xmxm−1) =αm.

Khi ta có

Sn=xn1α1+x22+· · ·+xnmαm Xét

P = α1 1−x1z

+ α2

1−x2z

+· · ·+ αm 1−xmz

(102)

Dùng công thức cấp số nhân với giả thiết rằngz chọn cho |x1z|<1, |x2z|<1, ,|xmz|<1,

ta khai triển tổng P thành chuỗi vô hạn sau :

P =α1(1 +x1z+x21z2+· · ·) +α2(1 +x2z+x22z2+· · ·)+ +· · ·+αm(1 +xm.z+x2mz

2

+· · ·)

hoặc

P = (α1+α2+· · ·+αm) + (x1α1+x2α2+· · ·+xmαm)z+ +(x21α1+x22α2+· · ·+x2mαm)z2+· · ·

tức

P =S0+S1z+S2z2+S3z3+· · ·. Để cho gọn, ta đặt

(1−x1z)(1−x2z) .(1−xmz) = Q. Khai triểnQtheo luỹ thừa củaz, ta viết

Q= 1−δ1z+δ2z2+· · ·+ (−1)mδmzm,

δ1 =x1+x2+· · ·+xm, δ2=x1x2+x1x3+· · ·+xm−1xm

.

Tiếp theo, nhân hai vế đẳng thức thứ hai với

(1−x1z)(1−x2z) .(1−xm.z), ta có

P Q=α1(1−x2z)(1−x3z) .(1−xmz)+

α2(1−x1z)(1−x3z) .(1−xm.z)+

α3(1−x1z)(1−x2z)(1−x4z) .(1−xmz) +· · ·+ αm(1−x1z)(1−x2z) .(1−xm−1z).

Như vậyP Q đa thức bậcm−1 đối vớiz Ta chứng minh nózm−1, tức có đồng thức

(103)

Thật vậy, biểu thứcP Qzm−1 triệt tiêu z=

x1 ,

x2

, , xm vì, chẳng hạn, với

z=

x1

α1

1−x2 x1

1−x3 x1

· · ·

1− xm x1

xm−11 =

1

xm−11 −

1

xm−11 = 0. Vậy nên P Qzm−1 = Do

zm−1 Q =P hay

zm−1

1−δ1z+δ2z2− .+ (−1)mδmzm =S0+S1z+· · ·+Sm−1z

m−1+· · ·

Nếu khai triển vế trái thành chuỗi vơ hạn theo luỹ thừa củazthì chuỗi bắt đầu hạng tử chứa zm−1 Vì vậy, hệ số hạng tử bậc0,1,2, , m−2

trong vế phải không, tức

S0=S1= .=Sm−2 = 0.

Ngoài ra, hệ số hạng tử ứng với bậcm= 1ở vế trái VậySm−1 =

Bây đẳng thức cần chứng minh có dạng sau : zm−1

1−δ1.z+δ2.z2− · · ·(−1)mδm.zm

=zm−1+Sm.zm+Sm+1.zm+1+· · · Uớc lượng hai vế chozm−1, ta thu được

1

1−δ1z+δ2z2− · · ·+ (−1)mδmzm = +Smz+Sm+1z

+· · ·

hoặc

(104)

Khai triển vế phải theo lũy thừa củaz so sánh hệ số hai vế, ta Smδ1 = 0,

δ2−δ1.Sm+Sm+1= 0, .

Như vậy, ta tính Sm, Sm+1, Sm+2,

Nhằm thiết lập mệnh đề mở rộng cấu trúc củaSm+k , ta xét

1

Q =

1 1−x1.z

.

1−x2.z

.

1−xm.z

=

X

p=0 xp1.zp.

X

q=0

xq2.zq· · · ∞

X

s=0

xsmzs=Xx1pxq2 xszp+q+···+s.

Mặt khác

1

Q = +Smz+Sm+1z

+· · ·+Sm+kzk+1+· · ·, nên ta

Sm+k = X

p+q+···+s=k+1

xp1xq2 xsm

Vì vậy, ta thu kết cuối cùngSm+k tổng tích, tích có k+ 1thừa số (giống khác nhau) lấy sốx1, x2, , xm.Nói riêng

Sm+1=x21+x22+· · ·+x2m+x1x2+x1x3+· · ·+xm−1xm Sm+2=x31+x

3

2+· · ·+x

m+x

2

1x2+x21x3+· · ·+x2m−1xm

+x1x2x3+· · ·+xm−2xm−1xm, (điều phải chứng minh) Hệ 2.2. Giả sử

Sk = a

k

(ab)(ac) +

bk

(ba)(bc) +

ck

(ca)(cb).

Khi đó

S0 =S1 = 0, S2= 1, S3 =a+b+c, S4=a2+b2+c2+ab+bc+ca,

(105)

Hệ 2.3. Giả sử Tk =

ak

(ab)(ac)(ad) +

bk

(ba)(bc)(bd)+

ck

(ca)(cb)(cd)+

+ d

k

(da)(db)(dc).

Khi đó

T0=T1 =T2= 0, T3 = 1, T4=a+b+c+d.

Bây ta chuyển sang khảo sát toán nội suy Lagrange ngôn ngữ tổng quát

Bài toán 2.3 (Bài toán nội suy Lagrange). Cho x0i, a0i∈R,vớix0i6=x0j ∀i6= j, (i, j= 1,2, , N).Hãy xác định đa thức L(x) có bậcdegL(x)6N −1 thỏa mãn điều kiện

L(x0i) =a0i, ∀i= 1,2, , N. (2.29) Để đơn giản, trước hết ta đồng nhấtx0i ≡xi, a0i ≡ai ký hiệu

Li(x) =

N

Y

j=1, j6=i

xxj xixj

, (i= 1,2, , N). Khi đó, dễ thấy

Li(xj) =

(

1 i=j

0 i6=j hayLi(xj) ==δij.

Tiếp theo, ta chứng minh đa thức L(x) =

N

X

i=1

aiLi(x) (2.30)

là đa thức thỏa mãn điều kiện toán nội suy Lagrange(2.29), ta gọi đa thức đa thức nội suy Lagrange

Thật vậy, dễ thấy

degL(x)6N−1. Ngồi ra, ta có

L(xi) =

N

X

j=1

ajLj(xi) =

N

X

(106)

hay

L(xi) =ai, ∀i= 1,2, , N.

Cuối cùng, có đa thức L∗(x), có bậc degL∗(x) với degL∗(x) N −1

cũng thỏa mãn điều kiện tốn(2.29)thì đó, đa thức P(x) =L(x)−L∗(x)

cũng có bậcdegP(x)6N −1 thỏa mãn

P(xi) = 0, ∀i= 1,2, , N.

Tức làP(x) đa thức có bậc degP(x) với degP(x) N −1 mà lại có nhấtN nghiệmx1, x2, , xN, nên P(x)≡0, đóL(x) =L∗(x).

Nhận xét 2.4. Về mặt hình học, việc xây dựng đa thức nội suy Lagrange(2.29)

có nghĩa xây dựng đường cong đại số bậc không quá N−1 đi qua tất cả các điểm Mi(xi, yi), ∀i= 1,2, , N cho trước.

2.3 Nội suy Newton khai triển Taylor - Gontcharov Bây giờ, ta chuyển sang xét toán nội suy Newton Ta nhắc lại phát biểu toán nội suy Taylor mục trước

Bài toán 2.4 (Nội suy Taylor). Cho x0, ak ∈ R, với k = 0,1, , N −1. Hãy xác định đa thức T(x) có bậc khơng q N−1 (degT(x)6N−1)thỏa mãn các điều kiện

T(k)(x0) =ak, ∀k= 0,1, , N−1.

Nhận xét xét điểmM(x0, T(k)(x0) (k= 0,1, , N−1),ta thấy chúng nằm đường thẳng x=x0 Khi ta cho x0 thay đổi nhận giá trị tuỳ ý phụ thuộc vàok ta điểm dạng

Mk(xk, T(k)(xk)), k= 0,1, , N−1,

sẽ trùng với điểm ban đầu cácxk trùng Khi ta thu toán nội suy gọi toán nội suy Newton Ta phát biểu tốn dạng sau

Bài toán 2.5 (Bài toán nội suy Newton). Cho xi, ai ∈ R, với i = 1,2, , N. Hãy xác định đa thức N(x) có bậc khơng q N−1 (degN(x)6N−1)và thỏa mãn điều kiện

(107)

Trước hết, ứng với mỗii= 2,3, , N,ta ký hiệu Ri(x1, x2, , xi, x) =

x

Z

x1

xZi−1

x1

. x2

Z

x1

dx1 xi−1.

Tiếp theo, ta xét số trường hợp riêng toán nội suy Newton i) NếuN = 1(ứng với i= 1) ta códegN(x) = N(x1) = a1, do N(x) =a1.

ii) NếuN = 2(ứng vớii= 1,2), ta có

(

N(x) =α1+α2x

N(i−1)(xi) =ai,(i= 1,2)

Từ suy raN(x) =a1+a2(xx1)hay

N(x) =a1+a2R(x1, x). iii) NếuN = 3(ứng với i= 1,2,3), ta có

(

N(x) =α1+α2x+α3x2 N(i−1)(xi) =ai,(i= 1,2,3). Từ suy 

     

α3= a3

2

α2=a2a3x2

α1=a1−(a2a3x2)x1a3

2 x

2 1. Do

N(x) =a1+a2(xx1) +a3

h(xx

2)2

2 −

(x1−x2)2

2

i

, hay

N(x) =a1+a2R(x1, x) +a3R2(x1, x2, x).

iv) Một cách tương tự, trường hợp tổng quát, với i = 1,2, , N, ta chứng minh

(108)

Thật vậy, dễ thấy degN(x) N − 1. Ngoài ra, ứng với i (= 1,2, , N,)ta có

N(i−1)(x) =ai+ai+1R(xi, x) +· · ·+aNRN−i(xi, xi+1, , xN−1, x). Từ suy

N(i−1)(xi) =ai, ∀i= 1,2, , N.

Cuối cùng, ta chứng minh tính nghiệm toán nội suy Newton Giả sử tồn đa thứcN∗(x), có bậcdegN∗(x)6N−1cũng thỏa mãn điều kiện tốn 2.5 đó, đa thức P(x) = N(x)−N∗(x) có bậc

degP(x) vớidegP(x)6N−1và thỏa mãn điều kiện P(i−1)(xi) = 0, ∀i= 1,2, , N.

Khi đó, theo cách xây dựng đa thứcN(x)ở trên, ứng với trường hợpai= 0, ∀i= 1,2, , N, ta suy raP(x)≡0, đóN(x) =N∗(x).

Nhận xét 2.5. Như lưu ý đa thức nội suy Taylor (2.8)là trường hợp riêng đa thức nội suy Newton (2.32) ứng với trường hợp xi =x1, ∀i = 1,2, , N Công thức khai triển hàm sốf(x) thành chuỗi (hữu hạn vô hạn) thoả mãn điều kiện

f(i−1)(xi) =ai, ∀i= 1,2, , N, được gọi khai triển Taylor - Gontcharov.

Công thức khai triển Taylor - Gontcharov có nhiều ứng dụng việc giải toán biên hỗn hợp thứ phương trình vi phân Bạn đọc quan tâm đến vấn đề lý thuyết ứng dụng dạng nội suy trừu tượng nội suy cổ điển xin tìm đọc [3]

2.4 Bài tốn nội suy Hermite

Như ta thấy toán nội suy Newton mở rộng tự nhiên đồng thức Taylor tương ứng, khai triển Taylor - Gontcharov mở rộng khai triển Taylor cổ điển Bây ta chuyển sang xét toán nội suy Hermite mở rộng tự nhiên toán nội suy Lagrange Taylor Với đa thức

L(x) =

n

Y

j=1

(xxj), xi 6=xj khii6=j, i, j= 1,2, , n điều kiện Lagrange

(109)

là điều kiện tự nhiên để xác định đa thức đơn P(x) KhiP(x) có nghiệm bội điều kiện Lagrange khơng đủ để xác địnhP(x) Vì thế, ta cần điều kiện tổng quát để đảm bảo tồn đa thức dạng

H(x) =

n

Y

j=1

(xxj)αj, xi 6=xj khi i6=j, i, j= 1,2, , n.

Rõ ràng đa thức H(x) có degH(x) =α1+α2+· · ·+αn H(k)(xj) = 0, k= 0,1, , αj−1; j = 1,2, , n. Vì thế, ta phát biểu toán nội suy Hermite dạng sau

Bài toán 2.6 (Nội suy Hermite). Cho xi, aki ∈ R, với i = 1,2, , n; k = 0,1, , pi−1 vàxi6=xj ∀i6=j,trong đó p1+p2+· · ·+pn=N.

Hãy xác định đa thức H(x) có bậcdegH(x)6N −1 thỏa mãn điều kiện H(k)(xi) =aki, ∀i= 1,2, , n; ∀k= 0,1, , pi−1. (2.33) Ký hiệu

W(x) =

n

Y

j=1

(xxj)pj

Wi(x) =

W(x) (xxi)pi

=

n

Y

j=1, j6=i

(xxj)pj.

Tiếp theo, giả sử H(x) đa thức có bậcdegH(x) vớidegH(x)6N −1 thỏa mãn điều kiện toán

Ta cần xác đinh hệ sốαli ∈Rsao cho đẳng thức sau thoả mãn H(x)

W(x) =

n

X

i=1 pXi−1

l=0

αli

(xxi)pi−l.

Ta có H(x)

Wi(x)

= (xxi)pi H(x)

W(x) = (xxi)

pi

n

X

i=1 pXi−1

l=0

αli

(xxi)pi−l

=

pXi−1

l=0

αli(xxi)l+ (xxi)pi

n

X

j=1, j6=i pXj−1

l=0

αlj

(110)

Trong phép biến đổi tiếp theo, để ý

(f g)(l)=

l

X

k=0

Clkf(k)g(l−k) p X l=0 l X k=0 Aki= p X k=0 p X l=k Aki, ta thu

hH(x)

Wi(x)

i(l) (x=xi)

=l!αli. Suy αli = l X k=0 aki k!(lk)!

h 1

Wi(x)

i(l−k) (x=xi)

. Do

H(x)

W(x) =

n

X

i=1 pXi−1

l=0 l

X

k=0 aki k!(lk)!

h 1

Wi(x)

i(l−k) (x=xi)

1 (xxi)pi−l

=

n

X

i=1 pXi−1

k=0 pXi−1

l=k

aki k!(lk)!

h 1

Wi(x)

i(l−k) (x=xi)

1 (xxi)pi−l

. Suy

H(x) =W(x)

n

X

i=1 pXi−1

k=0 pXi−1

l=k

aki k!(lk)!

h 1

Wi(x)

i(l−k) (x=xi)

1 (xxi)pi−l hay

H(x) =

n

X

i=1 pXi−1

k=0

aki(xxi) k k! Wi(x)

pXi−1

l=k

h 1

Wi(x)

i(l−k) (x=xi)

(xxi)l−k (lk)! .

Đổi số tổng cuối đẳng thức trên, ta thu H(x) =

n

X

i=1 pXi−1

k=0

aki(xxi) k k! Wi(x)

piX−1−k

l=0

h 1

Wi(x)

i(l) (x=xi)

(xxi)l l!

hay

H(x) =

n

X

i=1 pXi−1

k=0 aki

(xxi)k

k! Wi(x)T

n 1

Wi(x)

o(pi−1−k)

(x=xi)

,

T

n 1

Wi(x)

o(pi−1−k)

(x=xi) =

piX−1−k

l=0

h 1

Wi(x)

i(l) (x=xi)

(xxi)l

(111)

là đoạn khai triển Taylor đến cấp thứ (pi−1−k) x=xi hàm số Wi(x)

. Ký hiệu

Hki(x) =

(xxi)k

k! Wi(x) T

n 1

Wi(x)

o(pi−1−k)

(x=xi)

. Khi đó, dễ thấy

degHki(x)6k+ (Npi) + (pi−1−k) =N −1, Hki(l)(xj) =

(

1 k=l i=j

0 k6=l i6=j hayHki(l)(xj) =δklδij.

Bây giờ, ta chứng minh n

X

i=1 pXi−1

k=0

akiHki(x) = n

X

i=1 pXi−1

k=0 aki

(xxi)k

k! Wi(x)T

n 1

wi(x)

o(pi−1−k)

(x=xi)

(2.34) hay

H(x) =

n

X

i=1 pXi−1

k=0

aki(xxi) k

k! Wi(x)T

n 1

wi(x)

o(pi−1−k)

(x=xi)

là đa thức thỏa mãn điều kiện toán nội suy Hermite 2.33 ta gọi đa thức(2.34)làđa thức nội suy Hermite

Thật vậy, dễ thấy rằngdegH(x)6N −1 H(k)(xi) =

n

X

j=1 pXj−1

l=0

aljHlj(k)(xi) =

n

X

j=1 pXj−1

l=0

aljδklδij. Suy

H(k)(xi) =aki, (∀i= 1,2, , n; ∀k= 0,1, , pi−1).

Cuối cùng, ta chứng minh tính nghiệm toán nội suy Hermite Giả sử tồn đa thức H∗(x), có bậc degH∗(x) với degH∗(x) N −1, thỏa mãn điều kiện toán nội suy Hermite Khi đó, đa thức P(x) =

H(x)−H∗(x) có bậc degP(x)6N −1và thỏa mãn điều kiện P(k)(xi) = 0, (∀i= 1,2, , n; ∀k= 0,1, , pi−1).

Khi đó, theo cách xây dựng đa thức H(x) phần trên, ứng với trường hợp aki= 0, ∀i= 1,2, , n; ∀k= 0,1, , pi−1, ta suy raP(x)≡0

(112)

Chú ý đa thức nội suy Taylor đa thức nội suy Lagrange trường hợp riêng đa thức nội suy Hermite

Thật vậy, đa thức nội suy Taylor trường hợp riêng đa thức nội suy Hermite đa thức nội suy Hermite (2.33) ứng vớin= 1thì ta có

p1 =N, W(x) = (xx1)p1, W1(x)≡1. Suy

Hk1(x) =

(xx1)k k! .

Vậy nên

H(x) =

NX−1 k=0

ak1

(xx1)k

k! ≡T(x).

Tương tự, đa thức nội suy Lagrange trường hợp riêng đa thức nội suy Hermite đa thức nội suy Hermite (2.33), ứng vớik= 0, ta có

pi = ∀i= 1,2, , n, n=N. Suy

H0i(x) =

Wi(x) Wi(xi)

=

N

Y

j=1, j6=i

xxj xixj

=Li(x). Vậy nên

H(x) =

N

X

i=1

a0iLi(x)≡L(x).

Tiếp theo, ta xét số trường hợp riêng đơn giản đa thức nội suy Hermite Xét đa thức nội suy Hermite (2.33)

H(x) =

n

X

i=1 pXi−1

k=0

aki(xxi) k

k! Wi(x)T

n 1

Wi(x)

o(pi−1−k)

(x=xi)

,

T

n 1

Wi(x)

o(pi−1−k)

(x=xi) =

piX−1−k

l=0

h 1

Wi(x)

i(l) (x=xi)

(xxi)l

l! .

(113)

Trong đa thức nội suy Hermite (2.33), nếupi = 2, ∀i= 1,2, , nthì k= 0,1và ta có

H(x) =

n X i=1 X k=0

aki(xxi) k

k! Wi(x) T

n 1

Wi(x)

o(1−k) (x=xi)

. Suy

H(x) =

n

X

i=1 Wi(x)

h

a0i T

n 1

Wi(x)

o(1) (x=xi)

+a1i(xxi) T

n 1

Wi(x)

o(0) (x=xi)

i

hay

H(x) =

n

X

i=1

Wi(x) Wi(xi)

h

a0i

1−Wi

0

(xi) Wi(xi)

(xxi)

+a1i(xxi)

i = = n X i=1

Wi(x) Wi(xi)

h

a0i

a0iWi

0

(xi) Wi(xi) −a1i

(xxi)

i

. Cuối cùng, để ý ta có đồng thức

Wi(x)

Wi(xi) =

n

Y

j=1, j6=i

(xxj)2

(xixj)2 =L i(x), Wi0(xi)

Wi(xi)

=

h Yn

j=1, j6=i

xxj xixj

i0

x=xi

= 2L0i(xi).

Vậy, ta thu đa thức nội suy Hermite trường hợp sau H(x) =

n

X

i=1 L2i(x)

h

a0i− 2a0iL0i(xi)−a1i(xxi)

i

.

Tiếp theo ta xét trường hợp hệ điều kiện chứa giá trị đạo hàm cấp mốc nội suy trừ điểm Trong đa thức nội suy Hermite (2.33), ta xét trường hợp đặc biệt, chẳng hạn ∃k0, i0 cho ak0i0 = a 6=

aki= 0, ∀(k, i)6= (k0, i0). Khi đó, ta có

H(x) =ak0i0

(xxi0)

k0

k0!

Wi0(x)T

n 1

Wi0(x)

o(pi0−1−k0)

(x=xi0)

(114)

Nếu k0=p0−1 chọn ak0i0 = (pi0 −1)!Wi0(xi0) ta có

H(x) = W(x)

xxi0 Vậy, trường hợp đặc biệt

(

H(k)(xi0) = 0, ∀k= 0,1, , pi0−2; H(pi0−1)(xi

0) = (pi0−1)!Wi0(xi0);

H(k)(xi) = 0, ∀k= 0,1, , pi−1, ∀i= 1,2, , n, i6=i0 đa thức nội suy Hermite có dạng

H(x) = W(x)

xxi0

= (x−x1)p1· · ·(x−xi0−1)

pi0(x−x

i0)

pi0−1(x−x

i0+1)

pi0 .(x−x

n)pn. Ngược lại, đa thức có dạng

H(x) = (xx1)α1(xx2)α2· · ·(xxn)αn, xi 6=xj ∀i6=j, i, j= 1,2, , n

đều nghiệm toán Hermite

(

H(k)(xi) = 0, ∀k= 0,1, , αi−1, i= 1,2, , n H(αi0)(x

i0) =αi0!Wi0(xi0).

Chẳng hạn, ta viết

H(x) = (xx1)α1(xx2)α2 (xxn)αn =

(xx1)α1+1(xx2)α2 (xxn)αn xx1

thì H(x)là nghiệm tốn Hermite sau

(

H(k)(x1) = 0, ∀k= 0,1, , α1−1, H(α1)(x1) =α1!W1(x1) H(k)(x

i) = 0, ∀k= 0,1, , αi−1, ∀i= 1,2, , n, i6= 1.

Tiếp theo, phần ta nêu số ví dụ áp dụng kỹ thuật để xác định đa thức biết số đặc trưng chúng dạng nút nội suy

Bài toán 2.7. Cho 0< α <1 Xác định tất đa thức f(x) bậcn (n>2) sao cho tồn dãy số r1, r2, , rn (r1< r2 < < rn) thoả mãn điều kiện

sau (

f(ri) = 0,

(115)

Giải Nhận xét vớia < b x= αa+ (1−α)b ; α ∈(0,1)thì x∈ (a, b) Khi

p=

xa+

1

xb =

2α−1

α(1−α)(ba).

Do p >0 khiα >

2,p <0 α <

2 p=

chỉ α=

2 Theo giả thiết

f(x) =c n

Y

i=1

(xri) nên

f0(x) f(x) =

n

X

i=1

1

xri. Với n >3 < α6

2 ta đặt x =αr1 + (1−α)r2 Khi theo giả thiết

f0(x) = đồng thời ta lại có f0(x)

f(x) =

xr1

+

xr2

+

n

X

i=3

1

xri <0,

mâu thuẫn Tương tự vớin>3

2 < α <1 ta nhận điều vô lý Nếu

n= 2vàα6=

2,thì tương tự dẫn đến điều mâu thuẫn Do

cần xét trường hợp n = vàα =

2 Khi tam thức bậc hai có nghiệm

phân biệt thoả mãn toán cho

Bài toán 2.8. Xác định tất đa thức P(x) bậc nhỏ thua n và thoả mãn điều kiện

n

X

k=0

(−1)kCnkP(k) = 0.

Giải Sử dụng công thức nội suy Lagrange với nút nội suy xk =k, ta mọi đa thức P(x)bậc nhỏ thua nđều có dạng

P(x) =

n−1

X

k=0

(116)

nên

P(x) =

n−1

X

k=0

P(k)(x−0)· · ·(x−(k−1))(x−(k+ 1))· · ·(k−(n−1)) (k−(k−1))· · ·(k−(k+ 1))· · ·(k−(n−1)) .

Ta có

P(n) =

n−1X k=0

P(k)(n−0)· · ·(nk+ 1)(nk−1)· · ·1

k!(−1)n−k(nk−1)! . Suy

(−1)nCnnP(n) =

n−1

X

k=0

(−1)kCnkP(k). Vậy điều kiện(??) thỏa mãn

Tóm lại, đa thức cần tìm có dạng P(x) =

n−1

X

k=0

P(k)(x−0)· · ·(x−(k−1))(x−(k+ 1))· · ·(k−(n−1)) (k−(k−1))· · ·(k−(k+ 1))· · ·(k−(n−1)) ,

trong P(0), P(1), , P(n−1) giá trị tùy ý

Tiếp theo ta xét số tóan liên quan đến nội suy theo xấp xỉ Diophane Bài toán 2.9. Chứng minh tồn đa thức Pn(x) bậc n (n>1) với hệ số nguyên cho

Pn(x)−1

2

<

1000 ∀x∈

h1

10, 10

i

. Giải Xét đa thức

Pn(x) =

2[(2x−1)

n

+ 1].

Ta thấy hệ số củaPn(x)là số nguyên Khi vớix

h1

10, 10

i

ta có

Pn(x)−

1

=

2|2x−1|

n6

2(0,8)

n ∀n∈N∗.

(117)

Bài toán 2.10. Cho hai số nguyên dươngp, q Chứng minh tồn đa thức Pn(x) bậc nvới hệ số nguyên cho

Pn(x)−p q

<

q2 với mọix thuộc khoảng

1

2q,

3 2q

.

Giải Vớiq = 1thì ta chọnP(x)≡p Vớiq >1thì ta thấy khoảngI =

1

2q,

3 2q

có độ dài 1/q <1 Chọn m∈Nsao cho

3

2q

m <

q Chọn nđủ lớn để an<

pq với a= 1−

1

2q

m

và đặt

Pn(x) = p

q[1−(1−qx m

)n].

Khi rõ ràngPn(x)là đa thức với hệ số nguyên vớixI

Pn(x)−p q

= p

q

(1−qxm)n

< p

qa n

< q2

Pn(x) =

2[(2x−1)

n

+ 1]. Ta thấy hệ số củaPn(x)là số nguyên

Bài toán 2.11.Chứng minh không tồn đa thứcf(x)∈Z[x]màf(2005) = 2005 f(2007) = 2008.

Giải Giả sử

f(x) =anxn+an−1xn−1+· · ·+a0, ai ∈Z ∀i∈ {0,1, , n}. Khi đó, ta có

f(2007)−f(2005) =an(2007n−2005n)

(118)

Mặt khácf(2007)−f(2005) = 2008−1995 = 3và không chia hết cho2 Vậy không tồn đa thứcf(x) thỏa mãn điều kiện đề

Tiếp theo, ta xây dựng số toán nội suy hỗn hợp nhằm mở rộng toán nội suy cổ điển Việc xây dựng toán nội suy hỗn hợp thực chất tổ hợp toán nội suy cổ điển biết, nội suy Lagrange - Newton, nội suy Newton - Hermite, Việc mở rộng toán nội suy cổ điển dựa ý tưởng thêm mốc nội suy Ta khảo sát nghiệm tồn nghiệm toán nội suy cổ điển Bây giờ, ta chuyển sang xét xem với trường hợp cụ thể điều kiện bổ sung bảo tồn tồn nghiệm toán tương ứng

2.5 Bài toán nội suy Lagrange - Newton Trước hết ta xét toán nội suy hỗn hợp Lagrange - Newton

Bài toán 2.12 (Nội suy Lagrange - Newton). Cho xki, aki ∈ R, với xki 6= xkj ∀i6=j; k= 0,1, , n−1; i, j= 1, , rk+1;trong đó

r0= 0, r0+r1+· · ·+rk =sk, r0+r1+· · ·+rn=sn =N. Hãy xác định đa thức f(x) có bậcdegf(x)6N −1 và thỏa mãn điều kiện

f(sk)(x

ki) =aki, ∀k= 0,1, , n−1, ∀i= 1, , rk+1. (2.35) Ký hiệu

Lki(x) = rYk+1

j=1, j6=i

xxkj

xkixkj, (k= 0,1, , n−1)và phép lấy nguyên hàmR= x

Z

0 .

Khi đó, dễ thấy

degLki(x)6rk+1−1, Lki(xkj) =

(

1 i=j

0 i6=j hayLki(xkj) =δij.

Tiếp theo, ta đặtyn(x)≡0 xây dựng dãy hàm yk(x) =Rsk

rXk+1

i=1

[akiy (sk)

(119)

Dễ thấy

degyk(x)6N−1 degyk(x)< sk+1. Bây giờ, ta chứng minh đa thức

f(x) =y0(x) +y1(x) +· · ·+yn−1(x)

chính nghiệm toán nội suy Lagrange - Newton (2.35) Thật vậy, dễ thấy rằngdegf(x)6N−1.Ngồi ra, ta có

f(sn−1)(x

(n−1)i) = (y0+· · ·+yn−2+yn−1)(sn−1)(x(n−1)i)

=y(sn−1)

n−1 (x(n−1)i)

=

rn

X

j=1

a(n−1)j.L(n−1)j(x(n−1)i)

=

rn

X

j=1

a(n−1)j.δij =a(n−1)i f(sn−2)(x

(n−2)i) = (y0+· · ·+yn−3+yn−2+yn−1)(sn−2)(x(n−2)i)

=y(sn−2)

n−2 (x(n−2)i) +y (sn−2)

n−1 (x(n−2)i),

y(sn−2)

n−2 (x(n−2)i) = rXn−1

j=1

[a(n−2)j−y (sn−2)

n−1 (x(n−2)j)].L(n−2)j(x(n−2)i)

=

rXn−1

j=1

[a(n−2)j−y (sn−2)

n−1 (x(n−2)j)].δij

=a(n−2)i−y (sn−2)

n−1 (x(n−2)i). Vậy nên

f(sn−2)(x

(n−2)i) = a(n−2)i. Bằng cách tương tự, ta chứng minh

f(sk)(x

ki) =aki, ∀k= 0,1, , n−1, ∀i= 1, , tk+1.

Cuối cùng, có đa thức f∗(x) với bậc degf∗(x) N −1 thỏa mãn điều kiện toán (2.35), đa thức

(120)

cũng có bậcdegP(x)6N −1 thỏa mãn điều kiện

P(sk)(xki) = 0, ∀k= 0,1, , n−1, ∀i= 1, , tk+1.

Khi đó, theo cách xây dựng đa thứcf(x) phần trên, ứng với trường hợp aki = 0, ∀k= 0,1, , n−1, ∀i= 1, , tk+1,

ta có

yk = 0, ∀k= 0,1, , n−1.

Suy P(x)≡0, f(x) =f∗(x).

Nhận xét 2.6. Nếu k= 0, tương ứng r2 = r3 =· · ·= rn = 0,thì sk = 0 Khi đó tốn nội suy Lagrange - Newton tốn nội suy Lagrange quen biết.

Nếu i= 1, tương ứng r1 =r2=· · ·=rn= 1,thì sk =k Khi tốn nội suy Lagrange - Newton tốn nội suy Newton biết.

2.6 Bài toán nội suy Newton - Hermite Ta xét toán nội suy hỗn hợp sau

Bài toán 2.13 (Nội suy Newton - Hermite). Cho số

xki, ahki∈R k= 0,1, , n−1, i= 1, , rk+1, h= 0, , pki−1, xki 6=xkj∀i6=j, trong đó

pk1+· · ·+pkrk+1 =Nk+1, N0+· · ·+Nk =sk, N0+· · ·+Nn=sn=N, N0= 0. Hãy xác định đa thức f(x) có bậcdegf(x)6N −1 và thỏa mãn điều kiện

f(sk+h)(xki) =ahki, k= 0,1, , n−1, i= 1, , rk+1, h= 0, , pki−1.

(2.36) Trước hết, ta sử dụng ký hiệu

(Rf)(x) =

x

Z

0

f(t)dt, Wki(x) = rYk+1

j=1, j6=i

(xxkj)pkj,

Hhki(x) =

(xxki)h

h! Wki(x)

pki−1−h

X

l=0

h 1

Wki(x)

i(l) (x=xki)

(xxki)l

(121)

Khi đó, dễ thấy

degWki(x)6Nk+1pki, degHhki(x)6Nk+1−1

Hhki(l)(xkj) =

(

1 l=h i=j

0 l6=h i6=j hayHhki(l)(xkj) =δlhδij.

Tiếp theo, ta đặtfn≡0 xây dựng dãy hàm yk(x) =Rsk

rXk+1

i=1 pXki−1

h=0

[ahkif (sk+h)

k+1 (xki)]Hhki

(x), fk(x) =yk(x) +fk+1(x), k=n−1, ,1,0.

Khi đó, ta dễ dàng thấy

deg yk(x)6sk+Nk+1−1 =sk+1−1. Suy deg yk(x)6N−1và deg yk(x)< sk+1.

Bây giờ, ta chứng minh đa thức

f(x) =f0(x) =y0(x) +y1(x) +· · ·+yn−1(x) nghiệm toán nội suy Newton - Hermite(2.36)

Thật vậy, dễ thấy rằngdegf(x)6N−1.Ngồi ra, ta có f(sn−1+h)(x

(n−1)i) = (y0+· · ·+yn−2+yn−1)(sn−1+h)(x(n−1)i)

=y(sn−1+h)

n−1 (x(n−1)i)

=

rn

X

j=1

p(nX−1)j−1

l=0

al(n−1)j.Hl(n−1)j(h) (x(n−1)i)

=

rn

X

j=1

p(nX−1)j−1

l=0

al(n−1)j.δlhδij =ah(n−1)i f(sn−2+h)(x

(n−2)i) = (y0+· · ·+yn−2+yn−1)(sn−2+h)(x(n−2)i)

=y(sn−2+h)

n−2 (x(n−2)i) +y

(sn−2+h)

n−1 (x(n−2)i),

y(sn−2+h)

n−2 (x(n−2)i) = rXn−1

j=1

p(nX−2)j−2

l=0

[al(n−2)jf

(sn−2+h)

n−1 (x(n−2)j)].δlhδij

=ah(n−2)if(sn−2+h)

n−1 (x(n−2)i) =ah(n−2)iy

(sn−2+h)

(122)

Suy

f(sn−2+h)(x

(n−2)i) =ah(n−2)i. Một cách tương tự, ta chứng minh

f(sk+h)(xki) =ahki, ∀k= 0,1, , n−1, i= 1, , rk+1, h= 0, , pki−1.

Tiếp theo, ta chứng minh tính nghiệm toán nội suy Newton -Hermite Giả sử tồn đa thứcf∗(x) có bậc degf∗(x) vớidegf∗(x)6N −1và thỏa mãn điều kiện tốn (2.36), đa thức P(x) =f(x)−f∗(x) có bậcdegP(x)6N −1 thỏa mãn điều kiện

P(sk+h)(x

ki) = 0, k= 0,1, , n−1, i= 1, , rk+1, h= 0, , pki−1. Khi đó, theo cách xây dựng đa thứcf(x) phần trên, ứng với trường hợp

ahki = 0, k= 0,1, , n−1, i= 1, , rk+1, h= 0, , pki−1, ta thu đượcyk = 0, ∀k= 0,1, , n−1.

Suy raP(x)≡0và f(x) =f∗(x).

Nhận xét 2.7. Chú ý khi k = 0, tương ứng r2 = r3 = · · · =rn = 0, thì sk+h =h Khi tốn nội suy Newton - Hermite (2.36) chính tốn nội suy Hermite (2.33)đã xét trên.

Nếu h= 0, tương ứng với trường hợp

pki= 1, ∀k= 0,1, , n−1, i= 1, , rk+1,

thì sk +h = sk Khi tốn nội suy Newton - Hermite (2.36) chính là bài toán nội suy Lagrange - Newton (2.35) vừa xét Lúc này, nếu r1=· · ·=rn= 1thì tốn nội suy Newton - Hermite (2.36)chính toán nội suy Newton(2.32)đã khảo sát.

Bài tập

Bài 2.1. Xác định tam thức bậc hai f(x) thoả mãn điều kiện f(n) = (−1)n(2n2−n−1), n= 4,7,16.

(123)

Bài 2.3. Xác định tam thức bậc hai f(x) thoả mãn điều kiện f(n)(2n+ 1) = (−1)n(2n2−n−1), n= 0,1,2. Bài 2.4. Xác định đa thức bậc ba f(x) thoả mãn điều kiện

f(n)(3n+ 1) =n3−3n2+n+ 1), n= 0,1,2,3. Bài 2.5. Xác định đa thức bậc ba f(x) thoả mãn điều kiện

f(n)(1) =n3−3n2+n+ 1), n= 0,1,2,3. Bài 2.6. Xác định đa thức bậc ba f(x) thoả mãn điều kiện

f(n)(1) =n3−3n2+n+ 1), n= 0,1,2; f(2) =n. Bài 2.7. Xác định đa thức bậc năm f(x) thoả mãn điều kiện

(

f(n)(1) =n5, n= 0,1,2 f(n)(−1) =n4−n, n= 0,1,2.

Bài 2.8. Xác định đa thức bậc năm f(x) thoả mãn điều kiện

    

f(n)(1) =n5, n= 0,1

f(n)(−1) =n4−n, n= 0,1

f(n)(2) =n3−n2, n= 0,1

Bài 2.9. Xác định đa thức bậc năm f(x) thoả mãn điều kiện

    

f(n)(1) =n5, n= 0,1

f(n)(−1) =n4−n, n= 0,1

f(2) =−2, f(3) =

Bài 2.10. Xác định đa thức f(x) bậcn (n >6)và thoả mãn điều kiện

    

f(n)(1) =n, n= 0,1

f(n)(−1) =n2−n−1, n= 0,1

f(k) = (−1)k(2k+ 1), k= 3, , n−1

Bài 2.11. Xác định đa thức f(x)bậc6n (n >6)và thoả mãn điều kiện

(

f(n)(n) =n2−n−1, n= 0,1,2

(124)

Nội suy theo yếu tố hình học và biểu diễn hàm

Trong chương này, ta xét số toán nội suy liên quan đến giả thiết điểm đặc biệt đồ thị điểm dừng, điểm cực đại, cực tiểu, điểm uốn, Ngồi ra, ta cịn xét số dạng biểu diễn khai triển liên quan đến ước lượng hàm số xác định giá trị lớn nhất, nhỏ số biểu thức 3.1 Nội suy theo nút điểm dừng đồ thị

Nhờ dáng điệu hình học đồ thị ta đặt giải số toán nội suy với đặc trưng hình học cho trước điểm cực đại, cực tiểu, điểm uốn, đại lượng hình học khác khoảng cách, hệ số góc tiếp tuyến Bài toán 3.1. Xác định tam thức bậc hai f(x) =x2+bx+c khi đồ thị của hàm số y=f(x) đạt cực tiểu tại M(0,1).

Giải. Theo giả thiết đồ thị hàm số quaM(0,1)vàx= 0là điểm dừng hàm sốf(x) Vậy nên

f(0) = 1, f0(0) = 0, hay

c= 1, b= 0. Vậy tam thức bậc hai cần tìm có dạng

f(x) =x2+ 1.

Bài toán 3.2. Xác định tam thức bậc hai f(x) =x2+bx+c khi cặp đường phân giác góc thứ góc thứ hai hệ trục toạ độ tiếp tuyến đồ thị hàm số y=f(x).

(125)

Giải. Nhận xét đường phân giác góc thứ tiếp tuyến đồ thị hàm số cho điểm M(x0, f(x0))trên đồ thị phải có

f(x0) =x0, f0(x0) = 1. Vậy ta thu hệ (

x20+bx0+c=x0

2x0+b= (3.1)

Tương tự, đường phân giác góc thứ hai tiếp tuyến đồ thị hàm số cho điểmM(x1, f(x1) đồ thị ta phải có

f(x1) =−x1, f0(x1) =−1. Vậy ta thu hệ (

x21+bx1+c=−x1

2x1+b=−1

(3.2) Giải hệ (3.1) (3.2), ta thu b= 0,c=

2

Vậy tam thức bậc hai cần tìm có dạngf(x) =x2+1

2

Bài toán 3.3. Xác định đa thức P(x) bậc nhỏ cho đồ thị hàm số y=P(x) có điểm cực đại, cực tiểu tại A(0,1)vàB(1,0).

Giải. Theo giả thiết đa thứcP(x) có hai điểm dừng Do phương trình P0(x) = có hai nghiệm Suy degP0(x) > Xét trường hợp

degP0(x) = Khi đó, theo giả thiết

P(x) =ax3+bx2+cx+d, P0(x) = 3ax2+ 2bx+c, a6=

P0(0) = 0, P0(1) = 0, P(0) = 1, P(1) = 0. Suy c= 0,d= 1và (

a+b+ = 3a+ 2b=

Giải hệ này, ta thu a= 2,b=−3 Vậy đa thức bậc ba P(x) có dạng P(x) = 2x3−3x2+ 1.

Thử lại, ta thấy điều kiện đảm bảo để đồ thị hàm số y=P(x) có điểm cực đại, cực tiểu A(0,1)và B(1,0) thoả mãn Vậy đa thức P(x)

bậc nhỏ cho đồ thị hàm số y=P(x) có điểm cực đại, cực tiểu tạiA(0,1)và B(1,0)là đa thức

(126)

Bài toán 3.4. Xác định đa thức P(x) bậc nhỏ cho đồ thị hàm số y=P(x) có điểm cực tiểu tại A(0,1) vàB(1,1).

Giải. Theo giả thiết đa thứcP(x)có ba điểm dừng Thật vậy, hàm số P(x) có điểm dừng x= 0,x= 1và từ giả thiếtP(0) =P(1)(= 1), theo định lý Rolle, ta tìm khoảng (0,1) điểm x0 cho P0(x0) = Do phương trình P0(x) = 0có ba nghiệm phân biệt Suy degP0(x) > Xét trường hợpdegP0(x) = Khi đó, theo giả thiết

P(x) =ax4+bx3+cx2+dx+e, P0(x) = 4ax3+ 3bx2+ 2cx+d, a6=

P0(0) = 0, P0(1) = 0, P(0) = 1, P(1) = 1. Suy

P(x) =ax2(x−1)2+ 1, a6= 0.

Để đồ thị hàm số y = P(x) có điểm cực tiểu A(0,1) B(1,1)

thì a >0 Vậy đa thức bậc bốn cần tìm có dạng

P(x) =ax2(x−1)2+ 1, a >0.

Thử lại ta thấy điều kiện đảm bảo đồ thị hàm số y = P(x) có điểm cực tiểu A(0,1) B(1,0)là thoả mãn Vậy đa thức P(x) bậc nhỏ cho đồ thị hàm sốy=P(x) có điểm cực tiểu tạiA(0,1)và B(1,0)là đa thức

P(x) =ax2(x−1)2+ 1, a >0.

Bài toán 3.5. Xác định đa thức P(x) bậc nhỏ cho đồ thị hàm số y=P(x) có điểm uốn điểm cực tiểu tại A(0,0)vàB(1,0). Giải. Theo giả thiết đa thứcP(x)có ba điểm dừng Thật vậy, hàm số P(x) có điểm dừng x= 0,x= 1và từ giả thiếtP(0) =P(1)(= 0), theo định lý Rolle, ta tìm khoảng (0,1) điểm x0 cho P0(x0) = Do phương trình P0(x) = có bốn nghiệm (một nghiệm kép hai nghiệm đơn phân biệt) Suy degP0(x) > 4 Xét trường hợp degP0(x) = 4 Khi đó, theo giả thiết

P(x) =ax5+bx4+cx3+dx2+ex+h, P0(x) = 5ax4+ 4bx3+ 3cx2+dx+e, P00(x) = 20ax3+ 12bx2+ 6cx+d, a 6=

(127)

Suy

P(x) =ax3(x−1)2, a6= 0.

Để đồ thị hàm số y=P(x)có điểm uốn điểm cực tiểu A(0,0)

B(1,0)thìa >0 Vậy đa thức bậc bốn P(x) có dạng P(x) =ax3(x−1)2, a >0.

Thử lại, ta thấy điều kiện đảm bảo đồ thị hàm sốy =P(x)có điểm uốn điểm cực tiểu A(0,0)và B(1,0)là thoả mãn Vậy đa thức P(x)

bậc nhỏ cho đồ thị hàm số y=P(x) có điểm uốn điểm cực tiểu tạiA(0,0)vàB(1,0)là đa thức

P(x) =ax3(x−1)2, a >0.

Bài toán 3.6. Xác định đa thức P(x) bậc nhỏ cho đồ thị hàm số y = P(x) đi qua điểm A(2,0) và có điểm cực đại tại B(0,0) C(1,0).

Giải. Theo giả thiết đa thức P(x)có bốn điểm dừng kể bội Thật vậy, đa thức P(x) có nghiệm bội tạix = 0,x = nghiệm x = Do phương trình P(x) = 0có năm nghiệm (hai nghiệm kép nghiệm đơn) Suy đa thức bậc năm tương ứng có dạng

P(x) =ax2(x−1)2(x−2), a6= 0.

Để đồ thị hàm số y=P(x) qua điểmA(2,0)và có điểm cực đại B(0,0)vàC(1,0)thìa >0 Vậy đa thức bậc năm P(x) có dạng

P(x) =ax2(x−1)2(x−2), a >0.

Thử lại, ta thấy điều kiện đảm bảo đồ thị hàm số y = P(x) qua điểm A(2,0)và có điểm cực đại tạiB(0,0)vàC(1,0)là thoả mãn Vậy đa thứcP(x) bậc nhỏ cho đồ thị hàm số y=P(x) qua điểm A(2,0)và có điểm cực đại tạiB(0,0)vàC(1,0)là đa thức

P(x) =ax2(x−1)2(x−2), a >0. 3.2 Hàm số chuyển đổi tam giác

Ta nhắc lại số hệ thức đặc trưng đơn giản mô tả ràng buộc tự nhiên yếu tố cạnh góc tam giác

(128)

Tính chất 3.2. Điều kiện cần đủ để ba số dương a, b, c(khi gắn với một đơn vị đo lường) lập thành độ dài cạnh tam giácABC

a+b > c, b+c > a, c+a > b.

Nói cách khác, điều kiện cần đủ để ba số dươnga, b, clập thành độ dài ba cạnh tam giácABC

|b−c|< a <|b+c|.

Trong phần này, ta tìm lời giải cho số toán xác định hàm số thực phép chuyển tiếp yếu tố hình học tam giác

Bài tốn I (Nội suy hình học)

Xác định hàm sốf(x)liên tục đoạn[0;π]sao chof(A), f(B), f(C)

tạo thành độ đo góc tam giác ứng với tam giác ABC cho trước.

Bài tốn II (Nội suy hình học)

Xác định hàm sốf(x) liên tục trênR+sao chof(a), f(b), f(c)tạo thành độ đo cạnh tam giác ứng với tam giácABC cho trước.

Trước hết, ta khảo sát đặc trưng hàm số hàm số sinh phép biến hình sơ cấp dạng tịnh tiến, đồng dạng, phản xạ nghịch đảo đường thẳng thực

Bài toán 3.7. Xác định α để hàm số f(x) = x+α có tính chất là f(a), f(b), f(c) luôn lập thành độ dài cạnh tam giác ứng với tam giácABC cho trước.

Giải Đểf(a), f(b), f(c)là độ dài cạnh tam giác, trước hết ta phải có

f(a)>0, f(b)>0, f(c)>0, ∀∆ABC Suy

a+α >0, b+α >0, c+α >0, ∀∆ABC hay

α >−a, α >−b, α >−c, ∀∆ABC. Điều tương đương với điều kiện

α > max{ −a,−b,−c}, ∀∆ABC. Do đóα>0

(129)

Thật vậy, ta có

f(a) +f(b)> f(c), f(b) +f(c)> f(a), f(c) +f(a)> f(b), tức

a+α+b+α > c+α, b+α+c+α > a+α, c+α+a+α > b+α, hay

α+a+b > c, α+b+c > a, α+c+a > b. Doα>0nên

α+a+b>a+b > c, α+b+c>b+c > a, α+c+a>c+a > b. Vậy vớiα >0 hàm số f(x) = x+α có tính chất f(a), f(b), f(c) lập thành độ dài cạnh tam giác ứng với tam giác ABC cho trước Bài toán 3.8. Xác định α để hàm số f(x) =αx có tính chất là f(a), f(b), f(c)

lập thành độ dài cạnh tam giác ứng với tam giác ABC cho trước.

Giải Đểf(a), f(b), f(c)là độ dài cạnh tam giác, trước hết ta phải có:

f(a)>0, f(b)>0, f(c)>0, ∀∆ABC. Suy αa >0, αb >0, αc >0,∀∆ABC Do đóα >0

Thật vậy, nếuα60 f(a)60, vơ lý

Ngược lại, vớiα >0 thìf(a), f(b), f(c)là độ dài cạnh tam giác ba a, b, clà độ dài ba cạnh tam giácABC Thật vậy, ta có

    

f(a) +f(b)> f(c)

f(b) +f(c)> f(a)

f(c) +f(a)> f(b)

tức

+bα > cα, bα+cα > aα, cα+aα > bα

a+b > c, b+c > a, c+a > b.

(130)

Giải.

Đểf(a), f(b), f(c)là độ dài cạnh tam giác, trước hết ta phải có f(a)>0, f(b)>0, f(c)>0, ∀∆ABC.

Suy

αa+β >0, αb+β >0, αc+β >0,∀∆ABC. (3.3) Do đóα >0 Thật vậy, nếuα <0, βtuỳ ý cho trước ta chọn tam giácABCađủ lớn, theo tính chất nhị thức bậc ta nhận đượcαa+β <0 Tương tự, từ (3.3) suy raβ >0 Thật vậy, β <0 ta chọn tam giác ABCa đủ nhỏ, theo tính chất nhị thức bậc ta nhận αa+β <0

Trường hợp đồng thời xảy raα= 0, β= thìf(x) ≡0 khơng thoả mãn tốn

Ngược lại, với α >0, β >0, α+β > ta thấy f(a), f(b), f(c) độ dài cạnh tam giác a, b, clà độ dài ba cạnh tam giácABC Thật vậy, ta có

f(a) +f(b)> f(c), f(b) +f(c)> f(a), f(c) +f(a)> f(b),

tức 

   

αa+β+αb+β > αc+β αb+β+αc+β > αa+β αc+β+αa+β > αb+β

hay 

   

α(a+b) +β > αc α(b+c) +β > αa α(c+a) +β > αb Điều hiển nhiên α>0, β>0, α+β >0.

Vậy vớiα > 0, β > 0, α+β > hàm số f(x) = αx+β có tính chất f(a), f(b), f(c)luôn lập thành độ dài cạnh tam giác ứng với tam giác ABC cho trước

Bài toán 3.10. Xác địnhα, β để hàm sốf(x) =

αx+β có tính chấtf(a), f(b), f(c) là độ dài cạnh tam giác ứng với tam giác ABC cho trước. Giải.

(131)

Nhận xét rằng, phép nghịch đảog(x) =

x khơng có tính chấtg(a), g(b), g(c) độ dài cạnh tam giác ứng với tam giácABC cho trước Thật vậy, xét tam giác cân ABCa=b= 2, c= 1thì ta có

g(a) =

a =

1

2, g(b) =

b =

1

2, g(c) =

c = Khi

g(a) +g(b) = +

1

2 = =g(c)

Vậy hàm số g(x) = x1 khơng có tính chất g(a), g(b), g(c)là độ dài cạnh tam giác ứng với tam giácABC cho trước

Đểf(a), f(b), f(c)là độ dài cạnh tam giác, trước hết ta phải có f(a)>0, f(b)>0, f(c)>0, ∀∆ABC.

Suy

1

αa+β >0,

1

αb+β >0,

1

αc+β >0, ∀∆ABC hay

αa+β >0, αb+β >0, αc+β >0, ∀∆ABC. (3.4) Từ (3.4) ta thu α > Thật vậy, α < 0, β tuỳ ý cho trước ta chọn tam giácABCađủ lớn, theo tính chất nhị thức bậc ta nhận αa+β <0

Tương tự, từ (3.4) suy raβ >0 Thật vậy, β <0 ta chọn tam giác ABCa đủ nhỏ, theo tính chất nhị thức bậc ta nhận αa+β <0

Trường hợp đồng thời xảy raα= 0, β= thìf(x) ≡0 khơng thoả mãn toán

Vớiα= 0, β >0 ta thu hàm dương f(x) = β1 nên

f(a) = f(b) = f(c) > f(a), f(b), f(c) độ dài cạnh tam giác

Xét trường hợpα >0, β >0, vớia>b>cta có: αa+β >αb+β>αc+β >0

Suy

1

αa+β

1

αb+β

1

αc+β hay

(132)

Vậy ta cần xác định sốα > 0, β >0 cho ln có f(a) +f(b) > f(c)

ứng với tam giácABC thoả mãn a>b>c hay

1

αa+β +

1

αb+β >

1

αc+β, ∀∆ABC:a>b>c (3.5) Xét tam giác cân ABC đồng dạng với tam giác cân cạnh 3,3,1 tức a =

b= 3d, c=dvớid >0tuỳ ý Khi (3.5) có dạng

1 3+β +

1 3+β >

1

+β, ∀d >0, tương đương với

2 3+β >

1

+β,∀d >0 hay

2+ 2β >3+β, ∀d >0, tức làβ > dα, ∀d >0.Điều không xảy khidđủ lớn

Vậy vớiα= 0, β >0thì hàm số

f(x) =

αx+β

có tính chấtf(a), f(b), f(c)là độ dài cạnh tam giác ứng với tam giác ABC cho trước

Bài toán 3.11. Xác định hàm số f(x) liên tục đoạn [0;π], f(0) =

và có đạo hàm khoảng (0;π)sao cho f(A), f(B), f(C) tạo thành độ đo các góc tam giác ứng với tam giác ABC cho trước.

Giải Ta cần xác định hàm khả vif(x) cho

(

f(x)>0, ∀x∈(0;π), f(0) =

f(A) +f(B) +f(C) =π

Theo giả thiết thìf(0) = 0nên f(π) =π C=π−(A+B) =πAB.Suy

f(A) +f(B) +f(πAB) =π, ∀A, B, A+B ∈[0;π], hay

f(x) +f(y) +f(πxy) =π, ∀x, y, x+y∈[0;π]. Lấy đạo hàm khoảng(0;π)theo biến x,ta thu

(133)

hay

f0(x) =f0(πxy), ∀x, y, x+y∈[0;π]. Do đóf0(x)là hàm khoảng (0;π) Khi f(x) =px+q.

Dof(0) = 0nên q= 0và vậyf(x) =px Dof(π) = πnên p= 1và ta thu f(x) =x.

Vậy hàm sốf(x) =x hàm số liên tục đoạn[0;π], f(0) = 0và có đạo hàm khoảng (0;π) cho f(A), f(B), f(C) tạo thành độ đo góc tam giác ứng với tam giácABC cho trước

Bài toán 3.12. Xác định hàm số f(x) liên tục đoạn [0;π] f(0) = 0, f(x)> 0, ∀x ∈ (0;π) f(A), f(B), f(C) tạo thành độ đo góc một tam giác ứng với tam giác ABC cho trước.

Giải Ta phát biểu lại toán cho dạng: Xác định hàm số f(x) liên tục đoạn [0;π]

(

f(0) = 0, f(x)>0, ∀x∈(0;π)

f(x) +f(y) +f(πxy) =π, ∀x, y∈(0;π), x+y < π (3.6) Dof(0) = nên vớiy= 0, ta thu

f(x) +f(0) +f(πx) =π, ∀x∈[0;π], hay

f(x) +f(πx) = π, ∀x∈[0;π]. (3.7) Đặt f(x) =g(x) +x g(0) = g(x) hàm liên tục đoạn [0;π] Khi (3.7) trở thành

x+g(x) +πx+g(πx) =π, ∀x∈[0;π]

hay

g(x) +g(πx) = 0, ∀x∈[0;π]. Do

g(πx) =−g(x), ∀x∈[0;π]. (3.8) Thế f(x) = x+g(x) vào (3.6) sử dụng (3.8), ta thu

x+g(x) +y+g(y) +πxy+g(πxy) =π, ∀x, y∈[0;π], x+y 6π, hay

(134)

tức

g(x) +g(y)−g(x+y) = 0, ∀x, y∈[0;π], x+y6π. Suy

g(x) +g(y) =g(x+y), ∀x, y∈[0;π], x+y6π. (3.9) Dog(x) hàm liên tục đoạn[0;π]nên (3.9) phương trình hàm Cauchy Khi đó, từ (3.9) suy g(x) =αxf(x) = x+αx= (1 +α)x.

Đểf(x)>0, ∀x∈(0;π)ta cần có(1 +α)x >0, ∀x∈(0;π)hay1 +α >0 Đểf(A) +f(B) +f(C) =π

(1 +α)A+ (1 +α)B+ (1 +α)C=π hay(1 +α)(A+B+C) =π,tức tức α= 0.

Vậyf(x) = hàm số thoả mãn đề

Bài toán 3.13. Xác định hàm số f(x) liên tục đoạn [0;π] sao cho f(A), f(B), f(C)luôn tạo thành độ đo góc tam giác ứng với mọi tam giác ABC cho trước.

Giải Ta thấy có hai hàm số hiển nhiên thoả mãn tốn, làf(x) = x f(x) = π3

Ta phát biểu toán dạng sau:

Xác định hàm sốf(x) liên tục đoạn [0;π]

f(x)>0, f(x) +f(y) +f(πxy) =π, ∀x, y∈(0;π), x+y < π (3.10) Choy→0 ta thu đượcf(x) +f(0) +f(πx) =π, ∀x ∈(0;π)

hayf(πx) = πf(0)−f(x), ∀x∈(0;π)

Thế vào (3.10) ta thu

f(x) +f(y) + [πf(0)−f(x+y)] =π, ∀x, y∈(0;π), x+y6π hay

f(x) +f(y) =f(x+y) +f(0), ∀x, y∈[0;π], x+y 6π (3.11) Đặt f(x) =f(0) +g(x) Khi g(x) liên tục đoạn [0;π] (3.11) có dạng

f(0) +g(x) +f(0) +g(y) =f(0) +g(x+y) +f(0), ∀x, y∈[0;π], x+y6πg(x) +g(y) =g(x+y), ∀x, y∈[0;π], x+y 6π (3.12) Do g(x) liên tục đoạn [0;π] nên (3.12) phương trình hàm Cauchy Khi đó, từ (3.12) suy rag(x) =αxf(x) =f(0) +αx.

(135)

Ta cần xác định α, β để f(x)> 0, ∀x ∈(0;π)và f(A) +f(B) +f(C) =π

hay (

αx+β > 0,∀x∈(0;π)

αA+β+αB+β+αC+β=π

(

αx+β >0,∀x∈(0;π)

α(A+B+C) + 3β =π

(

αx+β >0,∀x∈(0;π)

απ+ 3β =π

(

αx+β >0,∀x∈(0;π)

β = (1−α)π3

Do

f(x) =αx+(1−α)π

3 , ∀x∈(0;π). (3.13)

Cho x→0, từ (3.13), suy

(1−α)π

3 >0⇔α61.

Cho xπ, từ (3.13), suy ra

απ+ (1−α)π >0

hayα>−1

2.

Vậy−1

2 6α 61.

Với−12 < α <1 (3.13) hiển nhiên thoả mãn Xétα=−1

2 f(x) =− 2x+

π

2 thoả mãn điều kiện

Thật vậy, với0< α < π f(x)> f(π) = 0.Suy raf(x)>0, ∀x∈(0;π). Xétα= 1thìf(x) =x hiển nhiên thoả mãn điều kiện

Vậy hàm số cần tìm có dạng f(x) =αx+(1−α)π

3 ,

1

2 6α61.

Đến ta có lời giải cho Bài toán I lớp hàm liên tục

(136)

Giải Xét đường trịnOđường kính1 Ký hiệu M(4) tập hợp tất tam giác nội tiếp đường trịnOđó Khi đó, điều kiện cần đủ để ba số dương α, β, γ ba góc tam giác thuộcM(4) làsinα,sinβ,sinγ tạo thành độ đo cạnh tam giác thuộcM(4)

Thật vậy, khiα, β, γ ba góc tam giác thuộcM(4),thì theo định lý hàm sốsin,ta có2Rsinα,2Rsinβ,2Rsinγ độ dài ba cạnh tam giác tương ứng với ba gócα, β, γ

Do2R= 1nên sinα,sinβ,sinγlà độ dài cạnh tam giác nội tiếp đường trịnO đường kính1

Ngược lại, sinα,sinβ,sinγ độ dài cạnh tương ứng tam giác nội tiếp đường trịn đường kính doα, β, γ góc dương nên α, β, γ ba góc tam giác Vậy theo kết tốn 3.13, hàm cần tìm có dạng

f(x) = arcsin(αx+ (1−α)π )với−

1

2 6α61.

Bài toán 3.15.Xác định hàm sốf(x)liên tục trongR+sao chof(a), f(b), f(c)

luôn tạo thành độ dài cạnh tam giác ứng với tam giác ABC cho trước.

Giải Theo kết toán 3.14, thìf(a), f(b), f(c)xác định f(x) = arcsin(αx+ (1−α)π

3 )với−

2 6α61

sẽ tạo thành độ đo cạnh tam giác nội tiếp đường trịn đường kính ứng với tam giácABC nội tiếp đường trịn đường kính cho trước Vậy

f(x) =uarcsin(αx+(1−α)π )với−

1

2 6α61

ứng vớiu >0tuỳ ý, hàm số thoả mãn đề

Tiếp theo, ta quan tâm đến hàm số thiết lập nên dãy tam giác ứng với giá trị tương ứng đối số

Trước hết, ta nhận thấy nghiệm phương trình vơ địnhx2+y2=z2 tập số thực dương mơ tả dạng tham số

    

x=ucosv y=usinv z=u với u∈R+, v∈(0;π

(137)

Bài toán 3.16. Chứng minh với mọi(u, v) trong đó u∈R+, v∈(0;π2)đều tồn tam giác mà số đo cạnh số:

P1(u, v) =ucosv P2(u, v) =usinv

P3(u, v) =u

và tam giác ứng với mọi (u, v) trong đó u∈R+, v∈(0;π

2) cho trước đều là tam giác vuông.

Giải Dễ thấy rằng

P1(u, v)>0, P2(u, v)>0, P3(u, v)>0, ∀u∈R+, v∈(0; π

2)

và ta có đẳng thức sau

[P1(u, v)]2+ [P2(u, v)]2= [P3(u, v)]2.

Từ suy P1(u, v), P2(u, v), P3(u, v) độ dài cạnh tam giác vng có độ dài cạnh huyền P3(u, v)

Tiếp theo, ta xét hàm số biến lớp đa thức tạo thành độ dài cạnh tam giác ứng với đối số miền cho trước Bài toán 3.17. Chứng minh với mọi x >1 đều tồn tam giác mà số đo cạnh số

P1 =x4+x3+ 2x2+x+

P2= 2x3+x2+ 2x+

P3=x4−1

và tam giác ứng với mọi x >1 cho trước có góc lớn nhau. Giải Ta có

P1(x) = (x2+ 1)(x2+x+ 1), P2(x) = (x2+ 1)(2x+ 1), P3(x) = (x2+ 1)(x2−1). Đặt x2+x+ = a, 2x + = b, x2 −1 = c. Với x > hiển nhiên a >0, b >0, c >0.

Khi đó|b−c|=|x2−2x−2|vàb+c=x2+ 2x. Dễ thấy|b−c|< a < b+c, tức là

(138)

Do đóa, b, clà độ dài ba cạnh tam giác Suy

P1(x) =a(x2+ 1), P2(x) =b(x2+ 1), P3 =c(x2+ 1)

cũng độ dài cạnh tam giác Cạnh có độ dài lớn tam giác ứng với P1(x) Gọiα góc lớn tam giác vừa nhận Khi đó, theo định lý hàm sốcos,ta có

[P1(x)]2= [P2(x)]2+ [P3(x)]2−2P2(x)P3(x) cosα

a2(x2+ 1)2=b2(x2+ 1)2+c2(x2+ 1)2−2bc(x2+ 1)2cosαa2 =b2+c2−2bccosα

⇔ cosα= b

2+c2−a2

2bc ⇔ cosα= (2x+ 1)

2+ (x2−1)2−(x2+x+ 1)2

2(2x+ 1)(x2−1) ⇔ cosα= −2x

3−x2+ 2x+ 1

2(2x+ 1)(x2−1) ⇔ cosα= −(2x+ 1)(x

2−1)

2(2x+ 1)(x2−1), tức làcosα= −1

2 hayα= 2π

3 .

Vậy với mọix >1 thìP1(x), P2(x), P3(x)là số đo cạnh tam giác ứng với mọix >1cho trước Các tam giác nhận có góc lớn

3.3 Biểu diễn đa thức theo hệ nghiệm nguyên hàm

Tiếp theo, ta xét số biểu diễn đa thức thông qua hàm đối xứng Vieete Ta nhắc lại kết sau định lý đảo dấu tam thức bậc hai Định lý 3.1. Xét tam thức bậc hai

f(x) =ax2+bx+c, a6= 0.

Khi đó, điều kiện cần đủ để tồn số α sao cho af(α) < > x1 < α < x2, đó x1,2 là nghiệm của f(x).

(139)

qua bất đẳng thức (kết so sánh biệt thức∆với 0) Định lý thuận định lý đảo dấu tam thức bậc hai cho ta phát biểu kết biểu diễn tam thức bậc hai theo nghiệm chúng

Định lý 3.2 (Định lí Vieete). Điều kiện cần đủ để tam thức bậc hai f(x) =ax2+bx+c, a, b, cR, a6= 0,

có nghiệm thực tồn số x1, x2∈R sao cho

(

b

a =−x1−x2 c

a =x1x2

Tuy nhiên, ta phát biểu kết tương tự trường hợp ta chưa tường minh nghiệmx1, x2 tam thức bậc hai Điều có ý nghĩa xét tốn tam thức bậc hai tổng qt Các định lí sau cho ta tiêu chuẩn nhận biết, thông qua biểu diễn hệ số, tam thức bậc hai f(x) = ax2+bx+c, a6= 0, có nghiệm Khơng tính tổng quát, ta xét tam thức bậc hai dạng

f(x) = 3x2+ 2bx+c. (3.14)

Ta chứng minh số kết quan quan trọng khác liên quan đến tam thức bậc hai f(x)dạng (3.14)

Định lý 3.3. Với tam thức bậc hai f(x) có nghiệm thực tồn một nguyên hàm F(x), đa thức bậc ba, có ba nghiệm thực.

Chứng minh. Khi f(x) có nghiệm kép, tức f(x) = a(xx0)2, ta cần chọn nguyên hàm dạng

F(x) = a

3(xx0)

3 . Khi f(x)có hai nghiệm phân biệt, tức

f(x) =a(xx1)(xx2), x1 < x2, a6= 0, ta chọn nguyên hàm F(x) thoả mãn điều kiện

F

x

1+x2

2

= 0.

Khi đó, rõ ràng hàm F(x) có cực đại cực tiểu tạix1 x2 điểm uốn đồ thị tương ứng làM

x1+x2

2 ,0

Từ suy điều cần chứng minh

(140)

Định lý 3.4. Tam thức bậc hai f(x) = 3x2+ 2bx+c có nghiệm (thực) và chỉ hệ số b, ccó dạng

(

b=α+β+γ

c=αβ+βγ+γα (3.15)

Chứng minh. Điều kiện đủlà hiển nhiên theo bất đẳng thức Cauchy, ta có

∆0=b2−3c= (α+β+γ)2−3(αβ+βγ+γα) =α2+β2+γ2−(αβ+βγ+γα)

=1

2(αβ)

2

+1

2(βγ)

2

+1

2(γα)

2

>0.

Điều kiện cần Giả sử phương trình bậc hai có nghiệm thực x1, x2 Khi đó, tồn đa thức bậc ba có ba nghiệm thực, nguyên hàm f(x), tức

F(x) = (x+α)(x+β)(x+γ).

Từ ta suy điều cần chứng minh

Tiếp theo ta tìm tiêu chuẩn để nhận biết, thông qua biểu diễn hệ số, đa thức bậc ba tổng quát

f(x) =ax3+bx2+cx+d, a, b, c, d∈R, a6=

có nghiệm thực

Khơng tính tổng quát, ta coi đa thức bậc ba tổng quát có dạng f(x) =−4x3+ 3ax2−2bx+c, a, b, c∈R.

Định lý 3.5. Đa thức bậc ba f(x) = −4x3+ 3ax2−2bx+c với hệ số thực có các nghiệm thực hệ số a, b, ccó dạng

    

a=α+β+γ+δ

b=αβ+αγ+αδ+βγ+βδ+γδ c=αβγ+αβδ+αγδ+βγδ

(3.16)

Chứng minh.

Điều kiện đủ. Xét đa thức F(x) = −x4 +ax3−bx2+cxm với m số tùy ý Thay a, b, ctừ công thức (3.16) vào biểu thức củaF(x),ta thu

F(x) =x4−(α+β+γ+δ)x3

(141)

Chọn m=αβγδ.Khi đó, ta có

F(x) =x4−(α+β+γ+δ)x3+ (αβ+αγ+αδ+βγ+βδ+γδ)x2 −(αβγ+αβδ+αγδ+βγδ)x+αβγδ

= (xα)(xβ)(xγ)(xδ).

Suy raF(x)có bốn nghiệm thực làα, β, γ, δ Theo định lý Rolle đạo hàm của hàmf(x) =F0(x) có ba nghiệm thực

Điều kiện cần.Giả sử phương trình bậc ba có ba nghiệm thực x1, x2, x3 Ta tồn đa thức bậc bốn có bốn nghiệm thực nguyên hàm f(x), tức

F(x) =−(xα)(xβ)(xγ)(xδ). Thật vậy, ta xét ba trường hợp sau

i) Nếuf(x)có nghiệm bội bậc ba làx0 thìf(x)có dạngf(x) =−4(xx0)3. Chọn F(x) =−(xx0)4 ta có α=β =γ=δ =x0.

ii) Nếuf(x)có hai nghiệm phân biệt phải có nghiệm nghiệm kép Giả sử nghiệmx0, nghiệm cịn lại làx1.Khơng giảm tính tổng quát, ta giả sử x0= 0.Khi f(x) có dạng

f(x) =−4x2(xx1) = 4x3−4x1x2. Suy

F(x) =−x4+4 3x1x

3+m (m là số). Chọn m = 0thì đa thức F(x) = x3(x−4

3x1) có hai nghiệm phân biệt, đó,x= 0là nghiệm bội ba, nghiệm cịn lại x= 43x1 Khi đó, ta thu

α=β=γ= 0, δ = 3x1.

Nhận xét trường hợp tổng quát, nếuf(x)có nghiệm kép (bội bậc hai) làx0, nghiệm cịn lại làx1 ta thu đượcα=β=γ=x0, δ=x0+43x1.

iii) Nếu f(x) có ba nghiệm phân biệt, khơng giảm tổng qt, ta coi f(x) có dạng

f(x) =−4(x+a)x(xb), a >0, b >0, hay

f(x) =−4x3−4(ab)x2+ 4abx, a >0, b >0. Khi

F(x) =−x4−4

3(ab)x

3

(142)

Chọn m= 0thì

F(x) =−x4−4

3(ab)x

3+ 2abx2

=−x2

h

x2+4

3(ab)x+ 2ab

i

. Suy F(x) =

x2[x2+

3(ab)x−2ab] = 0.

Nếu xảy trường hợpx2= 0thì x1 =x2 = 0. Nếu x2+43(ab)x−2ab= 0thì

∆0=a2+ab+b2 >0.

Vậy phương trình bậc hai cho có hai nghiệm thực phân biệt làx3, x4F(x)

sẽ có bốn nghiệm

x1,2= 0(α=β = 0), x3=γ, x4=δ.

Nhận xét đa thức có nghiệmx=x0 bội k (k >1)thì đạo hàm có nghiệm x =x0 bội bậc k−1 Kết hợp với định lý Rolle, ta thu kết quan trọng sau

Định lý 3.6. Nếu đa thứcP(x) cók nghiệm thực đạo hàm đa thức P0(x) có nhất k−1 nghiệm thực.

Trong trường hợp đặc biệt, ta thu kết có nhiều ứng dụng đại số sau

Hệ 3.1. Nếu đa thức P(x) có nghiệm thực đạo hàm là đa thức P0(x) cũng có nghiệm thực.

Tuy nhiên, để ý toán ngược, tức toán xác định điều kiện cần để ứng với đa thức có nghiệm thực cho cho ta nguyên hàm có nghiệm thực tốn phức tạp khó Các trường hợp riêng biệt ứng với đa thức bậc thấp (bậc nhất, bậc hai, bậc ba) khảo sát chi tiết Tuy nhiên, từ hệ 3.1, ta có điều kiện đủ có dạng đơn giản

Giả sử choansố dương{a1, a2, , an} (n>1, n∈N) Khi f(x) = (x+a1)(x+a2) .(x+an)

(143)

trong E1(a) =

n

X

i=1

ai, E2(a) =

X

1 i<j n

aiaj, , En(a) =a1a2 an. (3.17) Đặt E0(a) = Ta gọi Er(a) (r ∈ {1, , n}) hàm (đa thức) đối xứng sơ cấp thứ r (Er(a) tổng tất tích r số khác số a={a1, a2, , an}) hay gọi hàm đối xứng Vieete bậc r.

Định lý 3.7. Đa thức

f(x) = (n+ 1)(−1)n+1xn+n(−1)na1xn+ (n−1)(−1)n−1a2xn−1+· · · −an với hệ số a1, a2, , an có dạng

          

a1 =E1(x) a2 =E2(x)

· · ·

an =En+1(x),

(3.18)

trong đóEk(x)là hàm đối xứng Vieete bậcktheo biến thựcx1, x2, xn+1, luôn ln có nghiệm thực.

Chứng minh. Xét đa thức

F(x) = (−1)n+1xn+1+ (−1)na1xn+· · · −anx+an+1, a1, a2, , anxác định theo (3.18)và

an+1 =x1x2· · ·xn+1. Khi đó, theo định lý Vieete, ta có

F(x) = (xx1)(xx2)· · ·(xxn+1).

Từ F0(x) = f(x), ta có điều phải chứng minh

Tiếp theo ta xét toán ngược cho trường hợp đa thức bậc có nghiệm thực dạng

(144)

Định lý 3.8. Giả sử đa thức

f(x) = 5(xx1)(xx2)(xx3)(xx4), x16x2 6x3 6x4 có nguyên hàm F0(x)là đa thức bậc với hệ số thực

F0(x) =x5+a1x4+a2x3+a3x2+a4x.

Khi đó, điều kiện cần đủ để tồn số thực c sao cho nguyên hàm F(x) =F0(x)−c

là đa thức có nghiệm thực là

F0(x1)>F0(x4). (3.19) Chứng minh. Ta xét trường hợp theo phân bố nghiệm

Khif(x) có nghiệm trùng thìf(x) = 5(xx1)4 hiển nhiên(3.19)

là thoả mãn Do F0(x) = (xx1)5 nên cần chọn nguyên hàm ứng vớic= Nhận xét rằngc= 0là giá trị thoả mãn điều kiện định lý

Khi f(x) có nghiệm trùng nhau, chẳng hạnx1 =x2 < x3< x4 (x1 < x2 < x3 =x4) xảy raF0(x1)< F0(x2) hiển nhiên điều kiện (3.19)là không thoả mãn không tồn c để đa thứcF(x)tương ứng có nghiệm thực Khi F0(x1)>F0(x4) ta chọnc=F0(x1),ta thu đa thứcF(x) tương ứng có nghiệm thực Nếu xảy rax1 < x2 =x3 < x4 chọn c=F0(x2), ta thu đa thức F(x) có nghiệm thực

Khi f(x) có hai cặp nghiệm trùng x1 = x2 < x3 = x4, f(x) = 5(xx1)2(xx3)2 vàF0(x)là hàm đồng biến đường thẳng qua điểm dừng cắt đồ thị đa thứcF0(x)tại điểm (bội bậc 3), tức làF(x)có nghiệm thực (kể bội) Do đó, ứng với c, nguyên hàm tương ứng F(x)đều có khơng q nghiệm thực

Khif(x) có ba nghiệm trùng nhau, chẳng hạnx1 =x2 =x3 < x4 (x1< x2 =

x3 =x4).Khơng giảm tính tổng qt, ta coi x1 = 0, tức f(x) = 5x3(xx4). Khi

F0(x) =x5−

5 4x

4 .

Do đóF0(x)có nghiệm thực Vậy ta cần chọnc= 0,thì đa thứcF(x)tương ứng có nghiệm thực

Cuối ta xét trường hợp nghiệm f(x) phân biệt, tức x1 < x2 < x3 < x4, nguyên hàm đạt cực đại x =x1 x = x3, đạt cực tiểu tạix=x2 vàx=x4 Nếu điều kiện (3.19)được thoả mãn ta có

(145)

Suy

min{F0(x1), F0(x3)}>max{F0(x2), F0(x4)}

Vậy ta cần chọncsao choc∈(max{F0(x2), F0(x4)},min{F0(x1), F0(x3)},thì nguyên hàm F(x) tương ứng có nghiệm thực

Tư tưởng chứng minh định lý 3.8 gợi cho ta cách chứng minh định lý sau Định lý 3.9. Giả sử đa thức bậc n(n>4) có nghiệm thực

f(x) = (n+ 1)(−1)n+1(xx1)(xx2)· · ·(xxn), x1 6x2 6· · ·6xn và có nguyên hàm F0(x) là đa thức bậc n+ với hệ số thực

F0(x) = (−1)n+1xn+1+a1xn+· · ·+anx, F0(0) = 0.

Khi đó, điều kiện cần đủ để tồn số thực c sao cho nguyên hàm Fc(x) =F0(x)−c

là đa thức có nghiệm thực điều kiện sau thoả mãn:

min{F0(x1), F0(x3), ,}>max{F0(x2), F0(x4), }

3.4 Dạng nội suy tính chất hàm lồi, lõm bậc cao Tiếp theo, ta xét lớp hàm lồi bậc cao số tính chất chúng Trước hết, ta nhắc lại tính chất đặc trưng định nghĩa hàm đồng biến hàm lồi quen biết thông qua biểu diễn dạng nội suy chúng Tính chất 3.3 (Dạng nội suy). Hàm số f(x) đồng biến trên I(a, b) khi chỉ khi với cặp số phân biệt x0, x1I(a, b), ta có

f(x1) x1−x2

+ f(x2)

x2−x1

>0. (3.20)

Chứng minh hiển nhiên

Tính chất 3.4 (Dạng nội suy). Hàm số f(x) lồi trênI(a, b) khi với mọi ba số phân biệt x0, x1, x2I(a, b), ta có

f(x0)

(x0x1)(x0x2) +

f(x1)

(x1x2)(x1x0) +

f(x2)

(x2x0)(x2x1) >0. (3.21)

(146)

Tính chất 3.5 (Dạng nội suy). Hàm số f(x) lồi trênI(a, b) khi với mọi cặp x1, x2 ∈I(a, b),x1< x2, ta có

f(x)6 x2−x

x2x1f(x1) +

xx1

x2x1f(x2), khix1 < x < x2.

Từ tính chất nêu này, gợi ý cho ta ngầm coi hàm đồng biến hàm lồi "bậc 0", hàm lồi gọi "hàm lồi bậc một" Với cách ngầm hiểu vậy, ta phát biểu định nghĩa hàm lồi bậc cao (bậc k, k∈N∗) tuỳ ý, sau

Định nghĩa 3.1 (Dạng nội suy). Hàm sốf(x) được gọi làn-lồi trên I(a, b)nếu với bộn+ số phân biệt trong I(a, b), ta có

f[x0, x1, , xn] :=

n

X

j=0 f(xj)

ω0(xj) >0, trong đó

ω(x) :=

n

Y

k=0

(xxk). Tương tự, ta có định nghĩa hàm lõm bậc cao

Định nghĩa 3.2 (Dạng nội suy). Hàm số f(x) được gọi là n-lõm trên I(a, b)

nếu với bộ n+ số phân biệt trong I(a, b), ta có f[x0, x1, , xn] :=

n

X

j=0 f(xj)

ω0(x j)

60,

trong đó

ω(x) :=

n

Y

k=0

(xxk).

Từ định nghĩa hàmn-lồi (lõm) trênI(a, b), ta dễ dàng chứng minh tính chất sau

Tính chất 3.6. Nếu hàm số f(x) n-lồi trên I(a, b) với n > 2, tồn tại f(k)(x) với mọi k, k n−2 và hàm số g(x) := f(k)(x) nk-lồi trên I(a, b).

Tính chất 3.7. Hàm sốf(x) có đạo hàm bậcn trên I(a, b) làn-lồi trênI(a, b)

khi khi

(147)

Cũng tương tự phép biểu diễn hàm lồi (lõm) thơng thường, ta có Định lý 3.10. Nếu hàm số f(x) n-lồi trên [a, b](n > 2) tồn hàm số g(x)và đa thức P(x) bậc không quá n−1, cho

f(x) =P(x) +

x

Z

a

(xt)n−1

(n−1)! g(t)dt. (3.22)

Chứng minhđược suy trực tiếp từ biểu diễn hàm lồi f(n−2)(x) có dạng (xem [5])

f(n−2)(x) =a0+

x

Z

a

g(t)dt,

và lấy tích phân n−2 lần liên tiếp, ta thu được(3.22), điều phải chứng minh Về sau, khơng có thích đặc biệt, nói hàm lồi (lõm) ta ngầm hiểu lớp hàm lồi (lõm) khả vi bậc hai Như vậy, hàm f(x) đơn điệu tăng trongI(a, b)khi

f0(x)>0, ∀x∈I(a, b)

và hàmf(x)lồi trênI(a, b)khi

f”(x)>0, ∀x∈I(a, b).

Từ đó, ta có nhận xét hàm f(x) lồi trênI(a, b) đạo hàm bậc hàm đơn điệu tăng

Do vậy, ta mơ tả công thức biểu diễn hàm lồi sau

Định lý 3.11. Hàmf(x) lồi trênI(a, b)khi tồn hàmg(x)đơn điệu tăng trong I(a, b) và số c∈(a, b), cho

f(x) =f(c) +

x

Z

c

g(t)dt.

Để ý nhiều cách tiếp cận khác lớp hàm lồi hàm lõm người ta tìm cách biểu diễn chúng theo mục tiêu cụ thể để giải toán thực tiễn

(148)

Để ý rằng, f(x) hàm lồi liên tục [a, b] với cặp số dương

(α, β)với α+β = 1xảy đẳng thức

αf(a) +βf(b) =f(αa+βb)

thì f(x) hàm số (đa thức) bậc

Vì vậy, hàm sốf(x) lồi khả vi I(a, b) đồ thị ln ln thuộc nửa mặt phẳng tạo nên tiếp tuyến điểm tuỳ ý cho trước đồ thị Nói cách khác, f(x) lồi I(a, b)thì với cặpx0, xI(a, b), ta có

f(x)>f(x0) +f0(x0)(xx0). (3.23) Thật vậy, (3.23)tương đương với

f0(x0)6 f(x)−f(x0)

xx0 , x > x0; x0, xI(a, b), (3.24)

f0(x0)> f(x)−f(x0)

xx0 , x < x0; x0, xI(a, b). (3.25) Các bất đẳng thức(3.24)và(3.25)là hiển nhiên (theo Định lí Lagrange)

Dễ nhận thấy rằng(3.23) xảy đẳng thức x0 = x Vậy ta viết

(3.23) dạng

f(x) = max

u∈I(a,b)

[f(u) +f0(u)(xu)]. Tương tự, ta có biểu diễn hàm lõm

Khi hàm sốf(x) lõm khả vi I(a, b) đồ thị thuộc nửa mặt phẳng tạo tiếp tuyến điểm tuỳ ý thuộc đồ thị, tức với cặp x0, xI(a, b), ta có

f(x)6f(x0) +f0(x0)(xx0). (3.26) Dễ nhận thấy rằng(3.26)xảy đẳng thức x0 =x Vậy ta viết(3.26)

dưới dạng

f(x) =

u∈I(a,b)

[f(u) +f0(u)(xu)]. (3.27) Như có dạng biểu diễn hàm lồi hàm lõm thông qua cực trị hàm số bậc phụ thuộc tham biến Phép biểu diễn gọi (theo Bellman) biểu diễn tuyến tính đóng vai trị quan trọng cơng cụ hữu hiệu nhiều tốn tính tốn cực trị tối ưu

(149)

Xét hàm số thực nhiều biến F(x1, x2, , xn) Giả sử, ứng với số

(z1, z2, , zn), z1 >z2>· · ·>zn, ta có

F(x1, x2, , xn) >F(z1, z2, , zn) + n

X

i=1

(xizi) ∂F

∂zi(z1, z2, , zn). Khi đó, hiển nhiên

F(x1, x2, , xn) = max

(z1,z2, ,zn)

h

F(z1, z2, , zn) +

n

X

i=1

(xizi)∂F

∂zi(z1, z2, , zn)

i

. Sử dụng khai triển Taylor, ta dễ dàng chứng minh bất đẳng thức sau lớp hàm lồi bậc bốn Các kết luận lớp hàm lồi bậc chẵn tuỳ ý

Định lý 3.12. Cho hàm số f(x)có đạo hàm bậc bốn không âm trong (a, b),tức

f(4)(x)>0, ∀x∈(a, b). Khi đó, ta ln có bất đẳng thức sau

f(x)>f(x0)+f0(x0)(x−x0)+f

”(x0)(xx0)2

2! +

f000(x0)(xx0)3

3! , ∀x, x0∈(a, b).

Tương tự, hàm số f(x) có đạo hàm bậc bốn luôn không dương trong

(a, b),tức là

f(4)(x)60, ∀x∈(a, b), thì ta ln có bất đẳng thức sau

f(x)6f(x0)+f

0

(x0)(x−x0)+ f”(x

0)(xx0)2

2! +

f000(x0)(xx0)3

3! , ∀x, x0∈(a, b).

Hệ 3.2. Với đa thức bậc bốn

P(x) =x4+ax3+bx2+cx+d, ta có

P(x)>P(x0) + (4(x30+ 3ax

0+ 2bx0+c)(xx0)

+(12x

2

0+ 6ax0+ 2b)(xx0)2

2! +

(24x+ 6a)(x0)(xx0)3)

(150)

Tiếp theo ta nêu vài áp dụng trực tiếp chứng minh bất đẳng thức Bài toán 3.18. Chứng minh khi x >0 ta có

lnx6

2006

2006X n=1

h

lnn+

n(xn)

1 2!

1

n2(xn) 2+

3!

n3(xn) 3i.

Giải.Xét hàm số y=f(x) = lnx, ∀x >0 Khi f0(x) =

x, f

(x) =− x2;f

000

(x) =

x3; f

(x) =−6

x4 <0, ∀x >0. Suy

lnx6 lna+ +1

a(xa)−

1

a2

(xa)2

2! +

2

a3

(xa)3

3! , ∀x, a >0.

Lần lượt thayabởi 1,· · ·,2006,ta thu lnx ln1 + (x−1)−

2!(xa)

2−

3!(x−1)

3, lnx ln2 +

2(x−2)− 22

(x−2)2

2! +

2 23

(x−2)3 3! ,

lnx ln2006 +

2006(x−2006)− 20062

(x−2006)2

2! +

2 20063

(x−2006)3

3! .

Cộng vế theo vế bất đẳng thức trên, ta

2006lnx6

2006X n=1

lnn+

2006X n=1

1

n(xn)−

1 2! 2006 X n=1

n2(xn) 2+

3!

2006X n=1

1

n3(xn) 3. Suy

lnx6

2006

2006X n=1

h

lnn+

n(xn)

1 2!

1

n2(xn)

+ 3!

1

n3(xn) 3i

. Bài toán 3.19. Chứng minh với x∈(0, π), ta có

sinx+

π(xπ

2) + 23

π3

(xπ

2)

3

3 > ln 2x

π +

22

π2

(xπ

2)

2

(151)

Giải.Xét hàm số f(x) = sinx− ln2x

π Khi f0(x) = cosx

x, f00(x) = −sinx+

x2, f000(x) = −cosx

x3, f(4)(x) = sinx+

x4 >0, ∀x∈(0, π). Suy

sinx− ln2x

π >sina− ln

2a π +

cosa−1 a

(xa) +

−sina+

a2

(xa)2

2! +

−cosaa3

(xa)3

3! , ∀a∈(0, π).

Chọn a= π

2,ta thu sinx− ln2x

π >

sinπ − ln

2π

π

+cosπ

1

π

2

xπ

2

+−sinπ +

1

π

2

2

xπ

2

2

2! + (−cos

π − (π 2) 3)

(xπ

2)

3

3!

sinx− ln2x π >1−

2

π(xπ

2)−

(xπ

2)

2

2 +

22

π2

(xπ

2)

2

2 −

23

π3

(xπ

2)

3

3 .

1−

(xπ

2)

2

2 >0, ∀x∈(0, π)

nên

sinx+

π

xπ

2 +2 π3

xπ

2

3

3 > ln 2x

π +

22 π2

xπ

2

2

(152)

Bài toán 3.20. Cho ba sốu1, v1, w1 u2, v2, w2 thỏa mãn điều kiện

(

u1 > u2, v1 > v2

u1+v1+w1 =u2+v2+w2.

và cho hàm số f(x) có đạo hàm cấp hai dương với mọi x và đạo hàm cấp bốn dương với mọi x Chứng minh ta ln có bất đẳng thức sau

f(u1) +f(v1) +f(w1)>f(u2) +f(v2) +f(w2) +

1

2(u2−u1)(v2−v1)(w1−w2).

Giải.f(4)(x)>0, f(00)(x)>0 với mọix nên suy f(x)>f(a) +f0(a)(xa) +

3!f

000

(a)(xa)3, với mọia. Thay x bởiu1, v1, w1abởi u2, v2, vv2 ta

f(u1) > f(u2) +f0(u2)(u1u2) + 3!f

000

(u2)(u1u2)3, f(v1) > f(v2) +f0(v2)(v1v2) +

3!f

000

(v2)(v1v2)3, f(w1) > f(w2) +f

0

(w2)(w1−w2) +

1 3!f

000

(w2)(w1−w2)3. Suy

f(u1)+f(v1)+f(w1)>f(u2)+f(v2)+f(w2)+

h

f0(u2)−f

0

(w2)

ih

(u1−u2)+(v1−v2)

i

+

h

f0(u2)−f

0

(v2)

i

(u1−u2) +

1 3!

h

f000(v2)−f

000

(w2)

ih

(u1−u2)3+ (v1−v2)3

i

+1 3!

h

f000(u2)−f000(v2)

i

(u1u2)3+ 3!f

000

(w2)

h

(w1w2)3+ (u1−u2)3+ (v1v2)3

i

. Khơng tính tổng qt, ta giả sử

u1> v1 > w1, u2 > v2> w2.u1 > u2,v1 > v2 f

0

(x), f000(x) hàm đơn điệu tăng nên suy f(u1) +f(v1) +f(w1)>

>f(u2) +f(v2) +f(w2) +f000(w2)

h

(w1w2)3+ (u1u2)3+ (v1v2)3

i

>f(u2) +f(v2) +f(w2) +

3!f

000

(w2)

h

((u2−u1)−(v2−v1))3−(u2−u1)3−(v2−v1)3

i

>f(u2) +f(v2) +f(w2) +

1 2f

000

(153)

Vậy nên

f(u1) +f(v1) +f(w1)>f(u2) +f(v2) +f(w2) +

1

2(u2−u1)(v2−v1)(w1−w2).

Bài toán 3.21. Cho ba sốu1, v1, w1 u2, v2, w2 thỏa mãn điều kiện

(

u1> u2, v1> v2

u1+v1+w1=u2+v2+w2

và cho hàm số f(x) có đạo hàm cấp hai đạo hàm cấp bốn nhận giá trị âm với mọi x Chứng minh rằng

f(u1) +f(v1) +f(w1)6f(u2) +f(v2) +f(w2) +

1

2(u2−u1)(v2−v1)(w1−w2).

Cách chứng minhhoàn tồn tương tự tốn 3.20

Bài tốn 3.22. Cho (

x >3, y >2

x+y+z= 6. Chứng minh rằng

lnxyz

6

2

6(3−x)(2−y)(z−1). Giải.Xét hàm số f(x) = lnx Khi đó, ta có

f0(x) =

x, f

00

(x) = −1

x2 <0, f

000

(x) =

x3, f (4)

(x) =− x4 <0 Suy

lnx+ lny+ lnz ln3 + ln2 + ln1 +

2(3−x)(2−y)(z−1)

⇔ lnxyz

6

(3−x)(2−y)(z−1)

⇔ ln

xyz

6

2

6(3−x)(2−y)(z−1)

Bài toán 3.23. Giả sử hàm sốf(x)dương khả vi hai lần (bậc hai) trênI(a, b). Khi để lnf(x) là hàm lồi, điều kiện cần đủ là

(154)

Giải. Đặt g(x) = lnf(x) Khi điều kiện cần đủ để hàm g(x) lồi g00(x)>0 Tức là,

f0(x)

f(x)

0

>0 ⇔ f

00(x)f(x)−f02

(x)

f2(x) >0⇔f(x)f 00

(x)−[f0(x)]2>0. Bài toán 3.24. Chứng minh với a, b, x, y >0, thì

x lnx

a+y ln y

b >(x+y) ln x+y a+b. Giải. Xét hàm sốf(x) =x lnx.Ta có

y0= lnx+ 1, y00=

x >0, ∀x >0. Vậy hàm sốy =f(x) hàm lồi Khi đó, với x

a, y

b >0,ta có f

a a+b·

x a+

b a+b·

y b

6 a

a+b ·f

x

a

+ b

a+b·f

y

b

x+y a+b · ln

x+y a+b

x a+b ln

x a+

y a+b ln

y b ⇔ (x+y) lnx+y

a+b 6x ln x a +y ln

y b. Dấu đẳng thức xảy x

a = y b

Bài toán 3.25. Chứng minh ứng với bộn số dương (x) với trọng(α), ta có

dMh(x, α)

dh 60, ∀h∈R\ {0}, trong đó

Mh(x, α) = (α1x1k+· · ·+αnxhn)

1

h.

Dấu đẳng thức xảy khi x1 =x2 =· · ·=xn. Giải. Thật vậy, để ý

h2 Mh(x, α)

n

X

i=1 αixhi

dMh(x, α) dh =

=

n

X

i=1

αixhi lnxhi

Xn i=1

αixhi

ln n

X

i=1 αixhi

(155)

Mặt khác, dễ dàng kiểm tra tính lồi hàm sốg(x) :=x lnx trong(0,+∞) Ta thu

g(xi)>g(x) + (xix)g0(x), ∀x, xi >0. Từ đây, chọnx=

n

P

i=1

αixi, ta được n

X

i=1

αig(xi)>g

Xn i=1

αixi

hay

n

X

i=1

αixi lnxi>

Xn i=1

αixi

ln

Xn i=1

αixi

. (3.29)

Dấu đẳng thức trong(3.29)xảy sốxi nhận giá trị Từ(3.28)và(3.29),ta suy

dMh(x, α)

dh 60, ∀h∈R\ {0}, dấu đẳng thức xảy x1 =x2 =· · ·=xn.

Như vậy, sốxi không nhận giá trị thìMh(x, α)là hàm đồng biến theo biến h ∈R Đồ thị hàmy = Mh(x, α) có hai tiệm cận ngangy = min{xi; i= 1,2, , n}y= max{xi; i= 1,2, , n}.

Bài toán 3.26. Chứng minh ứng với bộn số dương (x) với trọng(α), ta có

f(h) =h lnMh(x, α) là hàm lõm trênR.

Giải.f(h) hàm khả vi, nên ta kiểm tra tính lồi thơng qua việc tính trực tiếp đạo hàm hàm số

f(h) = lng(h), g(h) :=

n

X

i=1 αixhi). Ta có

f0(h) =

g(h)

n

X

i=1

αixhi lnxi, f00(h) =

g(h)

Xn i=1

αixhi

Xn

i=1

αixhi( lnxi)2

n

X

i=1

αixhi lnxi

(156)

Theo bất đẳng thức Cauchy,

Xn i=1

αixhi

Xn

i=1

αixhi( lnxi)2

n

X

i=1

αixhi lnxi

2

60, nên ta cóf00(h)60

Từ đây, ta nhận kết qủa tốn tìm giá trị nhỏ liên quan đến dạng trung bình bậc h có trọng

Bài toán 3.27. Xét bộ n số dương (x) với trọng (α) và xét số (h) biến thiến với tổng

n

X

i=1

αihi =S, không đổi Chứng minh rằng

n

Y

i=1 ihi

hi (x, α)>M

S

S(x, α). (3.30)

Giải. Dễ kiểm tra hàm số f(h) := ln

Xn i=1

αixhi)

là hàm lõm khả vi bậc hai Do đó, theo bất đẳng thức Jensen, ta có n

X

i=1

αif(xi)>f

Xn i=1

αixi

. Từ đây, ta có ngay(3.30), điều phải chứng minh

Xét bộnsố dương tuỳ ý

(x) := (x1, x2, , xn).

Cũng tương tự trung bìnhMh(x, α)trong mục trước, ta xét tổng bậc h (x)

Định nghĩa 3.3. Xét số thực h6= 0 Khi tổng Sh(x) xác định theo cơng thức

Sh(x) =

Xn i=1

xhi

1

h

, (3.31)

(157)

Tính đơn điệu tổngSh(x) dạng(3.31)được phát biểu dạng sau Bài toán 3.28. Chứng minh ứng với bộ n số dương (x), tổng Sh(x) đồng biến khỏang (−∞,0) (0,+∞)

lim

h→−∞Sh(x) = min{xi; i= 1,2, , n},

lim

h→+∞Sh(x) = max{xi; i= 1,2, , n}.

Giải.Sh(x) khả vi khoảng (−∞,0) (0,+∞), nên ta kiểm tra tính đơn điệu cách tính đạo hàm xét dấu khoảng tương ứng Sử dụng đẳng thức

h lnSh(x) =

n

X

i=1 xhi

để tính đạo hàm hai vế theoh, ta dễ dàng suy kết luận toán.

Bài toán 3.29. Chứng minh với bộ n số dương (x), hàm số f(h) :=

h lnSh(x) là hàm lồi theo biến h.

Giải. Chứng minh suy từ tính chất lồi hàm sốF(h) :=h lnMh(x, α) Tương tự, dễ dàng kiểm tra tính lồi hàm sốg1(h) =Sh(x)vàg2(h) :=

lnSh(x) trong(0,+∞) Từ đây, ta thu

Nhận xét 3.1. Với bộ n số dương (x) và số dương (α), xét số

(h) =h1, h2, , hn sao cho tổng

α1h1+α2h2+· · ·+αnhn=M khơng đổi Khi đó

n

X

i=1

αiShi(x)6SM(x).

Nhận xét 3.2. Với bộ n số dương (x) và số dương (α), xét số

(h) =h1, h2, , hn sao cho tổng

α1h1+α2h2+· · ·+αnhn=M không đổi Khi đó

n

Y

i=1 i

(158)

Bài toán 3.30. Chứng minh với hàm số f(x) có đạo hàm liên tục tới cấp 2n+ (n ∈ N) f(2n+1)(x) 6= với mọi x ∈ (a, b), tồn đa thức P2n(x) bậc không quá 2n sao cho hàm số

h(x) :=f(x)−P2n(x) đơn điệu khoảng (a, b).

Giải. Cách chứng minh dựa vào khai triển Taylor tới cấp 2n+ hàm f(x)tại x0 ∈(a, b) Ta có

f(x) =f(x0)+f

0(x 0)

1! (x−x0)+· · ·+

f(2n)(x0)

(2n)! (x−x0)

2n

+f

(2n+1)(x 1)

(2n+ 1)! (x−x0)

2n+1 , ứng vớix1∈(a, b)

Từ đây, ta cần chọn P2n(x) =f(x0) +

f0(x0)

1! (xx0) +· · ·+

f(2n)(x0)

(2n)! (xx0)

2n

là đủ

Thật vậy, f(2n+1)(x) liên tục f(2n+1)(x) 6= với x ∈ (a, b), nên f(2n+1)(x) luôn dương luôn âm trong(a, b) Do hàm số

hn(x) := f

(2n+1)(x 1)

(2n+ 1)! (xx0)

2n+1 , tương ứng, đồng biến nghịch biến trong(a, b)

Tương tự, ta có kết luận sau

Bài toán 3.31. Chứng minh với hàm số f(x) có đạo hàm liên tục tới cấp 2n (n∈N+) vàf(2n)(x)6= 0với mọi x∈(a, b), tồn đa thức P2n−1(x)

bậc không quá 2n−1 sao cho hàm số

h(x) :=f(x)−P2n−1(x)

lồi lõm khoảng (a, b).

Giải. Cách chứng minh dựa vào công thức khai triển Taylor tới cấp2nđối với hàmf(x) tạix0∈(a, b)

f(x) =f(x0)+f

0

(x0)

1! (x−x0)+· · ·+

f(2n−1)(x0)

(2n−1)! (x−x0)

2n−1

+f

(2n)

(x1)

(2n)! (x−x0)

(159)

với x1 ∈(a, b)

Tiếp theo, ta cần chọn P2n−1(x) =f(x0) +

f0(x0)

1! (xx0) +· · ·+

f(2n−1)(x0)

(2n−1)! (xx0)

2n−1

là đủ Thật vậy, f(2n)(x) liên tục f(2n)(x) 6= với x ∈ (a, b), nên f(2n)(x) luôn dương ln âm trong(a, b) Do hàm số

hn(x) :=

f(2n)(x1)

(2n)! (xx0)

2n,

hn(x) =

f(2n)(x1)

(2n−2)!(xx0)

2n−2 ,

tương ứng, luôn>0 luôn60trong(a, b) Do đóh(x) hàm lồi lõm trong(a, b)

Bài tốn 3.32. Cho hàm sốf(x) liên tục khơng âm trên [a, b] Chứng minh rằng, với phép phân hoạch đoạn[a, b]bởi điểm thoả mãn điều kiện

x0, x1, , xn (a=x0 < x1< x2<· · ·< xn=b ta có

1) Nếuf(x) đồng biến trên[a, b] thì n

X

i=1

(xixi−1)f(xi−1)< b

Z

a

f(x)dx < n

X

i=1

(xixi−1)f(xi),

2) Nếu f00(x)>0 ∀x∈[a, b]thì b

Z

a

f(x)dx < n

X

i=1

(xixi−1)

f(xi) +f(xi−1)

2 .

Giải.Kí hiệu

Ci = (xi;f(xi+1));Bi= (xi+1;f(xi));Ai= (xi;f(xi));

A= (a,0);B= (b,0), S =

b

Z

a

(160)

GọiS1 diện tích đa giác

AC0A1C1 An−1Cn−1AnB, tổng diện tích hình chữ nhật với kích thước

f(xi), xixi−1, i= 1,2, , n. Khi đó, ta có

S1 =

n

X

i=1

(xixi−1)f(xi). GọiS2 diện tích đa giác

AA0B0A1 Bn−2An−1Bn−1B, tổng diện tích hình chữ nhật với kích thước

f(xi−1), xixi−1, i= 1,2, , n. Khi đó, ta có

S2 =

n

X

i=1

(xixi−1)f(xi−1).

Vậy nên, nếuf(x)đồng biến [a, b], thìS2 < S < S1,hay n

X

i=1

(xixi−1)f(xi−1)< b

Z

a

f(x)dx < n

X

i=1

(xixi−1)f(xi).

2) Gọi S3 diện tích đa giác AA0A1 AnB (là tổng diện tích hình thang có hai đáy làf(xi−1), f(xi)và đường caoxixi−1 với i = 1, 2, , n) Khi

S3 =

n

X

i=1

(xixi−1)f(xi) +f(xi−1)

2 .

f00(x)>0trong[a, b], nên ta có b

Z

a

f(x)dx < n

X

i=1

(xixi−1)

f(xi) +f(xi−1)

2 ·

Nếu f(x) nghịch biến trên[a, b], n

X

i=1

(xixi−1)f(xi−1)> b

Z

a

f(x)dx > n

X

i=1

(161)

Nếu có thêm f00(x)<0 đoạn [a, b], b

Z

a

f(x)dx > n

X

i=1

(xixi−1)

f(xi) +f(xi−1)

2 .

3.5 Biểu diễn số lớp hàm số

Trong phần trước, đề cập đến số tính chất liên quan đến dạng biểu diễn hàm lồi hàm lõm thông qua hàm số bậc (thường gọi tuyến tính) đa thức bậc thấp lớp hàm đơn giản dễ tính tốn ước lượng tập xác định tập giá trị chúng

Trong mục này, ta đặc biệt quan tâm đến dạng biểu diễn khác số lớp hàm số bất đẳng thức quen biết

Chẳng hạn, với hàm số gấp khúch(x) =|x|, ta có biểu diễn tuyến tính dạng sau:

|x|= max

−1 u 1(ux). (3.32)

Thật vậy, theo định nghĩa hàm dấu sign,

signx=

    

1, x >0, − 1, x <0,

0, x= 0. Vậy nên

|x|= (signx)x6ux, −16u61, dấu đẳng thức xảy u=signx.Từ ta có ngay(3.32)

Tư tưởng biểu diễn hàm số mô tả sau

Giả sử cho đa thức với hệ số thựcf(x, y), biếnx, y∈Rvà giả thiết f(ax1+bx2, y) =af(x1, y) +bf(x2, y), ∀x1, x2, y∈R.

Cho tập S⊂Rvà kí hiệu

g(x) = max

y∈S f(x, y). Khi

(162)

Thật vậy, ta có

g(x1+x2) = max

y∈S f(x1+x2, y)

= max

y∈S[f(x1, y) +f(x2, y)]

6max

y∈S f(x1, y) + maxy∈S f(x2, y)

=g(x1) +g(x2), điều phải chứng minh

Bài toán 3.33. Chứng minh nếuP(x) là đa thức với hệ số nguyên với mỗixnguyên thì P(x)nhận giá trị bình phương số nguyên thìP(x)

là bình phương đa thức với hệ số nguyên.

Giải Giả sử P(x) bình phương đa thức hệ số nguyên Khi tồn số nguyên avà hai đa thứcQ(x), R(x)với hệ số nguyên cho

a2P(x) = [Q(x)]2R(x),

trong Q(x) 1, cịn R(x) có khơng điểm bội nhỏ thua Theo Bài tốn 24 đa thức R(x) tồn số nguyên nđủ lớn để R(n)

chia hết cho số nguyên tố p không chia hết cho p2 Do R(n) P(n) khơng luỹ thừa bậc hai Điều mâu thuẫn với giả thiếtP(n) ln ln bình phương số ngun Từ ta có điều phải chứng minh

Tiếp theo, ta xét biểu diễn số lớp đa thức dương tập Ta xét số biểu diễn đa thức dạng tổng, hiệu, tích, đa thức có dạng đặc biệt cho trước

Bài toán 3.34. Chứng minh tam thức bậc hai thoả mãn điều kiện f(x) :=ax2+bx+c >0, ∀x∈R

đều biểu diễn dạng tổng bình phương hai nhị thức bậc nhất. Giải Từ giả thiết suy raa >0,∆<0 Ta viết tam thức bậc hai dạng

f(x) =

ax+ b 2√a

2

+

r−∆

4a

2 . Tiếp theo sử dụng đồng thức Lagrange

u+v

2

2

+

uv

2

2

(163)

Bài toán 3.35. Chứng minh tam thức trùng phương thoả mãn điều kiện

f(x) :=ax4+bx2+c >0, ∀x∈R

đều biểu diễn dạng tổng bình phương hai tam thức bậc hai. Giải Xét điều kiện để tam thức bậc hai

g(t) :=at2+bt+c >0, ∀x>0. Từ giả thiết suy a >0 vàc >0

Nếu ∆<0 ta thu tốn??và có điều cần chứng minh Nếu ∆>0 ta viết f(x) dạng tích hai tam thức bậc hai

f(x) =f1(x)f2(x), f1(x)>0, f2(x)>0, ∀x∈R.

Theo toán??, tam thức bậc haif1(x)và f2(x) biểu diễn dạng tổng bình phương hai nhị thức bậc Tiếp theo, ta cần dùng đồng thức Lagrange

(u2+v2)(p2+q2) = (up+vq)2+ (uqvp)2, ta có điều cần chứng minh

Bài toán 3.36. Cho f(x) = (x+ 1)(x2+ax+b) nhận giá trị dương với mọi x>0 Chứng minh rằng

f(x) = [u(x)]2+ [v(x)]2+x{[p(x)]2+ [q(x)]2}, trong đó u(x), v(x), p(x), q(x)là nhị thức bậc nhất.

Giải. Trước hết ta chứng minh tam thức bậc hai g(x) =x2+ax+b >0,∀x tổng bình phương hai nhị thức bậc

Thật vậy, viết hàm sốg(x) dạng g(x) =x2+ax+b=

x+b

2

+ −∆ ,

với ∆<0 Tiếp theo, sử dụng đẳng thức

2(α2+β2) = (α+β)2+ (αβ)2

hay

α2+β2=

α+β

2

2

+

αβ

2

(164)

Từ ta viết tam thức bậc haig(x) =x2+ax+b >0với mọix tổng bình phương hai nhị thức bậc

Vậy, g(x)>0với mọix g(x) tổng bình phương hai nhị thức g(x) = [u(x)]2+ [v(x)]2

nên

f(x) = [u(x)]2+ [v(x)]2+x{[u(x)]2+ [v(x)]2} Trong trường hợp ta thu điều phải chứng minh

Nếu g(x)có nghiệm hai nghiệm âm, nên b >0 vàc >0 Khi ta viết

f(x) =x(x2+b) +ax2+ (x2+b) +ax= [(a+ 1)x2+b] +x(x2+a+b), tam thức bậc hai(a+ 1)x2+b > 0vàx2+a+b >0với mọix nên chúng tổng bình phương hai nhị thức

Bài toán 3.37. Cho đa thức P(x) ∈ R[x] P(x) > với mọi x ∈ R Chứng minh đa thức P(x) có thể biểu diễn dạng

P(x) = [A(x)]2+ [B(x)]2, trong đó A(x), B(x) cũng đa thức.

Giải DoP(x) >0 với x ∈ R nên đa thức P(x) có bậc 2n phân tích dạng tích nhân tử bậc hai khơng âm, nghĩa

P(x) =

n

Y

j=1

[(ajx+xj)2+yj2],

trong aj, xj, yj∈R,j= 1,2, , n. Từ đẳng thức

(p21+q21)(p22+q22) = (p1p2+q1q2)2+ (p1q2p2q1)2,

ta có kết luận: Tích hai biểu thức dạng [u(x)]2+ [v(x)]2 biểu thức có dạng Sau hữu hạn bước thực quy trình đó, ta thu biểu thức dạng

P(x) = [A(x)]2+ [B(x)]2.

Bài toán 3.38. Cho f(x) = ax2+bx+c∈ R[x] thỏa mãn điều kiện f(x) >0

với mọi x >0 Chứng minh tồn đa thức P(x) sao cho đa thức Q(x) =

(165)

Giải. Dof(x)>0 với mọix>0 nên a >0 vàc=f(0)>0

Nếub>0 ta chọnP(x) = 1và ta nhận điều phải chứng minh Nếu b <0thìa >0 Ta tìmP(x) dạngP(x) = (x+ 1)n vớin>2 Ta có

P(x) = (x+ 1)n=

n

X

k=0 Cnkxk nên

f(x)P(x) = (ax2+bx+c)(x+ 1)n= =axn+2+ (b+na)xn+1+· · ·+

n

X

k=0

(aCnk−2+bCnk−1+cCnk)xk

+· · ·+ (b+nc)x+c. Ta chọn nsao cho

    

b+na>0

b+nc>0

aCnk−2+bCnk−1+cCnk >0 ∀k>2.

(3.33)

Nhận thấy với n > max{−b/a,−b/c} a > nên hai điều kiện đầu của(3.33)được thoả mãn

Ta biến đổi vế trái điều kiện thứ ba của(3.33) Ta có

h(k) = (ab+c)k2−[a−(n+ 2)b+ (2n+ 3)c]k+c(n+ 1)(n+ 2)>0. Dob <0,a>0,c>0 nên ab+c >0

Để điều kiện thứ ba của(3.33) với k ta chọn n cho biệt thức tam thức bậc haih(k)không dương (∆h 60) Biểu thức của∆h tam thức bậc hai theo ncó hệ số n2

(b−2c)2−4c(ab+c) =b2−4ac60

(dof(x)>0 ∀x>0 ;>0, a >0 Do vậy, ta có

lim

n→∞h =−∞

Do vớin đủ lớn thì∆h60

(166)

Bài tốn 3.39. Cho đa thức

g(x) =anxn+an−1xn−1+· · ·+a1x+a0 (n>3)

thỏa mãn điều kiện g(x) > với mọi x > 0 Chứng minh tồn tại s∈Nđể đa thức Q(x) dạng Q(x) =g(x)(x+ 1)s có hệ số không âm. Giải. Ta xét hai trường hợpdegg(x) = 2m vàdegg(x) = 2m+ với m∈N

Khin= 2m ta phân tích g(x) =

m

Y

k=1

(akx2+bkx+ck),

akx2+bkx+ck >0 ∀x >0.

Theo Bài toán , với đa thức akx2+bkx+ck tồn số tự nhiên mk cho đa thức

Qk(x) = (akx2+bkx+ck)(x+ 1)mk có hệ số khơng âm

Từ suy m

Y

k=1

Qk(x) =

m

Y

k=1

g(x)(x+ 1)mk =g(x)(x+ 1)m1+···+mm

là đa thức có hệ số không âm

Khi deg g(x) = 2m+ g(x) có nghiệm khơng dương −a (a>0) Ta có

g(x) = (x+a)h(x) với h(x)>0 với mọix >0và deg h(x) = 2m. Do deg h(x) = 2m nên theo trường hợp thứ tồn số nguyên dương s cho h(x)(x+ 1)s có hệ số khơng âm đa thứcg(x)(x+ 1)s

cũng có hệ số khơng âm

Bài tốn 3.40. Hỏi có tồn hay không tồn đa thức P(x), Q(x), T(x)

với hệ số nguyên dương thoả mãn hệ thức T(x) = (x2−3x+ 3)P(x), P(x) =

x2

20−

x

15+ 12

(167)

Giải. Viết lại đẳng thức đề dạng

60T(x) = 60(x2−3x+ 3)P(x) = (3x2−4x+ 5)Q(x). (3.34) Các đa thức(x2−3x+ 3)và(3x2−4x+ 5)vô nghiệm nguyên tố Vì từ (3.34) ta suy tồn đa thức P(x), Q(x), T(x)thoả mãn điều kiện dề tồn đa thức S(x)sao cho đa thức

(3x2−4x+ 5)S(x), 60(x2−3x+ 3)S(x)

(3x2−4x+ 5)(x2−3x+ 3)S(x)

đều đa thức với hệ số nguyên dương

Theo kết Bài tốn tồn số nguyên dương k1 đủ lớn cho

(3x2−4x+ 5)(x+ 1)k1

là đa thức có hệ số ngun khơng âm từ dễ dàng suy khơng có hệ số đa thức tồn số nguyên dương k2 đủ lớn để

(x2−3x+ 3)(x+ 1)k2

là đa thức có hệ số nguyên dương Từ suy

(3x2−4x+ 5)(x2−3x+ 3)(x+ 1)k1+k2

cũng đa thức với hệ số nguyên dương Như câu trả lời toán khẳng định Chẳng hạn ta chọn

P(x) = (3x2−4x+ 5)(x+ 1)k1+k2,

Q(x) = 60(x2−3x+ 3)(x+ 1)k1+k2,

T(x) = (3x2−4x+ 5)(x2−3x+ 3)(x+ 1)k1+k2.

(Bằng cách thử trực tiếp thấy k1 >3, k2 >15)

Bài toán 3.41. Chứng minh đa thức P(x) >0 với mọi x >0 thì tồn tại đa thức A(x), B(x), C(x), D(x)để P(x) biểu diễn dạng

(168)

Giải.

1 Trường hợp degP(x) = 2m.

Nếu m= 0thì ta dễ dàng viết biểu diễn (3.35) Vớim>1, ta phân tích đa thứcP(x)dưới dạng

P(x) =

m

Y

k=1

(akx2+bkx+ck), ak >0,akx2+bkx+ck >0 ∀x>0

Nhận xét với đa thức

akx2+bkx+ck >0 ∀x>0

ta viết

ax2+bx+c= (αkx2+βk)2+x(γk2+δ k), nên

P(x) =

m

Y

k=1

h

(αkx2+βk)2+x(γk2+δ k)

i

.

Mặt khác, ta có tích hai đa thức dạng (p2+q2) +x(r2+s2) đa thức có dạng Thật vậy, ta có

[(p21+q12) +x(r12+s21)][(p22+q22) +x(r22+s22)]

= [(p21+q12)(p22+q22)+x2(r21+s12)(r22+s22)]+x[(p21+q12)(r22+s22)+(r12+s21)(p22+q22)]. Theo Bài toán 3.37 ta có biểu diễn

(p21+q12)(p22+q22) +x2(r21+s21)(r22+s22) = [A(x)]2+ [B(x)]2,

(p21+q21)(r22+s22) + (r12+s21)(p22+q22) = [C(x)]2+ [D(x)]2. Vậy nên

P(x) = [A(x)]2+ [B(x)]2+x{[C(x)]2+ [D(x)]2} Trường hợp degP(x) = 2m+

Lập luận tương tự Bài toán 3.5 ta thu P(x) =

n

X

k=1

(x+d)(akx2+bkx+ck)

= [A(x)]2+ [B(x)]2+x{[B(x)]2+ [C(x)]2}= [B(x)]2+ [C(x)]2.

(169)

Bài toán 3.42. Cho đa thức f(x)∈R[x] thỏa mãn điều kiện f(x)>0 ∀x∈(−1; 1).

Chứng minh đa thức f(x)có thể biểu diễn dạng f(x) =

k

X

j=0

aj(1 +x)αj(1−x)βj

với aj >0, αj, βj ∈N.

Giải. Giả sửdegf(x) =m Đặt

1 +x

1−x =tx= t−1

t+ 1.

Dox∈(−1; 1)nênt >0 Vậyf(x) =ft−1

t+

>0 ∀t >0 Do đa thức Q(t) với Q(t) = (t+ 1)mft−1

t+

là đa thức thỏa mãn điều kiệnQ(t)>0 ∀t >0 Theo Bài tốn 3.5 tồn tạin∈Nsao cho

(t+ 1)m+nQ(t) = (t+ 1)m+nf(t−1

t+ 1)

là đa thức có hệ số khơng âm Suy

(t+ 1)m+nf

t−1

t+

=

k

X

j=0 btj,

với bj >0, k6m+n.

t+ = +x 1−x + =

2 1−x nên ta thu

2

1−x

m+n f(x) =

k

X

j=0 bj

1 +x

1−x

(170)

Suy

f(x) =

k

X

j=0 bj

2m+n(1 +x) j

(1−x)m+n−j

hay

f(x) =

k

X

j=0

aj(1 +x)j(1−x)m+n−j,

với aj = bj

2m+n >0 Từ ta có điều phải chứng minh

Bài tốn 3.43. Chứng minh tồn đa thức P(x) bậc n và nhận giá trị dương khoảng (−1; 1)và khơng thể biểu diễn dạng

P(x) =XAαβ(1−x)α(1 +x)β,

trong đó Aαβ >0, α+β 6n; α, β là số nguyên không âm.

Giải. Xét đa thức P(x) =x2+ε, đóε >0 Giả sử viếtP(x) dạng

P(x) =x2+ε= X

α+β≤2

Aαβ(ε)(1−x)α(1 +x)β, (6) ε >0, A(ε) >0, α β chạy tất số nguyên không âm mà α+β 62 Như vậy, vớiεbất kỳ tổng chứa vừa sáu số hạng Bằng cách thếx = 0vào (6) ta nhận Aαβ(ε) bị chặn với0< ε61 Cho ε dần tới0 cho tồn lim

ε→0A(ε) =A tất sáu số hạng, chuyển qua giới hạn ta nhận

x2 = X

α+β

Aαβ(1−x)α(1 +x)β.

Nhưng vớix= 0thì đồng thức khơng thoả mãn

Vậy đa thức P(x) bậc n nhận giá trị dương khoảng (−1; 1)đều biểu diễn dạng

P(x) = X

α+β n

(171)

Bài tập

Bài 3.1. Xác định đa thứcP(x) bậc nhỏ cho đồ thị hàm sốy =P(x) đi qua điểm A(2,0)và có điểm cực đại tại B(−1,0)C(0,0) vàD(1,0). Bài 3.2. Xác định đa thức P(x) bậc nhỏ cho đồ thị hàm số y=P(x)

có điểm uốn điểm cực đại tại A(0,0) B(1,0).

Bài 3.3. Xác định đa thức P(x) bậc nhỏ cho đồ thị hàm số y=P(x)

đi qua điểm A(−1,0)và có điểm uốn tại B(0,0) C(1,0).

Bài 3.4. Xác định hàm sốf(x)liên tục đoạn[0;π]cóf(0) = 0, f(x) >

0, ∀x ∈ (0;π) f(A), f(B), f(C) tạo thành độ đo góc tam giác vuông ứng với tam giác vuông ABC cho trước.

Bài 3.5.Xác định hàm sốf(x)liên tục đoạn[0; 1]sao chof(a), f(b), f(c)

luôn độ dài cạnh tam giác vuông nội tiếp đường trịn đường kính ứng với tam giác vngABC nội tiếp đường trịn đường kính

cho trước.

Bài 3.6. Cho đa thức bậcn (n>4) có nghiệm thực

f(x) = (n+ 1)(−1)n+1(xx1)(xx2)· · ·(xxn), x16x2 6· · ·6xn, trong có hai nghiệm kép kề Chứng minh nguyên hàmF(x)của f(x) là đa thức khơng thể có nghiệm thực.

Bài 3.7. Chứng minh nếu f(x) nghịch biến trên [a, b], thì n

X

i=1

(xixi−1)f(xi−1)> b

Z

a

f(x)dx > n

X

i=1

(xixi−1)f(xi).

Giả thiết có thêm điều kiện f00(x)<0 trong đoạn [a, b] Chứng minh rằng b

Z

a

f(x)dx > n

X

i=1

(xixi−1)

f(xi) +f(xi−1)

(172)

Nội suy bất đẳng thức

4.1 Nội suy bất đẳng thức bậc hai đoạn Ta nhắc lại số kiến thức liên quan đến biểu théc dạng bậc hai với hệ số thực Xét tam thức bậc hai f(x) =ax2+bx+c, a6= Khi

af(x) =

ax+b

2 − ∆

4,

với ∆ =b2−4ac.

Ta phát biểu kết quen thuộc dấu tam thức bậc hai Định lý 4.1. Xét tam thức bậc haif(x) =ax2+bx+c, a6= 0.

i) Nếu<0 thìaf(x)>0,∀x∈R.

ii) Nếu ∆ = thì af(x) > ∀x ∈ R Dấu đẳng thức xảy khi x=− b

2a.

iii) Nếu>0 thì af(x) =a2(xx1)(xx2) với x1,2 =−

b

2a∓ √

2|a|. (4.1)

Trong trường hợp này, af(x) <0 khi x ∈(x1, x2) af(x)>0 khi x < x1 hoặc x > x2.

Định lý 4.2 (Định lí đảo). Điều kiện cần đủ để tồn sốαsao choaf(α)<0

> x1 < α < x2, đó x1,2 là nghiệm của f(x) xác định theo

(4.1).

Để mô tả ý tưởng nội suy bất đẳng thức, ta xuất phát từ bất đẳng thức bậc hai quen biết sau

(173)

Dấu đẳng thức xảy x=y.

Cũng xuất phát từ bất đẳng thức dạng với(4.2)(ứng với06α61)

x

y

1−α

+

y

x

1−α

>2, (4.3)

hay

x y

α y2+

y

x

α

x2 >2xy, ta thu bất đẳng thức dạng tương đương

xαy2−α+x2−α >2xy. (4.4) Dấu đẳng thức xảy x=y.

Mặt khác, sử dụng đạo hàm để khảo sát hàm số, ta thu f(t) =t2−t2−α−+ 1>0, ∀t∈[0,1]. Vớit= x

y, ta thu

x2+y2 >xαy2−α+x2−αyα. (4.5) Hệ thức (4.5)có thể xem dạng bất đẳng thức nội suy bất đẳng thức bậc hai (4.2) Các dạng bất đẳng thức nội suy tương tự tham biến tuỳ ý xét phần cuối chương

Tiếp theo, ta xét vài dạng toán đánh giá ước lượng biểu thức có sử dụng tính chất tam thức bậc hai khoảng đoạn thẳng cho trước

Xét đa thức bậc hai hai biến (xem tam thức bậc hai x tham biến y)

F(x, y) =ax2+bxy+cy2, a6= 0,

∆ : = (b2−4ac)y2. Khi đó, ∆60 aF(x, y)>0, ∀x, y∈R

Vậy khib2 64aca >0 hiển nhiên

ax2+cy2 >|bxy|, ∀x, y∈R.

(174)

hay

u+v

2 >

uv, u, v>0.

Về sau, ta sử dụng tính chất dạng phân thức bậc hai y= a1x

2+b1x+c1 a2x2+b2x+c2 với điều kiện

a2>0, f2(x) = a2x2+b2x+c2 >0, ∀x∈R, để tìm cực trị số dạng tốn bậc hai

Bài toán 4.1. Xét tam thức bậc hai P(x) = x2−1 Tìm số nghiệm thực phân biệt phương trình sau:

P(P( .(P

| {z }

2006 chữ P

(x)) .)) =

Giải. Đặt

Pn(x) =P(P( .(P

| {z }

n chữ P

(x)) .)).

Nhận xét rằngP1(x)>−1với mọixnênPn+1(x) =P1(Pn(x))>−1với mọix

R vàn∈N∗ Vì phương trình Pn(x) = avới a <−1 khơng có nghiệm thực Ta chứng minh, phương pháp quy nạp tốn học, phương trình Pn(x) = a với a >0 ln có hai nghiệm thực phân biệt

Thật vậy, vớin= 1thì phương trìnhx1−1 =acó hai nghiệm phân biệt Giả sử phương trình Pn(x) =avớia >0 có hai nghiệm thực phân biệt Xét phương trìnhPn+1(x) =avới a >0 Ta có

Pn+1(x) =aP1(Pn(x)) =a⇔(Pn(x)− √

a+ 1)(Pn(x) +

a+ 1) = 0. Do phương trình Pn(x) +√a+ = 0vô nghiệm nên suy phương trình Pn+1(x) =acó hai nghiệm thực phân biệt

Tiếp theo, ta chứng minh, phương pháp quy nạp tốn học, phương trình Pn(x) = có n+ 1nghiệm thực phân biệt

Thật vậy, vớin= 1vàn= 2thì ta có kết hiển nhiên Giả sử, phương trìnhPn(x) = 0có n+ 1nghiệm thực phân biệt Xét phương trìnhPn+2(x) = 0, ta thu phương trình

Pn2(x)(Pn2(x)−2) = 0⇔Pn2(x)(Pn(x)− √

2)(Pn(x) + √

(175)

theo giả thiết quy nạp, có n+ 3nghiệm thực phân biệt

Kết luận: Vậy phương trình Pn(x) = an+ nghiệm phân biệt vậy, phương trình cho có 2007 nghiệm thực phân biệt

Tiếp theo ta trình bày số kết Lupas ước lượng tam thức bậc hai khoảng

Bài toán 4.2. Giả sử G(x) = P x2+Qx+R Khi bất đẳng thức G(x) >0

thoả mãn với mọi x∈[a, b], khi G(a)>0, G(b)>0 2Ga+b

2

G(a) +G(b)

2 >

p

G(a)G(b). (4.6) Giải. Giả sử (4.6) thoả mãn Ký hiệu

m=

p

G(b)−pG(a)

ba , n=

bpG(a)−apG(b)

ba , KK[G] :=

(ba)2

2G

a+b

2

G(a) +G(b)

2 −

p

G(a)G(b)

. (4.7) Khi K>0 Mặt khác

G(x) = (mx+n)2+K(xa)(ba)

suy

G(x)>0,∀x∈[a, b]. (4.8) Ngược lại, giả sử (4.8) thoả mãn Khi đóG(a)>0, G(b)>0và G(x)có thể viết dạng (Định lý Lukac)

G(x) = (m1x+n1)2+K1(xa)(bx) với K >0. (4.9) Nếu (4.9) ta chọn xa, a+b2 , b

(m1a+n1)2 = G(a)

(m1b+n1)2 = G(b) ; K1 =K, (4.10)

K chọn (4.7) Nhận xét hệ(4.10) cho ta m1, n1 ta có KK[G]>0,tức bất đẳng thức (4.6) chứng minh

Bài toán 4.3. Chứng minh với tam thức bậc haif(x) =Ax2+Bx+C ta có |f(x)|61 , ∀x ∈[a, b] xảy khi |f(a)|61,|f(b)|61 khi đó

(176)

Giải. Sử dụng kết toán 4.2

G(x)>0, ∀x∈[a, b],G(a)>0, G(b)>0, K[G]>0, (4.11) vớiG1(x) := 1−f(x)vàG2(x) =f(x) + 1.Thật

G1(x)>0 G2(x)>0

, ∀x∈[a, b], |f(a)|6 1,|f(b)|6 , K[G1]> 0, K[G2] > 0. Hai bất đẳng thức cuối với

          

1−2f

a+b

2

+f(a) +f(b)

2 −

p

(1−f(a)) (1−f(b))>0

1 + 2f

a+b

2

f(a) +f(b)

2 −

p

(1 +f(a)) (1 +f(b))>0

(4.12)

Bài toán 4.4. Cho p(x) = ax2+bx+c thoả mãn điều kiện

|p(0)|,

p

1

,|p(1)|

⊂[0,1].

Chứng minh rằng|a|68,|b|68, |c|61 |2ax+ 1|61, ∀x∈[0,1]. Giải. Để ý

a = 2p(0)−4p 12+ 2p(1) , b = −3p(0) + 4p 12−p(1)

c = p(0) , 2a+b = p(0)−4p 12+ 3p(1) , Sử dụng bất đẳng thức tam giác, ta có |a|68, |b|68,|c|61,|2a+b|68.Khi h(x) := 2ax+b ,thì|h(0)|68,|h(1)|68 kéo theo|h(x)|68 ,∀x∈[0,1]. Nhận xét 4.1. Chú ý đánh giá tối ưu Thật vậy, giả sử p(x) = 8x2−8x+ 1 Khi đó |p(x)|61 |p0(x)|=|16x−8|61 trên [0,1].

Bài toán 4.5. Giả sử M2 là tập hợp tất đa thức bậc không và M∗2 : =p∈ M2; |p(t)|61,∀t∈[0,1] .Tìm tất đa thức Q, Q∈ M∗

2, sao cho với mọi p từ M∗2 ta có

|p(x)|6Q(x) , ∀x∈x∈(−∞,0]∪[1,∞). Chứng minh nghiệm Q là nhất.

Giải. Ta chứng minh rằngQ(x) = 8x2−8x+ =T2(2x−1)trong đóT2(z) = 2z2−1là đa thức Chebychev loại Giả sử p(x) =ax2+bx+c∈ M∗2.

p(x) = (2x−1)(x−1)p(0)−4x(x−1)p

1

(177)

ứng với mọix∈(−∞,0)∪(1,∞) nên

|p(x)|6(2x−1)(x−1) + 4x(x−1) +x(2x−1) = 8x2−8x+ =:Q(x). Để ý |Q(t)|=|1−8t(1−t)|61,∀t ∈[0,1], nên Q∈ M∗2.Tính nghiệm hiển nhiên

Nhận xét 4.2. Kết toán tập M∗2 được mở rộng như sau. M∗2 : =p∈ M2 ; p k261, khi k= 0,1,2 .

Bài toán 4.6. Cho A, B, C ∈ R, M > . Xét f(x) =Ax2+Bx+C thoả mãn điều kiện pt(1−t) |f(t)|6 M , ∀t ∈ [0,1]. Chứng minh đó

ứng với x ta có

|f(x)|66M + 24M(|x(1−x)| −x(1−x) ) . (4.13) Giải. Nhận xét rằng, nếup(x) := f(x)

6M =ax

2+bx+c , thì ta cần chứng minh từ điều kiện 6px(1−x) |p(x)|61, ∀x∈[0,1],suy

|p(x)|6

1 , x∈[0,1]

8x2−8x+ , x∈(−∞,0)∪(1,∞)

Ta chứng minh

|p(x)|6

6px(1−x) , ∀x

∈(0,1), (4.14) kéo theo |p(x)|61,∀x∈[0,1].Xét [0,1] hệ điểm x1, x2, x3:

x1 =

1

2 −h , x2=

2 , x3 =

2 +h với h

0,1

2

.

Ký hiệuJ = [0, x1)∪(x3,1], ω(x) = (xx1)(xx2)(xx3).Khi vớixJ, ta có

|ω(x)|

X

k=1

1

|x−xk|

=ω(x)

3

X

k=1

1

xxk

= −h

2−

3x(1−x).

Mặt khác thìp(x) = ω(x) 2h2

p(x1)

xx1

−2 p(x2)

xx2

+ p(x3)

xx3

. Nếu xảy ra(4.14)thì ta có ngay2|p(x2)|6

3 , |p(xj)| ≤ 6√x1x3

, j ∈ {1,3}.Do

|p(x)|6 |ω(x)|

12h2√x1x3

1

|x−x1|+

4√x1x3 |x−x2|+

1

|x−x3|

(178)

Đặt 4√x1x3 = 1, tức h=h1 = √

3

4 ,ta thu

|p(x)|6 16

9 |ω(x)|

3

X

k=1

1

|x−xk|= 1−

16

3 x(1−x)61, ∀x∈J

Nếu x ∈ [x1, x3] h = h1 , 6px(1−x) > 6√x1x3 =

2

x∈[x1, x3]suy |p(x)|6

2 <1.

Giả thiết |p(x)|68x2−8x+ 1thoả mãn với x∈(−∞,0]∪[0,∞) kéo theo toán 4.5 xét

Nhận xét theo nghĩa kết nhận tốt toán xấp xỉ Thật vậy, xét

p∗(x) = 16 x

2− 16

3 x+ = 1− 16

3 x(1−x),

thì 6px(1−x)

p∗(x)

61, x∈[0,1] .Mặt khác |p∗(x)|6

1 , x∈[0,1]

8x2−8x+ , x∈(−∞,0)∪(1,∞)

Điều ta khơng thể có ước lượng tốt

Bài toán 4.7. Cho biết f(x) := 4x2+Bx+C = 0có hai nghiệm phân biệt trong

(0,1).Chứng minh hai hệ sốB C không nguyên. Giải. Ta chứng minh phương pháp phản chứng Giả sửBC số nguyên Ta thấyf(0)vàf(1)cũng số nguyên Từ giả thiếtf(0)f(1)>0

suy f(0)f(1)>1. Nếu xảy f(x1) = f(x2) = 0,0 < x1 < x2 <1,thì hiển nhiên

16f(0)f(1) = 16x1(1−x1)

| {z }

1

x2(1−x2)

| {z }

1

<1 .

Bất đẳng thức phía bên phải khơng xảy đẳng thức trường hợp kéo theo hai nghiệmx1 vàx2

1

2,mâu thuẫn với giả thiết

Bài toán 4.8. Giả thiết rằng x1, x2 là nghiệm tam thức bậc hai p(x) =x2−a

2(b+c) +b2(c+a) +c2(a+b) a+b+c x+

(179)

Giải. Ta có

p(ab) = abc

a+b+c(ca)(cb)>0 p(ac) = − abc

a+b+c(ba)(cb)<0 p(bc) = abc

a+b+c(ba)(ca)>0.

Vì rằngab < ac < bcnên nghiệm củap(x)là phân biệt nằm

(ab, ac),(ac, bc) Nhưng vì1< ab < ac < bc <4 nên ta có điều phải chứng minh

Bài toán 4.9. Đặt ϕ= 1+

2 f(x) := +ϕcosx+Acos 2x+Bcos 3x. Xác định giá trị A B sao cho f(x)>0 ứng với mọi x∈[0, π].

Giải. Giả sửF(t) =αt3+βt2+γt+δ thoả mãn điều kiện F(t)>0,∀t∈[−1,1].

Khi F biểu diễn dạng

F(t) = (1−t)(at+b)2+ (1 +t)(ct+d)2, a, b, c, d∈R. (4.15) Ta có bất đẳng thức f(x)>0,∀x∈[0, π]tương đương với

F0(t) :=f(arccost)>0,∀t∈[−1,1],trong

F0(t) = +ϕt+A(2t2−1) +B(4t3−3t) (4.16)

= 4Bt3+ 2At2+ (ϕ−3B)t+ 1−A (4.17) Dễ thấy rằngϕ= cosπ5, ϕ2 =ϕ+

cosπ =

ϕ

2,cos 2π

5 =

ϕ−1 ,cos

3π

5 =

1−ϕ

2 . (4.18)

  

f 3π5 = 12 −ϕ(A−B)

2 =

ϕ(ϕ−1−(A−B)) >0, f(π) = −(ϕ−1−(AB))>0.

(4.19) Từ (4.19) suy AB = ϕ−1và f 3π5 = f(π) = 0.

F0(−1) =F0(t0) = 0, t0 = 1−ϕ2 ,nên từ (??)-(4.16) ta thu a=b= 0vàd=−ct0.Do đóF0(t) =K(1 +t)(tt0)2, hay

F0(t) =K

t3+ϕt2+ 3ϕ−2

4 t+

2−ϕ

4

(180)

Từ (4.16)−(4.20)ta thu K= 4(3−ϕ)5 A= 2(2ϕ−1)

5 =

2

5

5 , B=

3−ϕ

5 =

5− √

5 10 .

Tương tự, ta có f(x) = 2K cosx−cos3π5 · cosx22 với K định nghĩa phần

Bài tốn 4.10. Tìm giá trị lớn biểu thức

P = (xy)(yz)(zx)(x+y+z), trong đó x, y, z là số thực thuộc đoạn [0,1].

Giải. Không giảm tổng quát, ta coi x số lớn nhất: x = max{x, y, z} Vậy xảy hai khả năng: x>y>z x>z>y.

Khix>y>z, thì P 60. Khix>z>y hiển nhiên

xz>0, zy>0, 06xz61. Suy

P = (xy)(yz)(zx)(x+y+z)6(zy)(xz)(x+y+z), hay

4P 6[2(zy)][(

3 + 1)(xz)][(

3−1)(x+y+z)].

Theo bất đẳng thức trung bình cộng trung bình nhân, vế trái cho ta bất đẳng thức

[2(z−y)][(

3+1)(x−z)][(

3−1)(x+y+z)]6

27[2(z−y)+(

3+1)(x−z)+(

3−1)(x+y+z)]3, hay

[2(zy)][(

3 + 1)(xz)][(

3−1)(x+y+z)]6

27[2

3x−(3− √

3)y]3. Để ý

062

3x−(3− √

3)y62

3. Suy

4P

(181)

hay

P

3 .

Do vậy, giá trị lớn biểu thứcP = (xy)(yz)(zx)(x+y+z)

8

3 ,

    

xy=

2(zy) = (√3−1)(xz) = (√3−1)(x+y+z)

x= 1, y = 0, hay(x, y, z) =

1,0,√1

3

4.2 Tam thức bậc tuỳ ý hàm phân thức quy Tiếp theo ta chuyển sang xét tam thức bậc tuỳ ý phân thức dạng quy Nhận xét bất đẳng thức dạng sơ đẳng dạng

x2+ 1>2x; ∀x∈R (4.21) bất đẳng thức "tam thức bậc hai" ứng với trường hợp dấu đẳng thức xảy khix= Khi có nhu cầu mở rộng dạng bất đẳng thức (4.21) cho tam thức bậc để thu bất đẳng thức có dạng tương tự cách thay số sốα để có cơng cụ sử dụng cho toán với lũy thừa tuỳ ý cho đảm bảo dấu đẳng thức xảy khix= 1.Ta thu phépnội suy bất đẳng thức (4.21) Khi đó, ta thu bất đẳng thức dạng

+α−1>αx; ∀x∈R+, (4.22) làbất đẳng thức Bernoulli quen biết Vì thế, xem bất đẳng thức (4.22) bất đẳng thức tam thức bậc α(α >1)

Trong trường hợp dấu đẳng thức xảy x = x0 (x0>0) cho trước ta cần thay bất đẳng thức (4.22) bất đẳng thức sau Tiếp theo, ta quay lại xét bất đẳng thức (4.21) xem bất đẳng thức tam thức bậc (2.1)(ứng với luỹ thừa luỹ thừa x), trường hợp dấu đẳng thức xảy x = Hồn tồn theo cách tương tự ta mở rộng bất đẳng thức cho tam thức (α, β) (α > β >0)để có bất đẳng thức tương tự (4.21) cách thay lũy thừa sốα lũy thừa bởiβ

+

α

β

−1> α

βx β

(182)

Dấu đẳng thức xảy x = Bất đẳng thức (4.23) bất đẳng thức Bernoulli tam thức bậc (α, β) , ứng với trường hợp đẳng thức xảy x=

Để sử dụng bất đẳng thức Bernoulli trường hợp dấu đẳng thức xảy x = x0 > cho trước ta cần thay bất đẳng thức (4.23) bất đẳng thức sau

x x0

α

+α

β −1> α β(

x x0

)β; ∀x∈R+;α > β >0. Dấu đẳng thức xảy x=x0

Tiếp theo, ta nêu ứng dụng quan trọng bất đẳng thức trung bình cộng trung bình nhân suy rộng liên quan đến lớp hàm số, thường gọi hàm phân thức quy Bất đẳng thức đảm bảo dấu đẳng thức xảy khix=

Trước hết ta xét lớp hàm phân thức quy biến

Định nghĩa 4.1. Hàm sốf(x) xác định tập R+ được gọi hàm phân thức chính quy, nếu

f(x) =

n

X

k=1 akxαk,

trong đó

ak >0, k= 1,2, , n;

n

X

k=1

akαk = 0. (4.24)

Ví dụ 4.1. Dễ dàng kiểm chứng hàm số sau phân thức quy f1(x) = + 2x+ 3x2+ 51

x +

1

x3, f2(x) = 4x+

1

x+

sinα xsinα +

cosα xcosα. Từ định nghĩa, ta dễ dàng kiểm tra tính chất sau

Tính chất 4.1. Nếu f(x) là hàm phân thức quy, thì f(x)>0 ứng với mọi x >0.

Nếu f(x) g(x) là hàm phân thức quy, với cặp số dương α, β, hàm số

h(x) :=αf(x) +βg(x)

cũng hàm phân thức quy.

(183)

cũng hàm phân thức quy.

Nếu f(x) là hàm phân thức quy, hàm số h(x) := [f(x)]m, m∈N∗, cũng hàm phân thức quy.

Tiếp theo, ta định nghĩa hàm phân thức quy nhiều biến

Định nghĩa 4.2. Hàm sốf(x1, x2, , xn)được gọi hàm phân thức quy trên tập

{x1 >0, x2 >0, , xn>0}, nếu

f(x1, x2, , xn) =

m

X

k=1 akxαk1

1 x αk2

2 · · ·x αkn

n , ak >0, k= 1,2, , m, (4.25)

trong đó

         

a1α11+a2α21+· · ·+amαm1 =

a1α12+a2α22+· · ·+amαm2 = · · · · a1α1n+a2α2n+· · ·+amαmn =

(4.26)

Định nghĩa 4.3. Giả sử hàm số f(x1, x2, , xn) là hàm phân thức quy, tức f(x1, x2, , xn) thoả mãn điều kiện(4.25)-(4.26) Khi hàm số

hj(xj) := m

X

k=1 akx

αkj

j , j= 1,2, , n,

được gọi phân thức thành phần biến xj của f(x1, x2, , xn). Từ định nghĩa, ta dễ dàng kiểm tra tính chất

Định lý 4.3. Hàm số f(x1, x2, , xn) là hàm phân thức quy chỉ khi hàm phân thức thành phần của f(x1, x2, , xn) cũng hàm phân thức quy.

Tiếp theo, ta chứng minh định lý sau

Định lý 4.4. Với hàm phân thức quy f(x1, x2, , xn) trên tập {x1>0, x2>0, , xn>0} dạng

f(x1, x2, , xn) = m

X

k=1

(184)

trong đó           

a1α11+a2α21+· · ·+amαm1=

a1α12+a2α22+· · ·+amαm2= · · · ·

a1α1n+a2α2n+· · ·+amαmn= 0, ta có

f(x1, x2, , xn)>

m

X

k=1 ak.

Chứng minh. Để chứng minh định lý, ta nhắc lại hệ bất đẳng thức trung bình cộng trung bình nhân suy rộng Với cặp dãy số dươngu1, u2, , un; α1, α2, , αn, ta có

u1α1+u2α2+· · ·+unαn α1+α2+· · ·+αn

>

h

1

1 u α2

2 · · ·u αn

n

i

α1 +α2+···+αn

. Dấu đẳng thức xảy

u1=u2=· · ·=un= 1. Sử dụng kết này, ta thu

f(x1, x2, , xn) a1+a2+· · ·+an

=

m

P

k=1 akx

αk1

1 x αk2

2 · · ·x αkn

n a1+a2+· · ·+an

>

x

m

k=1

akαk1 x

m

k=1

akαk2 · · ·x

m

k=1

akαkn n

a1+a2+···+an = 1, giả thiết

m

X

k=1

akαkj = 0, j = 1,2, , n.

Dấu đẳng thức xảy x1 =x2 =· · ·=xn= 1. Đối với hàm phân thức tuỳ ý (khác hằng) với hệ số không âm , ta có

Nhận xét 4.3. Với hàm phân thức dạng g(x) =

m

X

k=1

(185)

đặt

a1+a2+· · ·+am=p,

a1α1+a2α2+· · ·+amαm =q, thì hàm số

f(x) :=x

q pg(x)

là hàm phân thức quy. Chứng minh. Thật vậy, ta có

f(x) = x−qp

m

X

k=1

akxαk = m

X

k=1 akx

αk−qp

m

X

k=1 ak

αkq p

=

m

X

k=1

akαkm

X

k=1 akq

p =qp q

p =qq = 0,

điều phải chứng minh

Từ đây, ta thu kết quan trọng sau Định lý 4.5. Mọi hàm phân thức dạng

g(x) =

m

X

k=1

akxαk; ak >0; k= 1, , n, đều có tính chất

g(x)>g(1)x

q

p, ∀x >0,

trong đó

p=a1+a2+· · ·+am,

q=a1α1+a2α2+· · ·+amαm. Chứng minh. Thật vậy, theo nhận xét trên, ta có

f(x) =xqp

m

X

k=1 akxαk phân thức quy, nên theo định lý 3.5,

f(x)>f(1).

(186)

4.3 Chuyển đổi điều chỉnh số theo thứ tự dần đều

Định nghĩa 4.4. Xét hai số {xk, ykI(a, b), k= 1,2, , n}, thoả mãn các điều kiện

x1>x2>· · ·>xn, y1 >y2 >· · ·>yn

             

x1 >y1

x1+x2 >y1+y2 .

x1+x2+· · ·+xn−1>y1+y2+· · ·+yn−1 x1+x2+· · ·+xn=y1+y2+· · ·+yn

(4.27)

Khi ta nói số{yk} gần số {xk}. Ta nhắc lại định lý Karamata

Định lý 4.6 (Karamata). Xét hai dãy số{xk, ykI(a, b), k= 1,2, , n}, thoả mãn điều kiện

x1>x2>· · ·>xn, y1 >y2 >· · ·>yn

             

x1 >y1

x1+x2 >y1+y2 .

x1+x2+· · ·+xn−1>y1+y2+· · ·+yn−1 x1+x2+· · ·+xn=y1+y2+· · ·+yn

(4.28)

Khi đó, ứng với hàm lồi thực sự f(x) (f”(x)>0) trênI(a, b), ta có f(x1) +f(x2) +· · ·+f(xn)>f(y1) +f(y2) +· · ·+f(yn). (4.29) Tuy ta phải sử dụng nhiều giả thiết có tính nhân tạo giả thiết sử dụng tự nhiên rộng rãi ứng dụng nhiều lĩnh vực thực tiễn khác Hệ điều kiện(4.28)chỉ dựa so sánh tuyến tính (của tổng số học) đại lượng cho trước, coi dạng giả thiết dạng đơn giản

(187)

Định lý 4.7 (I Schur). Điều kiện cần đủ để hai dãy số đơn điệu giảm {xk, yk;k= 1,2, , n}, thoả mãn điều kiện

              

x1 >y1

x1+x2 >y1+y2 .

x1+x2+· · ·+xn−1>y1+y2+· · ·+yn−1 x1+x2+· · ·+xn=y1+y2+· · ·+yn

(4.30)

là chúng có phép biến đổi tuyến tính dạng

yi =

n

X

j=1

aijxj, i= 1,2, , n,

trong đó

akl>0, n

X

j=1

akj = 1, n

X

j=1

ajl= 1; k, l= 1,2, , n.

Khi đẳng thức cuối giả thiết định lý bị phá vỡ, ta cần phải điều chỉnh có thêm giả thiết hàm số cho để kết luận Định lý dạng Karamata tương tự hiệu lực Rõ ràng cho hai số tuỳ ý, việc thứ tự chiều đưa vào hệ thống so sánh thường rườm rà cảm giác có tính áp đặt, khơng thật tự nhiên nên giả thiết Karamata thường vào lãng quên, ý Nhưng phân tích kỹ, ta thấy giả thiết Karamata hữu ích việc minh hoạ giả thiết thường hiển nhiên tốn thường gặp chương trình ntốn phổ thơng Mặc dù định lý có nhiều ứng dụng toán điều khiển tối ưu quy hoạch, nhu cầu cấp bách cần có giải pháp hữu hiệu để cải tiến cách trình bày đơn giản cách cho giả thiết Đặc biệt, ta cần có cách mơ tả Định lý Schur mà không cần viện trợ đến lý thuyết ma trận

Để dễ nắm bắt tư tưởng thứ tự gần theo hệ thống tổng quát, ta quan sát trường hợp đơn giản với ba số thực {a, b, c} Xét trọng số {α, β, γ}, tức ba số khơng âm có tổng Khi đó, tổng

s:=αa+βb+γc

chính giá trị trung bình theo trọng số cho hiển nhiên có thứ tự tự nhiên

(188)

Xét số {a1, b1, c1}phụ thuộc vào tham số α, β, γ, d:

    

a1=a+(sa) b1 =b+(sb)

c1 =c+(sc)

Ta dễ dàng chứng minh

Bổ đề 4.1. Khi d > thì số {a1, b1, c1} gần hơn {a, b, c}, khi d <0

thì có điều ngược lại.

Định lý 4.8. Cho hàm sốy=f(x)có đạo hàm cấp hai mọix∈(a;b)sao cho f00(x)>0 với mọi x∈(a;b) Xét ba số thực {a, b, c} và trọng số {α, β, γ}, tức ba số khơng âm có tổng Xét số{a1, b1, c1}phụ thuộc vào tham số α, β, γ, d:

   

a1=a+(sa) b1 =b+(sb)

c1 =c+(sc)

trong đó

s:=αa+βb+γc. Khi đó, ta ln có

f(a1) +f(b1) +f(c1)6f(a) +f(b) +f(c).

Nhận xét rằng, điều kiện (4.30) cho ta thuật toán nắn số cho trước phương pháp biến đổi tuyến tính Đối với bạn đọc có kiến thức đại số tuyến tính tốn học cao cấp, dễ nhận thấy phép biểu diễn hàm lồi (lõm) biến mở rộng cho hàm lồi nhiều biến Cụ thể là, ứng với hàm lồiF(x1, x2, , xn), ta có

F(x1, x2, , xn) =

= max

(t1, ,tn)

h

F(t1, t2, , tn) +

n

X

i=1

(xiti)∂F

∂ti

i

. (4.31)

Sử dụng cách chứng minh tương tự bất đẳng thức Karamata, ta thu

Định lý 4.9. Giả sửF(x1, x2, , xn) thoả mãn điều kiện (??) Khi với mọi cặp số đơn điệu giảm (x1, x2, , xn), (y1, y2, , yn), thoả mãn điều kiện Schur (4.30), ta có

(189)

Nhận xét 4.4. Có thể nói Định lý 4.9 cho ta cơng cụ mạnh để thực q trình dần thuật toán dồn biến để chứng minh nhiều dạng bất đẳng thức phức tạp.

Thật vậy, từ(4.32), ta có hệ F(x1, x2, , xn)>F

x1+x2

2 ,

x1+x2

2 , x3, , xn

.

Các giả thiết Karamata cho ta xây dựng tường minh khái niệm gần hệ thống tam giác

Định nghĩa 4.5. Với tam giác ABC cho trước, ta kí hiệu δ∆ABC = max{A, B, C} −min{A, B, C} và gọi δ∆ABC là độ gần tam giác ABC.

Rõ ràngδ∆ABC >0vàδ∆ABC = 0khi tam giácABC tam giác

Định nghĩa 4.6. Với cặp tam giácA1B1C1 vàA2B2C2 thoả mãn đồng thời các điều kiện

max{A1, B1, C1}6max{A2, B2, C2}, min{A1, B1, C1}>min{A2, B2, C2}, thì ta nói cặp tam giác A1B1C1 A2B2C2 là cặp thứ tự tam giác A1B1C1 gần tam giácA2B2C2.

Vậy trường hợp có thứ tự ứng với cặp tam giác A1B1C1 A2B2C2 (với A1 > B1 > C1, A2 > B2 >C2) thoả mãn đồng thời điều kiện

A1 6A2, C1>C2,

thì ta có tam giác A1B1C1 gần tam giác A2B2C2 tam giác A2B2C2 xa tam giácA1B1C1.Rõ ràng tam giác gần tam giác khác

Nhận xét 4.5. Trong tập hợp tam giác khơng nhọn tam giác vng cân gần tam giác khác.

Ta nhắc lại tính chất sau

Tính chất 4.2. Cho hàm số y=f(x) có đạo hàm cấp haif”(x) trong (a, b). a) Nếu f”(x)>0 với mọi x∈(a, b) thì

f(x)>f(x0) +f0(x0)(xx0), với x, x0 ∈(a, b). b) Nếuf”(x)60 với mọi x∈(a, b) thì

(190)

Tính chất 4.3. Cho tam giácABC và cho ba số không âm α, β, γ sao cho α+β+γ= 1 Đặt

   

A0=αA+βB+γC B0 =αB+βC+γA C0 =αC+βA+γB. Khi tam giác A0B0C0 gần tam giác ABC.

Nhận xét 4.6. Vì ta quan tâm đến góc dương A, B, C có tổng bằng π, nên kết toán với lớp hàm tựa lồi (tựa lõm) trong(0, π). Kết sau bao hàm hầu hết bất đẳng thức biểu thức đối xứng dạng tam giác

Bài toán 4.11. Cho tam giác A2B2C2 gần tam giác A1B1C1 và cho hàm số f(x) f00(x)60 với mọix∈(0, π) Khi đó

f(A1) +f(B1) +f(C1)6f(A2) +f(B2) +f(C2). Giải Dof”(x)60với x∈(0, π), nên

f(x)6f(x0) +f0(x0)(xx0), ∀x0 ∈(0, π). (4.33) Khơng tính tổng quát, ta coi

A1>B1 >C1, A2 >B2 >C2.

Khi đó, tam giácA2B2C2 gần tam giácA1B1C1 nên

    

A1>A2

A1+B1 >A2+B2

A1+B1+C1 =A2+B2+C2.

(4.34)

Theo (4.33)và (??),

    

f(A1) 6f(A2) +f0(A2)(A1−A2) f(B1)6f(B2) +f0(B2)(B1B2)

f(C1)6f(C2) +f0(C2)(C1−C2).

(4.35)

Cộng vế tương ứng (4.34), ta

f(A1) +f(B1) +f(C1)6f(A2) +f(B2) +f(C2)

+ [f0(B2)−f0(C2)][(A1+B1)−(A2+B2)] + [f0(A2)−f0(B2)](A1−A2)

(191)

Tiếp theo ta xét số hệ bất đẳng thức đan dấu sinh hàm lồi Các bất đẳng thức dạng quy bất đẳng thức Karamata quen biết

Định lý 4.10 (Bất đẳng thức Szego). Cho hàm sốf(x)xác định lồi tập

[0, a]với a >0 và cho dãy 2n−1 số không âm đơn điệu giảm a>a1 >a2 >· · ·>a2n−1 >0.

Khi ta có bất đẳng thức 2n−1X

j=1

(−1)j−1f(aj)>f

2n−1X j=1

(−1)j−1aj

.

Cũng vậy, theo cách thức lập luận trên, ta có kết sau (bạn đọc tự chứng minh)

Định lý 4.11 (Bất đẳng thức Bellman). Cho hàm số f(x) xác định lồi trên tập [0, a]với a >0 vàf(0)60 Xét dãy n số không âm đơn điệu giảm

a>a1 >a2 >· · ·>an>0. Khi ta có bất đẳng thức

n

X

j=1

(−1)j−1f(aj)>f

Xn j=1

(−1)j−1aj

.

Định lý 4.12 (Bất đẳng thức Olkin). Cho hàm sốf(x) xác định lồi tập

[0, a]với a >0 Xét cặp dãy số không âm đơn điệu giảm a1 >a2>· · ·>an>0

1>α1>· · ·>αn>0. Khi ta có bất đẳng thức

h

1− n

X

j=1

(−1)j−1αj

i

f(0) +

n

X

j=1

(−1)j−1αjf(aj)

>f

2n−1X j=1

(−1)j−1αjaj

(192)

Tiếp theo ta xét vài toán liên quan đến áp dụng giả thiết định lý Karamata theo cách nắn dần số cho trước Chúng ta biết đặc điểm chung bất đẳng thức nhiều biến dạng dấu đẳng thức thường xảy tất vài phận biến số Vì vậy, tuỳ theo đặc thù toán cho, ta chọn cách tiếp cận trình điều chỉnh dần toàn phần phận biến số Ta thu phương pháp điều chỉnh trình nắn (làm) dần biến số cho trước Thơng thường, q trình nắn dần phận biến số thực đơn giản, thường thơng qua giá trị trung bình quen biết trung bình cộng, trung bình nhân, trung bình điều hồ, Q trình điều chỉnh nắn dần cho phép ta giảm dần lượng biến số biểu thức xét nên thường gọi phương pháp dồn biến gộp biến Tiếp theo sau đây, ta xét vài dạng ví dụ dồn biến dạng đơn giản Đó q trình dồn biến trung bình cộng trung bình nhân, tức điều chỉnh theo hướng dần số cho trước Cịn có nhiều dạng điều chỉnh khác dựa vào chuyển đổi trung bình điều hồ, trung bình đồng bậc, Tuy nhiên, phương pháp dồn biến dựa vào trung bình cộng trung bình nhân dạng đơn giản Ngoài ta nêu rõ điều kiện thực phương pháp điều chỉnh nêu

Để chứng minh bất đẳng thức

f(x1, x2, , xn)>0, (4.36) ta chứng minh

f(x1, x2, , xn)>f

x1+x2

2 ,

x1+x2

2 , , xn

. (4.37)

hoặc

f(x1, x2, , xn)>f

x1x2,

x1x2, , xn

. (4.38)

Sau đó, chuyển sang việc chứng minh (4.36)về chứng minh bất đẳng thức g(x2, x3, , xn)>0,

tức chứng minh bất đẳng thức có biến số Dĩ nhiên, bất đẳng thức

(4.37),(4.38)có thể khơng đúng, số điều kiện Vì ta thay đổi hai biến số nên kiểm tra tính đắn bất đẳng thức cách dễ dàng

Câu hỏi hiển nhiên xuất Bài tốn mở 1. Với điều kiện thì

f(x1, x2, , xn)>f

x1+x2

2 ,

x1+x2

2 , , xn

(193)

ứng với số (x1, x2, , xn). Bài tốn mở 2. Với điều kiện thì

f(x1, x2, , xn)>f

x1x2,x1x2, , xn

, ứng với số (x1, x2, , xn).

Như ta nêu đầu chương, câu trả lời (điều kiện đủ) cho hai tốn mở dễ dàng tìm lớp hàm lồi (lõm) nhiều biến Phép chứng minh dựa vào tính chất cặp dãy thoả mãn điều kiện Schur Ta sử dụng biểu diễn dạng tuyến tính

Định lý 4.13. Với hàm lồi F(x1, x2, , xn), ta có biểu diễn F(x1, x2, , xn) = max

(t1,t2, ,tn)

h

F(t1, t2, , tn) +

n

X

k=1

(xktk)∂F

∂tk(t1, t2, , tn)

i

. Nhờ định lý này, ta dễ dàng chứng minh (theo cách chứng minh bất đẳng thức (định lý) Karamata)

Định lý 4.14 (Bất đẳng thức Ostrowski). Với hàm lồi F(x1, x2, , xn) với số y1, y2, , yn gần số x1, x2, , xn, ta có bất đẳng thức

F(x1, x2, , xn)>F(y1, y2, , yn).

Ta xét toán sau để minh hoạ phương pháp (xem [12], [5]) Bài toán 4.12. Chứng minh nếux, y, z, >0 thì

2(x2+y2+z2) + 3(xyz)2/3>(x+y+z)2. Chứng minh. Xét hàm số

F(x, y, z) = 2(x2+y2+z2) + 3(xyz)2/3−(x+y+z)2 =x2+y2+z2−2xy−2yz−2zx+ 3(xyz)2/3

Không tính tổng quát, ta giả sửx6y6z Ta cần chứng minhF(x, y, z)>0 Thực điều chỉnh dồn biến trung bình nhân, ta chứng minh

(194)

Thật vậy, xét hiệu d=F(x, y, z)−F(x,yz,yz) Ta có

d=x2+y2+z2−2xy−2yz−2zx−(x2+yz+yz−2xyz−2xyz−2yz) + 3(xyz)2/3−3(xyz)2/3

=y2+z2−2yz+ 4xyz−2x(y+z) = (yz)2+ 2x(−y−z+ 2√yz) = (yz)2−2x(√y−√z)2 = (√y−√z)2[(√y+√z)2−2x]

= (√y−√z)2[(y+z−2x) + 2√yz]>0

x6y6z nên suy ray+z>2x Từ suy bất đẳng thức (4.39)đúng Mặt khác, ta có

F(x,yz,yz) =x2−4xyz+ 3(xyz)2/3. Do

x2+ 3(xyz)2/3=x2+ (xyz)2/3+ (xyz)2/3+ (xyz)2/3

>4(x4y2z2)1/4= 4xyz

(theo bất đẳng thứcgiữa trung bình cộng trung bình nhân cho bốn số khơng âmx2,(xyz)2/3 ,(xyz)2/3,

(xyz)2/3.)

Do vậyF(x,yz,yz)>0 Từ suy bất đẳng thức cần chứng minh

4.4 Một số mở rộng định lý Jensen

Trong ứng dụng, việc xây dựng kỹ thuật biến đổi để áp dụng định lý, nhà toán học quan tâm đến bất đẳng thức hàm, tức mở rộng bất đẳng thức tổng quát cho lớp hàm xét Những thành tựu theo hướng bùng nổ năm gần Điều dẫn đến lượng tập sáng tác trở nên phong phú hỗn độn, thiếu tính hệ thống Vì vậy, nhu cầu hệ thống hoá cho tiêu chuẩn nhận biết tính đắn nhiều lớp bất đẳng thức trở nên cấp bách

Vào năm 1965, T Popoviciu chứng minh định lý sau

Định lý 4.15 (T Popoviciu). Với hàm lồi trên I(a, b) và với mọix, y, zI(a, b), ta có bất đẳng thức

f(x) +f(y) +f(z) + 3f

x+y+z

3

>2f

x+y

2

+ 2f

y+z

2

+ 2f

z+x

2

(195)

Nhận xét định lý mở rộng thực kết qủa quen biết (bất đẳng thức Jensen) hàm lồi Thật vậy, theo bất đẳng thức Jensen,

f(x) +f(y) +f(z)>f

x+y

2

+f

y+z

2

+f

z+x

2

.

3fx+y+z

3

6fx+y

2

+fy+z

2

+fz+x

2

. Do vậy, định lý Popoviciu cho ta thực phép cộng trái chiều

Hệ 4.1. Với hàm lồi trênI(a, b)và với mọix, y, zI(a, b),06α63, ta có bất đẳng thức

f(x) +f(y) +f(z) +αf

x+y+z

3

>

1 + α

h

f

x+y

2

+f

y+z

2

+f

z+x

2

i

. Chứng minh. [Chứng minh định lý Popoviciu]

Ta coix>y >z Khi xảy hai khả năng: x> x+y+z

3 >y>z

hoặc

x>y> x+y+z

3 >z.

Ta cần xét trường hợpx>y> x+y+z

3 >z đủ

Khi dễ dàng kiểm tra x>y> x+y+z

3 >

x+y+z

3 >

x+y+z

3 >z, (4.40)

x+y

2 >

x+y

2 >

x+z

2 >

x+z

2 >

y+z

2 >

y+z

2 (4.41)

x+y+z+

x+y+z

3

=

x+y

2 +

y+z

2 +

z+x

2

.

Ta thu dãy(4.40)gần hơn(4.41) Theo định lý Karamata, ta điều

phải chứng minh

(196)

Định lý 4.16 (A Lupas). Với số dươngp, q, rvà với mọix, y, zI(a, b), ta có bất đẳng thức

pf(x) +qf(y) +rf(z) + (p+q+r)f

px+qy+rz

p+q+r

>(p+q)f

px+qy

p+q

+ (q+r)f

qy+rz

q+r

+ (r+p)f

rz+px

r+p

(4.42) Chứng minh. Tương tự cách chứng minh định lý Popoviciu, không tính tổng quát, giả thiết

x>y > px+qy+rz

p+q+r >z.

Từ đó, ta áp dụng định lý Karamata cho số có trọng để thu được(??)

Ta nhắc lại giả thiết Karamata hai số sau Ta nói véctơ −→A = [a1, a2, , an]với giả thiết a1 > a2 >· · ·> an xa so với véctơ −→B = [b1, b2, , bn] với giả thiết b1 >b2 > · · ·>bn, ký hiệu −→A −→B,

a1 >b1 a1+a2 >b1+b2

a1+a2+· · ·+an−1 >b1+b2+· · ·+bn−1 a1+a2+· · ·+an=b1+b2+· · ·+bn

Dựa vào định lý Karamata, ta dễ dàng chứng minh định lý sau

Định lý 4.17 (Vasile Cirtoaje).Với hàm lồif(x)trênI(a, b)vàa1, a2, , anI(a, b), ta có bất đẳng thức sau

f(a1) +f(a2) +· · ·+f(an) +n(n−2)f

a1+a2+· · ·+an n

>(n−1)(f(b1) +f(b2) +· · ·+f(bn)) (4.43) trong đó bi = n−11 P

j6=i

aj với mọi i.

Chứng minh. Không tổng quát, ta coi n>3 a1 6a2 6· · ·6an Khi tồn số tự nhiên m cho16m6n−1và

(197)

trong ˆa= (a1+· · ·+an)/n Ta có

b1 >· · ·>bm >a>bm+1 >· · ·>bn.

Dễ thấy điều cần chứng minh suy từ hai bất đẳng thức sau

f(a1) +f(a2) +· · ·+f(am) +n(nm−1)fa)

> (n−1)(f(bm+1) +f(bm+2) +· · ·+f(bn)) f(am+1) +f(am+2) +· · ·+f(an) +n(m−1)f(a)

> (n−1)(f(b1) +f(b2) +· · ·+f(bm))

Để chứng minh bất đẳng thức trên, ta áp dụng bất đẳng thức Jensen hàm lồi

f(a1) +f(a2) +· · ·+f(am) + (nm−1)fa)>(n−1)fb),

ˆb= a1+a2+· · ·+am+ (nm−1)ˆa

n−1 .

Vậy ta phải chứng minh

(nm−1)fa) +fb)>f(bm+1) +f(bm+2) +· · ·+f(bn).

ˆ

a>bm+1 >bm+2>· · ·>bn

(nm−1)ˆa+ ˆb=bm+1+bm+2+· · ·+bn, ta thấy ngay−→An−m = [ˆa, ,ˆa,ˆb]xa

− →

Bn−m = [bm+1, bm+2, , bn] Vậy bất đẳng thức (??) suy từ định lý Karamata

Bất đẳng thức(??) chứng minh tương tự cách sử dụng bất đẳng thức Jensen quen biết

f(am+1) +f(am+2) +· · ·+f(an) + (m−1)f(a)

n−1 >f(c),

ứng dụng cho

f(c) + (m−1)f(a)>f(b1) +f(b2) +· · ·+f(bm),

c= am+1+am+2+· · ·+an+ (m−1)a

(198)

Bất đẳng thức cuối suy từ bất đẳng thức Karamata, b1 >· · ·>bm>a

c+ (m−1)a =b1+b2+· · ·+bm, và −→Cm = [c, a, , a] xa − → Dm =

[b1, b2, , bm]

Định lý 4.18 (Vasile Cirtoaje). Cho a1, a2, , anI(α, β) và hàm lồi f(x) trên I(α, β) Khi đó, ta ln có bất đẳng thức sau

(n−2)(f(a1) +f(a2) +· · ·+f(an)) +nf

a1+a2+· · ·+an n

>2 X

1 i<j n

fai+aj

2

Chứng minh. Ta chứng minh phương pháp quy nạp

Với n = 2, ta thu đẳng thức Giả thiết n > bất đẳng thức vớin−1 Ta chứng minh vớin.

Giả sửa= (a1+a2+· · ·+an)/nvà giả sửx= (a1+a2+· · ·+an−1)/(n−1) Theo giả thiết quy nạp

(n−3)(f(a1) +f(a2) +· · ·+f(an−1)) + (n−1)f(x)>2

X

1 i<j n−1

fa1+aj

2

.

Vậy cần

f(a1) +f(a2) +· · ·+f(an−1) + (n−2)f(an) +nf(a)

>(n−1)f(x) +

n−1

X

i=1

fai+an

2

. Từ bất đẳng thức Jensen, ta có

f(a1) +f(a2) +· · ·+f(an−1)>(n−1)f(x). Vậy nên

(n−2)f(an) +nf(a)>2 n−1

X

i=1

fai+an

2

,

(n−2)an+na= n−1

X

i=1

ai+an

(199)

ta sử dụng bất đẳng thức Karamata cho hai trường hợp Trường hợp 1.Khi2a>min{a1, a2, , an}+ max{a1, a2, , an} Không tổng quát, coia1>a2>· · ·>an Vậy thìa>

a1+an

2 Theo bất

đẳng thức Karamata, ta cần chứng minh an6min{

a1+an

2 ,

a2+an

2 ,· · ·,

an−1+an

2 }

a>max{a1+an

2 ,

a2+an

2 ,· · ·,

an−1+an

2 }

Điều kiện thứ hiển nhiên, điều kiện thứ hai suy từa> a1+an

2

Trường hợp 2.Khi2a <min{a1, a2, , an}+ max{a1, a2, , an} Không giảm tổng quát coia1 6a2 6· · ·6an Khi đóa6

a1+an

2 Theo bất

đẳng thức Karamata, ta cần chứng minh a6min{a1+an

2 ,

a2+an

2 ,· · ·,

an−1+an

2 }

an>max{a1+an

2 ,

a2+an

2 ,· · ·,

an−1+an

2 } Điều kiện thứ hai hiển nhiên

và điều kiện thứ suy từ a6 a1+an

2

Sau đây, ta mô tả số áp dụng trực tiếp

Bài toán 4.13. Xét số dươnga1, a2, , an thoả mãn điều kiệna1+a2+· · ·+ an=n Khi đó, ta ln có

(na1)(na2)· · ·(nan)>(n−1)n(a1a2· · ·an)

1

n−1.

Giải.Chỉ cần áp dụng định lý 4.18 với hàm lồif(x) =− lnx ứng vớix >0, ta có điều phải chứng minh

Nhận xét 4.7. Với a1+a2+· · ·+an =n kéo theoa1a2 an61, ta thấy bất đẳng thức trung bình cộng trung bình nhân sắc bất đẳng thức

(na1)(na2)· · ·(nan)>(n−1)na1a2· · ·an. Thật vậy, từ bất đẳng thức

na1 =a2+a3+· · ·+an>(n−1)(a2a3· · ·an)n−11,

na2 =a1+a3+· · ·+an>(n−1)(a1a3· · ·an)

1

n−1,

nan=a1+a2+· · ·+an−1 >(n−1)(a1a2· · ·an−1)

1

(200)

ta có điều phải chứng minh

Bài tốn 4.14. Xét số dươnga1, a2, , an và đặtbi= n−11

P

j6=i

aj ứng với mọi i Chứng minh đó

b1 a1

+ b2

a2

+· · ·+ bn

an

> a1

b1

+a2

b2

+· · ·+an

bn

. (4.44)

Giải.Giả sử a= (a1+a2+· · ·+an)/n Sử dụng hệ thức

(n−1)bi ai

= na

ai

−1 ai bi

= na

bi

n+

với i= 1,2, , n, ta thấy(4.44)tương đương với bất đẳng thức

1

a1

+

a2

+· · ·+

an

+n(n−2)

a >(n−1)

1

b1

+

b2

+· · ·+

bn

. Bất đẳng thức dễ dàng nhận từ định lý 4.18 ứng dụng cho hàm lồi

f(x) =

x, x >0.

Bài toán 4.15. Xét số dương x1, x2, , xn thoả mãn điều kiện x1+x2+· · ·+xn=

x1

+

x2

+· · ·+

xn

. (4.45)

Chứng minh đó

1

1 + (n−1)x1 +

1

1 + (n−1)x2 +· · ·+

1

1 + (n−1)xn >1 (4.46) Giải.Bất đẳng thức cần chứng minh suy từ(4.44)theo cách sau Giả thiết ngược lại

1 + (n−1)x1

+

1 + (n−1)x2

+· · ·+ 1 + (n−1)xn

<1. (4.47) Ta chứng minh rằng(4.45)không thoả mãn Giả sử

ai =

1 + (n−1)xi

, i= 1,2, , n. Nhận xét rằngai>0và

xi =

1−ai

Ngày đăng: 08/02/2021, 17:02

Từ khóa liên quan

Tài liệu cùng người dùng

Tài liệu liên quan